Anda di halaman 1dari 137

Shared by: www.catatanmatematika.

com

Olimpiade Sains Nasional Bidang Matematika SMA/MA


Seleksi Tingkat Kota/Kabupaten
Tahun 2010

m
co
Soal :

a.
1. Diketahui bahwa ada tepat 1 bilangan asli n sehingga n2 + n + 2010 merupakan kuadrat
sempurna. Bilangan asli n tersebut adalah ⋅⋅⋅⋅⋅⋅⋅

ik
at
2. Bilangan bulat yang memenuhi pertidaksamaan x4 ≤ 8x2 − 16 sebanyak ⋅⋅⋅⋅⋅

em
3. Pasangan bilangan asli (x, y) yang memenuhi 2x + 5y = 2010 sebanyak ⋅⋅⋅⋅⋅⋅

at
4. Diberikan segitiga ABC, AB = AC. Jika titik P diantara A dan B sedemikian rupa sehingga AP =
PC = CB, maka besarnya sudut A adalah ⋅⋅⋅⋅⋅⋅
m
5. Nilai n terkecil sehingga bilangan
an

20102010
14424 ...2010
43
n buah 2010
habis dibagi 99 adalah ⋅⋅⋅⋅⋅
at
at

6. Perempat final Liga Champion 2010 diikuti 8 team A, B, C, D, E, F, G dan H yang bertemu
seperti tampak dalam undian berikut
.c
w
w
//w
s:

Setiap team mempunyai peluang 1


2 untuk melaju ke babak berikutnya. Peluang kejadian A
bertemu G di final dan pada akhirnya A juara adalah ⋅⋅⋅⋅⋅⋅
tp
ht

7. Polinom P(x) = x3 − x2 + x − 2 mempunyai tiga pembuat nol yaitu a, b, dan c. Nilai dari
a3 + b3 + c3 adalah ⋅⋅⋅⋅⋅⋅⋅⋅

1
Shared by: www.catatanmatematika.com

8. Jika a dan b bilangan bulat sehingga 2010 + 2 2009 merupakan solusi kuadrat
x2 + ax + b = 0 maka nilai a + b adalah ⋅⋅⋅⋅⋅

m
9. Banyaknya himpunan X yang memenuhi
{1, 2, 3, ⋅⋅⋅, 1000} ⊆ X ⊆ {1, 2, 3, ⋅⋅⋅, 2010}

co
adalah ⋅⋅⋅⋅

a.
10. Diketahui grid berukuran 4 x 8. Jika langkah yang dimungkinkan Kanan, Kiri, Atas, dan Bawah.
Cara menuju B dari A dalam 8 langkah atau kurang ada sebanyak ⋅⋅⋅ (A adalah titik pada ujung

ik
kanan atas pada kotak paling kiri bawah, sedangkan B adalah titik pada ujung kiri bawah pada
kotak paling kanan atas)

at
em
at
11. Diberikan segitiga ABC; AC : CB = 3 : 4. Garis bagi luar sudut C memotong perpanjangan BA di
P (A terletak antara P dan B). Perbandingan PA : AB adalah ⋅⋅⋅⋅⋅
m
an
12. Misalkan S menyatakan himpunan semua faktor positif dari 20102. Sebuah bilangan diambil
secara acak dari S. Peluang bilangan yang terambil habis dibagi 2010 adalah ⋅⋅⋅⋅⋅
at

13. Diketahui p adalah bilangan prima sehingga terdapat pasangan bilangan bulat positif (x, y)
at

yang memenuhi x2 + xy = 2y2 + 30p. Banyaknya pasangan bilangan bulat positif (x, y) yang
memenuhi ada sebanyak ⋅⋅⋅⋅⋅⋅
.c

14. Pada sebuah persegi panjang berukuran 25 x 20 akan dibuat bujursangkar sehingga menutupi
w

seluruh bagian persegi panjang tersebut. Berapa banyak bujursangkar yang mungkin dapat
dibuat ?
w
//w

15. AB, BC dan CA memiliki panjang 7, 8, 9 berturut-turut. Jika D merupakan titik tinggi dari B,
tentukan panjang AD.
s:

16. Jika −5x + 2000 merupakan sisa pembagian suku banyak P(x) oleh x2 − x − 2, maka sisa
tp

pembagian P(x) oleh x + 2 adalah ⋅⋅⋅⋅⋅


ht

17. Diketahui n adalah bilangan asli. Jika himpunan penyelesaian dari


n
xx ≤ x
2
n x2

2
Shared by: www.catatanmatematika.com

adalah {x⏐0 < x ≤ 5


216 }, maka n = ⋅⋅⋅⋅⋅⋅

18. Misalkan persegi 4 x 4 akan diberi warna hitam dan putih pada tiap kotaknya. Cara pewarnaan

m
sedemikian sehingga warna hitam hanya diberikan pada 3 kotak dan sisanya warna putih
sebanyak ⋅⋅⋅⋅⋅⋅ (Pewarnaan dianggap sama jika didapat dari hasil rotasi yang sama terhadap
persegi 4 x 4)

co
19. Nilai x yang memenuhi 0 ≤ x ≤ π dan

a.
1
( )=2
2010
2 cos( 2x ) cos( 4x )L cos ( )x

ik
x 2 2010
sin 2 2010
adalah ⋅⋅⋅⋅

at
em
20. Diketahui segitiga ABC siku-siku di A, dan pada masing-masing sisi dibuat setengah lingkaran ke
arah keluar. Jika luas setengah lingkaran pada sisi AB dan AC adalah 396 dan 1100, berturut-
turut, maka luas setengah lingkaran pada sisi BC adalah ?

at
. m
an
at
at
.c
w
w
//w
s:
tp
ht

3
Shared by: www.catatanmatematika.com

Solusi Olimpiade Matematika Tk Kabupaten/Kota 2010

1. Misalkan n2 + n + 2010 = k2 untuk suatu bilangan asli k.


n2 + n + 2010 − k2 = 0 yang merupakan persamaan kuadrat dalam n.
Karena n bilangan bulat maka diskriminan persamaan tersebut harus merupakan bilangan
kuadrat sempurna.

m
12 − 4(1)(2010 − k2) = m2 untuk suatu bilangan asli m.
8039 = 4k2 − m2 = (2k + m)(2k − m)

co
Karena 8039 merupakan bilangan prima maka
2k + m = 8039 dan 2k − m = 1
Maka 4k = 8040 sehingga k = 2010 dan m = 4019

a.
Jadi n2 + n + 2010 = 20102
(n − 2009)(n + 2010) = 0

ik
∴ Jadi, bilangan asli n yang memenuhi adalah n = 2009.

at
2. x4 ≤ 8x2 − 16

em
(x2 − 4)2 ≤ 0
Karena bilangan kuadrat tidak mungkin negatif maka penyelesaian ketaksamaan tersebut adalah
x2 − 4 = 0
Bilangan bulat x yang memenuhi adalah x = 2 atau x = −2.

at
∴ Jadi, bilangan bulat x yang memenuhi ada sebanyak 2.
m
3. 2x + 5y = 2010 untuk pasangan bilangan asli (x, y)
an
Karena 5y dan 2010 habis dibagi 5 maka x habis dibagi 5 sehingga x = 5a dengan a ∈ N.
Karena 2x dan 2010 habis dibagi 2 maka y habis dibagi 2 sehingga y = 2b dengan b ∈ N.
10a + 10b = 2010
at

a + b = 201
Karena a, b ∈ N maka ada 200 pasangan (a, b) yang memenuhi sehingga ada 200 pasangan (x, y)
at

yang memenuhi.
∴ Jadi, banyaknya pasangan bilangan asli (x, y) yang memenuhi ada sebanyak 200.
.c

4. Misalkan besarnya sudut A = α


w
w
//w
s:

Karena AP = PC maka ∠ACP = α sehingga ∠BPC = 2α.


tp

Karena PC = CB maka ∠CBP = 2α sehingga ∠PCB = 180o − 4α


Karena AB = AC maka ∠CBP = ∠ACB = ∠ACP + ∠PCB
ht

2α = (α) + (180o − 4α)


α = 36o
∴ Jadi, besarnya sudut A adalah 36o.

SMA Negeri 5 Bengkulu Eddy Hermanto, ST

4
Shared by: www.catatanmatematika.com

Olimpiade Matematika Tk Kabupaten/Kota 2010

5. Misalkan M = 20102010
14424 43 habis dibagi 99.
...2010
n buah 2010
Karena M habis dibagi 99 maka M habis dibagi 9 dan 11.
Jumlah angka-angka M = 3n yang harus habis dibagi 9 sebab M habis dibagi 9.

m
Selisih jumlah angka pada posisi genap dan posisi ganjil dari M sama dengan 3n yang harus habis
dibagi 11 sebab M habis dibagi 11.

co
Jadi, 3n habis dibagi 9 dan 11.
Nilai terkecil n yang memenuhi adalah 33.
∴ Jadi, nilai terkecil n yang memenuhi adalah 33.

a.
ik
6. Hanya ada 5 pertandingan yang berpengaruh sehingga tercapai hasil A bertemu G di final dan A
menjadi juara yaitu A mengalahkan B, A mengalahkan pemenang C atau D, G mengalahkan H, G

at
mengalahkan E atau F dan A mengalahkan G.
Pada masing-masing pertandingan, peluang salah satu tim tertentu memenangkan pertandingan
( 12 )5 .

em
adalah 1
2 . Maka peluang A mengalahkan G di final adalah
∴ Jadi, peluang A mengalahkan G di final adalah 1
32 .

at
7. Polinom P(x) = x3 − x2 + x − 2 mempunyai tiga pembuat nol yaitu a, b dan c. Maka
a+b+c=1
m
ab + ac + bc = 1
abc = 2
an

Alternatif 1 :
(a + b + c)3 = a3 + b3 + c3 + 3a2b + 3a2c + 3ab2 + 3ac2 + 3b2c + 3bc2 + 6abc
(a + b + c)3 = a3 + b3 + c3 + 3(ab + ac + bc)(a + b + c) − 3abc
at

13 = a3 + b3 + c3 + 3(1)(1) − 3(2)
a3 + b3 + c3 = 4
at

Alternatif 2 :
Karena a, b, dan c adalah akar-akar persamaan x3 − x2 + x − 2 = 0 maka
.c

a3 − a2 + a − 2 = 0
b3 − b 2 + b − 2 = 0
w

c3 − c2 + c − 2 = 0
Jumlahkan ketiga persamaan didapat
w

a3 + b3 + c3 − (a2 + b2 + c2) + (a + b + c) − 6 = 0
a3 + b3 + c3 = (a + b + c)2 − 2(ab + ac + bc) − (a + b + c) + 6
//w

a3 + b3 + c3 = 12 − 2(1) − 1 + 6
a3 + b3 + c3 = 4
∴ Jadi, nilai a3 + b3 + c3 = 4.
s:
tp

8. 2010 + 2 2009 = 2009 + 1


Alternatif 1 :
ht

2009 + 1 merupakan solusi persamaan x2 + ax + b = 0, maka


( 2009 + 1)2 + a( 2009 + 1) + b = 0

SMA Negeri 5 Bengkulu Eddy Hermanto, ST

5
Shared by: www.catatanmatematika.com

Olimpiade Matematika Tk Kabupaten/Kota 2010

2010 + 2 2009 + a 2009 + a + b = 0


Karena a dan b bilangan bulat maka
2 2009 + a 2009 = 0 dan 2010 + a + b = 0
Didapat a = −2 dan b = −2008

m
Maka a + b = −2010
Alternatif 2 :

co
x2 + ax + b = 0
x1, 2 = − a ± a 2 − 4b
2

a.
Maka − a + a 2 − 4b
2 = 2009 + 1

ik
−a + a 2 − 4b = 2 2009 + 2
Karena a dan b bilangan bulat maka −a = 2 sehingga a = −2

at
a2 − 4b = 4 ⋅ 2009
1 − b = 2009 sehingga b = −2008

em
Maka a + b = −2 − 2008 = −2010.
∴ Jadi, a + b = −2010.

at
9. {1, 2, 3, ⋅⋅⋅, 1000} ⊆ X ⊆ {1, 2, 3, ⋅⋅⋅, 2010}
Jika H memiliki k elemen maka banyaknya himpunan bagian dari H adalah 2k.
m
Elemen 1, 2, 3, ⋅⋅⋅, 1000 haruslah merupakan elemen dari X. Persoalannya sama saja dengan
X ⊆ {1001, 1002, 1003, ⋅⋅⋅, 2010}
Banyaknya himpunan bagian dari X tersebut adalah 21010.
an

∴ Jadi, banyaknya himpunan X yang memenuhi adalah 21010.


at

10. Jalan terpendek dari A ke B adalah jika jalannya hanya Kanan dan Atas saja. Ukuran grid 4 x 7.
at
.c

Banyaknya langkah terpendek adalah 7 sebab banyaknya langkah ke Kanan ada 5 dan ke Atas ada
w

2. Tidak ada jalan dengan banyaknya langkah tepat 8 sebab jika berjalan ke Kiri atau ke Bawah
sekali, maka banyaknya langkah terpendek yang diperlukan adalah 9.
w

Jadi cukup dihitung banyaknya jalan dengan banyaknya langkah tepat 7.


Alternatif 1 :
//w

Misalkan langkah ke Kanan diberi tanda 1 dan langkah ke Atas diberi tanda 2. Maka persoalannya
sama dengan banyaknya susunan angka-angka 1111122, yaitu melangkah ke Kanan sebanyak 5
kali dan melangkah ke Atas sebanyak 2 kali.
s:

Banyaknya susunan bilangan 1111122 sama dengan 5!7⋅2! ! = 21.


Maka banyaknya cara melangkah dari A ke B sama dengan 21.
tp

Alternatif 2 :
Banyaknya langkah ada 7. Dua di antaranya adalah ke Atas dan 5 ke Kanan. Maka persoalan ini
ht

adalah sama dengan menempatkan 5 obyek identik pada 7 tempat berbeda.


Maka banyaknya cara melangkah dari A ke B sama dengan 7C2 = 21.
∴ Jadi, banyaknya cara melangkah dari A ke B sama dengan 21.

SMA Negeri 5 Bengkulu Eddy Hermanto, ST

6
Shared by: www.catatanmatematika.com

Olimpiade Matematika Tk Kabupaten/Kota 2010


Catatan : Ada perbedaan antara kata-kata pada soal dan gambar pada soal. Berdasarkan kata-
kata pada soal maka ukuran gridnya adalah 4 x 8 sedangkan pada gambar ukuran gridnya adalah
4 x 7. Kunci jawaban dari pusat mengacu pada gambar. Jika yang diacu adalah kata-kata pada
soal maka jawabannya adalah 8C2 = 28.

m
11.

co
a.
ik
PC adalah garis bagi ∆ABC sehingga berlaku

at
CB
AC = PB
PA

em
4
3 = PA
PB

Maka dapat dimisalkan PB = 4k dan PA = 3k sehingga AB = k


Maka PA : AB = 3k : k = 3 : 1
∴ Jadi, perbandingan PA : AB adalah 3 : 1.

at
m
12. 20102 = 22 ⋅ 32 ⋅ 52 ⋅ 672.
2010 = 2 ⋅ 3 ⋅ 5 ⋅ 67
an
Faktor-faktor positif dari 20102 akan berbentuk 2a ⋅ 3b ⋅ 5c ⋅ 67d dengan 0 ≤ a, b, c, d ≤ 2 dengan
a, b, c, d bilangan bulat.
Banyaknya faktor positif 20102 = 3 ⋅ 3 ⋅ 3 ⋅ 3 = 81
at

Agar faktor tersebut merupakan kelipatan 2010 maka 1 ≤ a ≤ 2, 1 ≤ b ≤ 2, 1 ≤ c ≤ 2, 1 ≤ d ≤ 2.


Banyaknya faktor positif 20102 yang merupakan kelipatan 2010 = 2 ⋅ 2 ⋅ 2 ⋅ 2 = 16.
at

∴ Jadi, peluang bilangan yang terambil habis dibagi 2010 adalah 16 81


.c

13. x2 + xy = 2y2 + 30p


w

(x − y)(x + 2y) = 30p


Jika x dan y keduanya tidak memiliki sisa yang sama jika dibagi 3 maka x − y dan x + 2y
w

keduanya tidak ada yang habis dibagi 3. Padahal 30p habis dibagi 3. Jadi, x dan y haruslah
keduanya memiliki sisa yang sama jika dibagi 3.
//w

Akibatnya x − y dan x + 2y masing-masing habis dibagi 3 sehingga 30p harus habis dibagi 9.
Karena 30 habis dibagi 3 tetapi tidak habis dibagi 9 maka p harus habis dibagi 3.
Karena p adalah bilangan prima maka p = 3.
(x − y)(x + 2y) = 90
s:

Karena x + 2y ≥ x − y maka akan ada 2 kasus.


* x + 2y = 30 dan x − y = 3
tp

Didapat x = 12 dan y = 9
* x + 2y = 15 dan x − y = 6
ht

Didapat x = 9 dan y = 3
Maka pasangan bilangan bulat positif (x, y) yang memenuhi adalah (12, 9) dan (9, 3).
∴ Jadi, banyaknya pasangan bilangan bulat positif (x, y) yang memenuhi ada sebanyak 2.

SMA Negeri 5 Bengkulu Eddy Hermanto, ST

7
Shared by: www.catatanmatematika.com

Olimpiade Matematika Tk Kabupaten/Kota 2010

14. Perhatikan gambar.

m
co
a.
ik
Banyaknya persegi dengan ukuran 1 x 1 ada sebanyak 25 x 20 = 500

at
Banyaknya persegi dengan ukuran 2 x 2 ada sebanyak 24 x 19 = 456
Banyaknya persegi dengan ukuran 3 x 3 ada sebanyak 23 x 18 = 414

em
M
Banyaknya persegi dengan ukuran 20 x 20 ada sebanyak 6 x 1 = 6
Banyaknya semua persegi yang ada = 500 + 456 + 414 + ⋅⋅⋅ + 6 = 3920.
∴ Jadi, banyaknya semua persegi yang ada = 3920.

at
m
15. Perhatikan gambar.
an
at
at

Alternatif 1 :
s = 12 (a + b + c) = 12
.c

Dengan rumus Heron didapat


[ABC] = s(s − a )(s − b )(s − c ) = 12 5
w

1
2 ⋅ AC ⋅ BD = 12 5
w

9 ⋅ BD = 24 5 sehingga BD = 8
3 5
//w

2 2 2
AD = AB − BD = 49 − 320
9 = 121
9

AD = 113
Alternatif 2 :
s:

a2 = b2 + c2 − 2bc cos A
82 = 92 + 72 − 2 ⋅ 9 ⋅ 7 cos A
tp

cos A = 11
21

AD = AB cos A = 7 ⋅ 11
ht

21

AD = 11
3

∴ Jadi, panjang AD = 11
3

SMA Negeri 5 Bengkulu Eddy Hermanto, ST

8
Shared by: www.catatanmatematika.com

Olimpiade Matematika Tk Kabupaten/Kota 2010

16. P(x) = Q(x) ⋅ (x + 2)(x − 1) − 5x + 2000


P(−2) = 0 − 5(−2) + 2000 = 2010
P(−2) menyatakan sisa jika P(x) dibagi x + 2.
∴ Jadi, sisa jika P(x) dibagi x + 2 adalah 2010.

m
Catatan : Soal aslinya adalah P(x) dibagi x2 − x − 2 bersisa −5x + 2000, tetapi Penulis
berkeyakinan seharusnya adalah P(x) dibagi x2 + x − 2 bersisa −5x + 2000.

co
a.
n
xx ≤ x
2
x2
dipenuhi oleh {x⏐0 < x ≤
n
17. Penyelesaian 5
216 } untuk suatu bilangan asli n.
x2 2

ik
x n
≤x xn

• Jika x ≥ 1 maka

at
2
x2
n ≤ xn
n

em
x ≤ n 2n−2
• Jika 0 < x < 1 maka
2
x2
n ≥ xn

at
n
x ≥ n 2n−2
n
Karena n bilangan asli maka n 2 n − 2 ≥ 1.
m
2

2
Jika x < 0 maka karena xn dan x n tidak dapat dipastikan merupakan bilangan rasional maka
an
tidak ada definisi jika x < 0.
n
Maka penyelesaian ketaksamaan tersebut adalah 1 ≤ x ≤ n 2 n − 2 .
3
at

Karena penyelesaian ketaksamaan tersebut adalah 0 < x ≤ 5


216 = 6 5 maka
n = 6k dan 2 nn− 2 = 53k untuk suatu bilangan asli k.
at

6k ⋅ 5k = 6 ⋅ 6k − 6
61-k = 6 − 5k
.c

Jika k > 1 maka ruas kiri merupakan pecahan sedangkan ruas kanan merupakan bilangan bulat
sehingga tidak akan tercapai kesamaan.
Jika k = 1 maka 1 = 6 − 5(1) yang memenuhi persamaan.
w

Jadi, n = 61 = 6
w

∴ Jadi, nilai bilangan asli n yang memenuhi adalah n = 6.


Catatan : Terbukti bahwa dalam batas 0 < x < 1 tidak memenuhi ketaksamaan. Maka pada soal,
//w

himpunan penyelesaian ketaksamaan tersebut seharusnya {x⏐1 ≤ x ≤ 5


216 }

18. Perhatikan gambar. Rotasi yang dimaksud adalah 90o, 180o dan 270o sehingga jika sebuah petak
s:

berwarna hitam dirotasi akan timbul 3 petak lain yang berbeda dengan petak semula. Jadi, jika
3 petak berwarna hitam dirotasikan maka tidak akan ada hasilnya yang menempati ketiga petak
tp

semula.
ht

SMA Negeri 5 Bengkulu Eddy Hermanto, ST

9
Shared by: www.catatanmatematika.com

Olimpiade Matematika Tk Kabupaten/Kota 2010


Persoalan ini sama saja dengan banyaknya memilih 3 petak dari 16 petak yang ada = 16C3 lalu
hasilnya dapat dibagi ke dalam 16C4 : 4 kelompok dengan masing-masing kelompok merupakan
rotasi dari petak-petak lainnya.
16 C3
Maka banyaknya cara pewarnaan = 4 = 140.

m
∴ Jadi, banyaknya cara pewarnaan = 140.

co
19. sin α cos α = 1
2 sin 2α
1
2 cos( 2x )cos( 4x )L cos ( )

a.
=2
sin ( )
2010 x
x 2 2010
2 2010

( 2x ) cos ( 4x ) ⋅⋅⋅ cos (2 x ) sin ( )

ik
1 = 22010 2 ⋅ cos 2010
x
2 2010

⋅ cos ( 2x ) cos ( 4x ) ⋅⋅⋅ sin (2 x )

at
1 = 22009 2 2009

1 = 22008 2 ⋅ cos ( 2x ) cos ( 4x ) ⋅⋅⋅ sin (2 x )

em
2008

Sehingga didapat
1 = 2 sin x
sin x = sin π4

at
π 3π
Maka x = 4 atau x = 4
π
m 3π
∴ Jadi, nilai x yang memenuhi adalah x = 4 atau x = 4
an

20. Perhatikan gambar.


at
at
.c
w

Luas setengah lingkaran AB = 1


πc2 = 396.
w

Luas setengah lingkaran AC = 1


8 πb2 = 1100.
//w

Luas setengah lingkaran BC = 1


8 πa2 = 1
8 π(b2 + c2) = 1100 + 396 = 1496.
∴ Jadi, luas setengah lingkaran pada sisi BC sama dengan 1496.
Catatan : Kunci dari pusat terhadap persoalan ini adalah 704 yang menurut Penulis,
s:

kesalahannya ada pada segitiga ABC siku-siku di A yang mungkin seharusnya di B atau C.
tp
ht

SMA Negeri 5 Bengkulu Eddy Hermanto, ST

10
Shared by: www.catatanmatematika.com

Olimpiade Sains Nasional Bidang Matematika SMA/MA


Seleksi Tingkat Kota/Kabupaten
Tahun 2011

m
co
Soal :

a.
1. Misalkan kita menuliskan semua bilangan bulat 1, 2, 3, ..., 2011. Berapa kali kita menuliskan
angka 1 ?

ik
at
2. Sekelompok orang akan berjabat tangan. Setiap orang hanya dapat melakukan jabat tangan
sekali. Tidak boleh melakukan jabat tangan dengan dirinya sendiri. Jika dalam sekelompok
orang terdapat 190 jabat tangan, maka banyaknya orang dalam kelompok tersebut ada

em
berapa?

at
3. Dalam suatu permainan, jika menang mendapat nilai 1 dan jika kalah mendapat nilai −1. Jika
(a, b) menyatakan a putaran permainan dan b menyatakan total nilai seorang pemain, maka
seluruh kemungkinan (a, b) pada putaran ke-20 adalah ⋅⋅⋅⋅⋅⋅
m
an
4. Di lemari hanya ada 2 macam kaos kaki yaitu kaos kaki berwarna hitam dan putih. Ali, Budi
dan Candra berangkat di malam hari saat mati lampu dan mereka mengambil kaos kaki secara
acak di dalam lemari dalam kegelapan. Berapa kaos kaki minimal harus mereka ambil untuk
at

memastikan bahwa akan ada tiga pasang kaos kaki yang bisa mereka pakai ? (Sepasang kaos
kaki harus memiliki warna yang sama).
at

5. Misalkan batas suatu kebun dinyatakan dalam dalam bentuk persamaan x + y = 4 dengan (x, y)
.c

bilangan bulat tak negatif dan dinyatakan dalam satuan km. Pemilik kebun setiap pagi biasa
w

berjalan kaki dengan kecepatan 2 2 km/jam searah jarum jam. Jika pemilik kebun pada
pukul 06.00 berada pada koordinat (0, 4), dimanakah pemilik kebun pada pukul 07.00 ?
w
//w

6. Ani mempunyai sangat banyak dadu dengan ukuran 3 cm x 3 cm x 3 cm. Jika ia memasukkan
dadu- dadu tersebut ke dalam sebuah kardus dengan ukuran 50 cm x 40 cm x 35 cm maka
berapa banyak dadu yang bisa masuk ke dalam kardus tersebut ?
s:

7. Bilangan asli disusun seperti bagan di bawah ini.


tp

1
2 3 4
ht

5 6 7 8 9
10 11 12 13 14 15 16
...
Besar bilangan ketiga dalam baris ke-50 adalah ....

11
Shared by: www.catatanmatematika.com

12
8. Jumlah dari seluruh solusi persamaan 4
x = adalah ....
7−4 x

m
9. Enam dadu dilempar satu kali. Probabilitas banyaknya mata yang muncul 9 adalah ....

co
10. Luas daerah di dalam lingkaran x2 + y2 = 212 tetapi di luar lingkaran x2 + (y − 7)2 = 142 dan

a.
x2 + (y + 7)2 = 142 adalah ⋅⋅⋅⋅⋅⋅⋅⋅

ik
11. Tentukan semua bilangan bulat positif p sedemikian sehingga p, p + 8, p + 16 adalah bilangan
prima.

at
em
12. Jika A = 5x + 5−x dan B = 5x – 5−x maka A2 – B2 adalah ⋅⋅⋅⋅⋅⋅⋅

13. Diketahui segitiga ABC, titik D dan E berturut-turut pada sisi AB dan AC, dengan panjang

at
AD = 12 BD dan AE = 12 CE. Garis BE dan CD berpotongan di titik F. Diketahui luas segitiga
ABC = 90 cm2 maka luas segiempat ADFE adalah ⋅⋅⋅⋅⋅⋅⋅⋅
m
an

14. Ada berapa banyak bilangan bulat positif berlambang “abcde” dengan a < b ≤ c < d < e ?
at

15. Bilangan asli terkecil lebih dari 2011 yang bersisa 1 jika dibagi 2,3,4,5,6,7,8,9,10 adalah ⋅⋅⋅⋅⋅
at

16. Bilangan bulat positif terkecil a sehingga 2a + 4a + 6a + ... + 200a merupakan kuadrat
.c

sempurna adalah ....


w

17. Misalkan A dan B adalah sudut-sudut lancip yang memenuhi


w

tan (A + B) = 12 dan tan (A − B) = 13 maka besar sudut A adalah ....


//w

18. Jika ax + 2y = 3 dan 5x + by = 7 menyatakan persamaan garis yang sama maka a + b = ⋅⋅⋅⋅
s:

19. Terdapat 5 orang pria dan 5 orang wanita duduk dalam sederetan kursi secara random. Berapa
banyaknya cara untuk menduduki kursi tersebut dengan syarat tidak boleh ada yang duduk
tp

berdampingan dengan jenis kelamin yang sama ?


ht

20. Ada berapa faktor positif dari 27355372 yang merupakan kelipatan 10 ?

12
Shared by: www.catatanmatematika.com

Solusi
Olimpiade Sains Nasional Bidang Matematika SMA/MA
Seleksi Tingkat Kota/Kabupaten
Tahun 2011
Waktu : 120 Menit
©Yudi Setiawan, M.Pd., M.Si
SMAN 1 Cikembar Kab. Sukabumi

m
Email : yd_smarsi@yahoo.com

co
1. Misalkan kita menuliskan semua bilangan bulat , 2, 3, ..., smapai dengan 2011. Berapa
kali kita menuliskan angka 1?.

a.
Solusi :

ik
Banyaknya angka 1 yang dituliskan sama dengan banyaknya angka 1 yang muncul dari
barisan bilangan 1, 2, 3, 4, ..., 2011

at
Banyaknya angka 1 pada bilangan 1 angka adalah 1
Banyaknya angka 1 pada bilangan 2 angka adalah

em
9 .1  9
1 .1 0  1 0

at  19
Banyaknya angka 1 pada bilangan 3 angka adalah
9 .1 0 .1  9 0
m
9 .1 .1 0  9 0
an

1 .1 0 .1 0  1 0 0

 280
at

Banyaknya angka 1 pada bilangan 4 angka yang kurang dari 2000 adalah
1.10.10.1  100
at

1.10.1.10  100
1.1.10.10  100
.c

1.10.10.10  1000

w

 1300
Banyaknya angka 1 pada bilangan 2000 - 2011 adalah
w

1.1.2.1  2
//w

1.1.1.2  2

4
Banyaknya angka 1 yang dituliskan adalah 1 + 19 + 280 + 1300 + 4 = 1604
s:

 1604 kali
tp

2. Sekelompok orang akan berjabat tangan. Setiap orang hanya dapat melakukan jabat
ht

tangan sekali. Tidak boleh melakukan jabat tangan dengan diringa sendiri. Jika dalam
sekelompok orang tersebut terdapat 190 jabat tangan, maka banyaknya orang dalam
kelompok tersebut ada berapa?.

Solusi :
n  n  1
Jika terdapat n orang, maka banyaknya jabat tangan adalah
2

13
Solusi OSK Matematika SMA 2011
Shared by: www.catatanmatematika.com SMAN 1 CIKEMBAR KAB. SUKABUMI
B-Right SCHOOL

n  n  1
Sehingga  190
2
n  n  1   2.10.19

n  n  1   20.19
 n  20

m
Banyaknya orang adalah 20 orang
 20 orang

co
3. Dalam suati permainan, jika menang mendapat nilai 1, jika kalah mendapat nilai -1.(a, b)
menyatakan a putaran permainan dan b menyatakan total nilai seorang pemain. Maka

a.
seluruh kemungkinan (a, b) pada putaran ke 20 adalah ...

ik
Solusi :
Misalkan banyaknya menang = x , x  0

at
Banyaknya menang = y, y  0
x y a

em
Sehingga
1. x    1  y  b
Pada putaran ke 20 maka nilai a = 20
x  y  20
x y b
at
m

 2 x  20  b  b  2 x  20
x  y  20 dan x, y merupakan bilangan bulat positif maka x, y  20
an

Karena
Sehingga seluruh kemungkinan (a, b) pada putaran ke 20 adalah
 a , b    20, 2 x  20  , 0  x  20 ada sebanyak 21 pasang
at

  a , b    20, 2 x  20  , 0  x  20
at

4. Dilemari hanya ada dua macam kaos kai, yakni hitam dan putih. Ali, Budi, dan Candra
.c

berangkat di malam hari saat mati lampu, dan mereka mengambil kaus kaki secara acak
dari lemari dalam kegelapan. Berapa kaus kaki minimal yang harus mereka ambil untuk
w

memastikan bahwa akan ada 3 pasang kaus kaki yang bisa mere pakai? (sepasang kaus
kaki harus memiliki warna yang sama).
w

Solusi :
//w

Misalkan (m, n) menyatakan m buah kaos kaki hitam dan n buah kaos kaki putih.
Banyaknya pasangan kaus kaki yang diharapkan adalah 3 pasang,
sehingga m  n  2.3
s:

mn6
tp

namun 6 buah kaus kaki tidak dapat memastikan terdapat 3 pasang kaus kaki, contohnya
adalah (5, 1) hanya diperoleh 2 pasang kaus kaki.
Sehingga m  n  7
ht

m  n  7 maka m  4 atau n  4
Misalkan m  4
 (4, 3) maka terdapat dua pasang kaos kaki hitam dan sepasang kaos kaki putih.
 (5, 2) maka terdapat dua pasang kaos kaki hitam dan sepasang kaos kaki putih.
 m  6, maka terdapat tiga pasng kaos kaki hitam
(untuk n  4 serupa).

14
Solusi OSK Matematika SMA 2011
Shared by: www.catatanmatematika.com SMAN 1 CIKEMBAR KAB. SUKABUMI
B-Right SCHOOL

Dengan demikan diperoleh bahwa banyaknya kaos kaki minimal yang harus mereka
ambil untuk memastikan bahwa akan ada 3 pasang kaos kaki adalah 7
7

5. Misalkan batas suatu kebun dinyatakan dalam bentuk persamaan x  y  400 dengan x, y
dinyatakan dalam satuan meter. Pemilik kebun setiap pagi biasa berjalan kaki berkeliling

m
kebun dengan kecepatan 2 2 km/jam searah jarum jam. Jika pemilik kebun pada pukul
6 berada di koordinat (0, 4), dimanakah posisi pemilik kebun pada pukul..

co
Solusi :
Menurut penulis, terdapat kekeliruan dalam pengetikan naskah soal, yaitu

a.
 Soal tersebut tidak dapat dikerjakan karena posisi pemilik kebun tergantung pada

ik
waktu, sedangkan waktu tidak diketahui (tidak di tulis).
 Persamaan x  y  400 tidak membatasi sebuah daerah tertutup seperti terlihat

at
pada gambar di bawah, sehingga mustahil pemiliki kebun dapat mengelilingi
daerah tersebut. Kecuali ditambah pembatas, yaitu Sumbu X positif dan Sumbu Y

em
positif. Sehingga daerahnya berbentuk segitiga, namun demikian tanda harga
mutlak jadi tidak berguna.
 Titik (0, 4) tidak terletak pada grafik x  y  400 , seperti terlihat pada gambar di
bawah ini
at
m
Y

400
an

300 |x + y| = 400
at

200

100
at

X
.c

-400 -300 -200 -100 100 200 300 400

-100
w

-200
w

-300
//w

-400
s:
tp
ht

Sebaiknya soal seperti berikut


Misalkan batas suatu kebun dinyatakan dalam bentuk persamaan x  y  400 dengan
x, y dinyatakan dalam satuan meter. Pemilik kebun setiap pagi biasa berjalan kaki
berkeliling kebun dengan kecepatan 2 2 km/jam searah jarum jam. Jika pemilik
kebun pada pukul 6 berada di koordinat (0, 400), dimanakah posisi pemilik kebun
pada pukul 06.06

15
Solusi OSK Matematika SMA 2011
Shared by: www.catatanmatematika.com SMAN 1 CIKEMBAR KAB. SUKABUMI
B-Right SCHOOL

400

|x| + |y| = 400


300

200

100

m
X

-400 -300 -200 -100 100 200 300 400

-100

co
-200

a.
-300

-400

ik
Waktu tempuh dari pukul 06.00 sampai dengan pukul 06.06 = 6 menit = 0,1 jam

at
Sehingga jarak tempuhnya adalah = 0,1 jam x 2 2 km/jam = 0, 2 2 km = 200 2 m
Jarak antara titik (0, 400) dan (400, 0) adalah 400 2 m = 2 kali jarak tempuh.

em
Sehingga koordinat pemilik kebun pada pukul 06.06 merupakan titik tengah antara titik
(0, 400) dan (400, 0), yaitu (200, 200)
 (200, 200)

Alternatif lain
at
m
Misalkan koordinat pemilik kebun pada pukul 06.06 adalah (a, b)
Karena pemilik kebun baru berjalan selama 6 menit dan arahnya searah jarum jam, maka
an

jelas bahwa (a, b) akan terletak di kwadran I sehingga |a| = a dan |b| = b
0  a    400  b   200 2
2 2
Maka
at

a  400  800 b  b  80000


2 2 2
at

a  b  80000  800.b  400


2 2 2

Dan (a, b) terletak pada x  y  400 , maka a  b  400


.c

a  b  400

a  b
w

2
 400
2

a  2 ab  b  400
2 2 2
w

a  b  400  2 ab
2 2 2
//w

Sehingga 80000  800 b  400  400  2 ab


2 2

ab  400 b  400  40000


2
s:

b  a  400   400  200


2 2

b  a  400    200   600 


tp

Dengan demikian diperoleh bahwa a = 200 dan b = 200


ht

Koordinat pemilik kebun pada pukul 06.06 adalah (200, 200)


 (200, 200)

16
Solusi OSK Matematika SMA 2011
Shared by: www.catatanmatematika.com SMAN 1 CIKEMBAR KAB. SUKABUMI
B-Right SCHOOL

6. Ani mempunyai sangat banyak dadu dengan ukuran 3 x 3 x 3 cm 3, jika ia memasukkan


dadu-dadu tersebut ke dalam sebuah kardus dengan ukuran 50 x 40 x 35 cm 3, maka
berapa banyak dadu yang bisa masuk ke dalamnya?.

Solusi :
Ukuran Dadu 3 x 3 x 3 cm3
Ukuran Kardus 50 x 40 x 35 cm3

m
Misalkan panjang, lebar, dan tinggi kardus tersebut masing-masing adalah 50 cm, 40 cm
,dan 35 cm.

co
Sehingga
Berdasarkan ukuran panjang, kardus tersebut hanya dapat menampung

a.
 50 : 3  = 16 buah
Berdasarkan ukuran lebar, kardus tersebut hanya dapat menampung

ik
 4 0 : 3  = 13 buah

at
Berdasarkan ukuran tinggi, kardus tersebut hanya dapat menampung
 35 : 3  = 11 buah secara

em
Dengan demikian banyaknya dadu yang dapat dimasukkan kedalam kardus adalah
16 x 13 x 11 = 2288 buah
 2288

7. Bilangan asli disusun seperti bagan di bawah ini


at
m
1
2 3 4
an

5 6 7 8 9
10 11 12 13 14 15 16
at

Solusi :
at

Perhatikan barisan bilangan berikut


B aris ke 1 1
.c

B aris ke 2 2 3 4
w

B aris ke 3 5 6 7 8 9
B aris ke 4 10 11 12 13 14 15 16
w

atau
//w

2
B aris ke 1 1
1 1 1 2
2 2
B aris ke 2 4
2 1 2 2 2 3 2 4
2 2 2 2 2
B aris ke 3 3
s:

3 1 3 2 3 3 3 4 3 5 3 6
2 2 2 2 2 2 2
B aris ke 4 4
tp

Sehingga baris ke n bilangan ke m adalah


 n  1   m , dengan m bilangan asli, m  2 n  1 .
2
ht

Bilangan ketiga pada baris ke 50 adalah  50  1   3  49 2  3


2

 2404
 2404

17
Solusi OSK Matematika SMA 2011
Shared by: www.catatanmatematika.com SMAN 1 CIKEMBAR KAB. SUKABUMI
B-Right SCHOOL

8. Jumlah sdari seluruh solusi persamaan


12
4
x 
7 4
x
Adalah ...

Solusi :

m
12
Perhatikan persamaan 4
x 
7 4
x

co
12
Misalkan 4
x  a, maka a
7a

a.
7 a  a  12
2

a  7 a  12  0
2

ik
 a  3  a  4   0

at
a3 atau a4

em
4
x 3 4
x 4
x  81 x  256
Jumlah dari seluruh solusi adalah 81 + 256 = 337
 337 at
m
9. Enam dadu berbeda dilempar satu kali. Probabilitas banyak mata dadu yang muncul 9
adalah...
an

Solusi :
S : Ruang sampel pelemparan 6 dadu berbeda  N(S) = 66
at

A: Kejadian munculnya jumlah mata dadu 9


Misal xi menyatakan mata dadu yang muncul pada dadu ke-i, maka
at

x1  x 2  x 3  x 4  x 5  x 6  9 dengan 1  x i  4
Kemungkinan dari A adalah
.c

5!
Terdapat mata dadu 4 yaitu 4,1,1,1,1,1  5
4!
w

5!
Terdapat mata dadu 3 yaitu 1,1,1,1,2,3  20
w

3!
5!
 30
//w

Terdapat mata dadu 2 yaitu 1,1,1,2,2,2


2 !2 !
N(A) = 5 + 20 + 30 = 56
N ( A) 56
s:

P(A)   6
N (S ) 6
tp

56
 6
6
ht

18
Solusi OSK Matematika SMA 2011
Shared by: www.catatanmatematika.com SMAN 1 CIKEMBAR KAB. SUKABUMI
B-Right SCHOOL

10. Luas daerah yang didalam lingkaran x 2  y 2  212 tetapi diluar lingkaran
x   y  7   14 dan x 2   y  7   14 2 adalah ....
2 2 2 2

Solusi :
Misalkan Luas daerah lingkaran besar = LB
Y
Luas daerah lingkaran kecil = LK

m
14 L u as d aerah yan g d iarsir  L B – L K

co
 21   14 
2 2
7

X  (2 1  1 4 )(2 1  1 4 ) 
-21 -14 -7 7 14 21

a.
 2 4 5
-7

ik
-14

at
Namun penulis berkeyakinan bahwa ada kesalahan pengetikan dalam naskah soal,

em
seharusnya “Luas daera yang didalam lingkaran x 2  y 2  212 tetapi diluar lingkaran
x   y  7   14 dan x 2   y  7   14 2 adalah ....”
2 2 2 2

14 Misalkan
at
Luas daerah lingkaran besar = LB
Luas daerah lingkaran kecil = LK
m
7 Luas daerah Tembereng = LT
X Perhatikan salah satu lingkaran kecil (gambar 2)
an

-14 -7 7 14 21 Segitiga AOB siku-siku di O, dengan AB = 14 dan AO = 7


-7 AO 7 1
co s  B A O      BAO  60
0
at

-14 AB 14 2
 B A C  2 . B A O   BAO  120
0
at

0
120 1
L . Ju rin g B A C  dan L.  A B C  0
LK A B . A C . sin 120
Gambar 1 360
0
2
.c

1 1
 14  
2 2
14
w

3 4
Y LT = L. Juring B A C  L.  A B C
w

14 1 1
 14  
2 2
14 3
//w

7
A 3 4
B C X 1 1  2
   3  14
-14 -7 O 7 14 21
3 4 
s:

-7

Luas daerah yang diarsir  L B – 2L K + 2L T


tp

-14

1 1  2
 21   2 14   2  3   4
2 2
3  14
ht

Gambar 2  

1 1  2
 21   2 14
2
 2
  2  3   4 3  14
 

19
Solusi OSK Matematika SMA 2011
Shared by: www.catatanmatematika.com SMAN 1 CIKEMBAR KAB. SUKABUMI
B-Right SCHOOL

11. Tentukan semua bilangan bulat positif p sedemikian sehingga p, p +8, dan p + 16 adalah
prima.

Solusi :
p bilangan bulat positif sedemikian sehingga p, p +8, dan p + 16 merupakan bilangan
prima.
Jelas bahwa p harus merupakan bilangan prima

m
Jika p > 3, maka p  6 k  1 atau p  6 k  5
 Jika p  6 k  1 , maka p  8  6 k  1  8  6 k  9  3  2 k  3  bukan bilangan

co
prima.
 Jika p  6 k  5 , maka p  16  6 k  5  16  6 k  21  3  2 k  7  bukan bilangan

a.
prima.

ik
Jika p  3, maka p = 3
 p = 2, maka p  8 dan p  16 bukan bilangan prima.

at
 p = 3, maka p  8  3  8  11 dan p  16  3  16  19 merupakan bilangan

em
prima.
Nilai p sehingga p, p +8, dan p + 16 merupakan bilangan prima adalah 3
 3

12. Jika A  5 x  5  x dan B  5 x  5  x , maka A 2  B 2 adalah ...


at
m
Solusi :
x x
A5 5 dan B  5  5
x x
an

A  B   A  B A  B
2 2

 x x
  5  5    5  5    5 5
x
  5 x  5 x  
at

x x x
  


 2.5
x
  2.5 x
at

1
 2.5 .2.
x
.c

x
5
4
w

 4
w

13. Diketahui  A B C , titik D dan E berturut-turut pada sisi AB dan AC, dengan panjang
//w

1 1
AD  BD dan A E  CE . Garis BE dan CD berpotongan di titik F. Diketahui luas
2 2
ABC adalah 90 cm2. Luas segiempat ADFE adalah...
s:

Solusi :
tp

Misalkan Panjang AE = x
Panjang AD = y
ht

tinggi  A B C melalui titik B = t


tinggi  A F C melalui titik F = s

20
Solusi OSK Matematika SMA 2011
Shared by: www.catatanmatematika.com SMAN 1 CIKEMBAR KAB. SUKABUMI
B-Right SCHOOL

m
D
F

co
a.
A E C

ik
1 1
L ABC  . A C .t dan L ACF  . A C .s
2 2

at
1 1
 .3 x .t  .3 x . s

em
2 2
1 1
 3. x .t  3. x .s
2 2

 3.
1
2
A E .t  3.
at 1
2
A E .s
m
 3 .L  A E B  3. L  A E F
an

Dengan cara yang serupa dapat ditunjukkan bahwa


L  A B C  3. L  A D C dan L  A B F  3. L  A D F
at

1
Sehingga L  A E B  L  A D C  L ABC = 30 cm2
at

3
.c

Misal L  AD F  M maka L  ABF  3 M


L  AEF  N L  AC F  3 N
w
w

Perhatikan segitiga dan


L AEB  L AEF  L ABF dan L  A D C  L  A D F  L  A C F
//w

30  N  3 M (1) 30  M  3 N (2)

N  3M  30
s:

Dari (1) dan (2) diperoleh


3 N  M  30

tp

4 N  4 M  60
N  M  15
ht

L A D F E  L  A D F  L  A E F  N  M  15
2
 15 cm

21
Solusi OSK Matematika SMA 2011
Shared by: www.catatanmatematika.com SMAN 1 CIKEMBAR KAB. SUKABUMI
B-Right SCHOOL

14. Ada berap banyak bilangan bulat positif berlambang “abcde” dengan a  b  c  d  e ?

Solusi :
Banyaknya bilangan bulat positif berlambang “abcde” dengan a  b  c  d  e
Jelas bahwa a > 1, sehingga banyaknya angka yang dapat dipilih sebanyak 9.
 Jika a  b  c  d  e

m
Artinya kelima angka tersebut berbeda, sehingga banyaknya bilangan bulat positif
berlambang “abcde” sama dengan banyaknya cara memilih 5 angka berbeda dari

co
9!
9 angka yang tersedia, yaitu C 59   126 cara
5 !.4 !
 Jika a  b  c  d  e

a.
Artinya terdapat empat angka berbeda, sehingga banyaknya bilangan bulat positif
berlambang “abcde” sama dengan banyaknya cara memilih 4 angka berbeda dari

ik
9!
9 angka yang tersedia, yaitu C 49   126 cara

at
4 !.5 !
Banyaknya bilangan bulat positif berlambang “abcde” dengan a  b  c  d  e adalah =

em
C 5  C 4  2 C 5  2 C 4 = 252
9 9 9 9

 C 5  C 4  2 C 5  2 C 4  252
9 9 9 9

at
15. Bilangan asli terkecil lebih dari 2011 yang bersisa 1 jika dibagi 2, 3, 4, 5, 6, 7, 8, 9, dan
m
10 adalah ...
an

Solusi :
Bilangan asli terkecil lebih dari 2011 yang bersisa 1 jika dibagi 2, 3, 4, 5, 6, 7, 8, 9, dan
10.
at

Misal bilangan tersebut adalah A, maka A dapat dinyatakan dalam bentuk A  q .k  1


dengan q merupakan KPK dari 2, 3, 4, 5, 6, 7, 8, 9, dan 10. Dan k merupakan bilangan
at

asli.
Perhatikan bahwa,faktor prima dari masing-masing pembagi adalah
.c

22 

w

3 3 
4 2 
2
w


5 5 
//w


6 K P K  2 .3 .5.7  2520
3 2
2.3 
7 7 

s:

8 2 
3


9
2
3 
tp

10  2.5 
ht

Sehingga A  2 5 2 0 .k  1 .
Karena A adalah bilangan asli terkecil yang lebih dari 2011, maka haruslah k = 1
sehingga diperoleh A  2 5 2 0 .k  1  2 5 2 0 .1  1  2 5 2 1
 2521

22
Solusi OSK Matematika SMA 2011
Shared by: www.catatanmatematika.com SMAN 1 CIKEMBAR KAB. SUKABUMI
B-Right SCHOOL

16. Bilangan bulat positif terkecil a sehingga 2 a  4 a  6 a   200a merupakan kuadrat


sempurna adalah ....

Solusi :
Misalkan 2 a  4 a  6 a   200 a  k
2
untuk suatu bilangan bulat positif k. Perhatikan
barisan berikut
 200 a  a  2  4  6   200 

m
2a  4a  6a 

 2  200  .100

co
k  a.
2

2
k  a .101.100
2

a.
k  a .101.100

ik
k  10 a .101
karena k merupakan bilangan bulat positif, maka haruslah a  101n 2

at
dan karena a bilangan bulat positif terkecil, maka n =1 , sehingga a = 101
 101

em
17. Misalkan A dan B adalah sudut-sudut lancip yang memenuhi
1 1
tan ( A  B )  dan tan ( A  B ) 
at
2 3
Besar sudut A adalah ...
m
Solusi :
an

tan  2 A   tan   A  B    A  B  

tan  A  B   tan  A  B 
at


1  tan  A  B  . tan  A  B 
at

1 1

 2 3
.c

1 1
1 .
2 3
w

tan  2 A   1
w

A sudut lancip maka 2 A  4 5 0


//w

A  2 2, 5
0

0
 22,5
s:

18. Jika ax  2 y  3 dan 5 x  by  7 menyatakan persamaan garis yang sama, maka a  b


sama dengan....
tp

Solusi :
ht

a 2 a 5 15 
ax  2 y  3  x y 1   a
3 3 3 7 7 
 15 14 143
Sehingga  ab  
5 b b 2 14  7 3 21
5 x  by  7  x y 1   b
7 7 7 3 3 

143

21

23
Solusi OSK Matematika SMA 2011
Shared by: www.catatanmatematika.com SMAN 1 CIKEMBAR KAB. SUKABUMI
B-Right SCHOOL

19. Terdapat 5 orang pria dan 5 orang wanita duduk dalam sederetan kursi secara random.
Berapa banyaknya cara untuk menduduki kursi tersebut, dengan syarat tidak boleh ada
yang duduk berdampingan dengan jenis kelamin yang sama?

Solusi :
Cara duduk yang mungkin adalah LPLPLPLPLP atau PLPLPLPLPL

m
Maka banyaknya cara adalah 5!.5!.2 = 28.800 cara
 28.800 cara

co
20. Ada berapa faktor positif dari 2 7 3 5 5 3 7 2 yang merupakan kelipatan 10?

a.
Solusi :

ik
Misal 2 7 35 5 3 7 2  A.10 untuk suatu bilangan bulat A
Karena 2 7 35 5 3 7 2  2 6 35 5 2 7 2.10 maka A  2 6 3 5 5 2 7 2

at
Sehingga banyaknya faktor positif dari 2 7 3 5 5 3 7 2 yang merupakan kelipatan 10 sama

em
dengan banyaknya faktor positif dari Ayaitu sebanyak
 6  1   5  1   2  1   2  1   7.6.3.3  378
 378 faktor
at
m
an
at
at
.c
w
w
//w
s:
tp
ht

24
Shared by: www.catatanmatematika.com

Tutur Widodo Pembahasan OSK Matematika SMA 2012

Pembahasan Olimpiade Matematika SMA


Tingkat Kabupaten
Tahun 2012
Oleh Tutur Widodo

1. Banyaknya bilangan bulat n yang memenuhi

m
(n − 1)(n − 3)(n − 5)(n − 2013) = n(n + 2)(n + 4)(n + 2012)

co
adalah ...

a.
Jawaban : 0 ( tidak ada )

ik
Jika n genap maka ruas kanan genap tetapi ruas kiri ganjil. Sedangkan jika n ganjil
maka ruas kanan ganjil tetapi ruas kiri genap. Jadi, tidak ada nilai n yang memenuhi.

at
2. Banyaknya pasangan bilangan asli berbeda yang selisih kuadratnya 2012 adalah ...
Jawaban : 1 em
Misal kedua bilangan tersebut adalah a dan b maka a2 −b2 = 2012 ⇔ (a+b)(a−b) =
2012. Oleh karena itu, (a + b) dan (a − b) adalah faktor positif dari 2012. Karena faktor
at
positif dari 2012 adalah 1, 2, 4, 503, 1006 dan 2012. Selain itu, karena (a+b) dan (a−b)
m

paritasnya sama maka nilai yang mungkin adalah a + b = 1006 dan a − b = 2. Sehingga
diperoleh, a = 504 dan b = 502.
an

3. Bilangan terbesar x kurang dari 1000 sehingga terdapat tepat dua bilangan asli n
n2 + x
sehingga merupakan bilangan asli adalah ...
at

n+1
Jawaban : 960
at

Perhatikan,
n2 + x x+1
=n−1+
.c

n+1 n+1
n2 + x
w

maka agar bulat, haruslah n + 1 faktor dari x + 1. Oleh karena itu, supaya
n+1
hanya ada tepat dua nilai n maka x + 1 harus memiliki tepat 3 faktor. Dengan kata
w

lain x + 1 adalah kuadrat suatu bilangan prima. Jadi, diperoleh x + 1 = 312 = 961
//w

sehingga x = 960.

4. Diketahui suatu kelas terdiri dari 15 siswa. Semua siswa tersebut akan dikelompokkan
s:

menjadi 4 kelompok yang terdiri dari 4, 4, 4 dan 3 siswa. Ada berapa cara pengelom-
tp

pokan tersebut? ! ! !
15 11 7
ht

4 4 4
Jawaban :
3!
Misal kelompok yang terbentuk adalah A, B, C dan D dengan A, B dan C terdiri dari
4 anggota dan D terdiri dari 3 anggota. Maka :
!
15
• Banyaknya cara menyusun A ada
4
!
11
• Banyaknya cara menyusun B ada
4
!
7
• Banyaknya cara menyusun C ada
4

25
Shared by: www.catatanmatematika.com

Tutur Widodo Pembahasan OSK Matematika SMA 2012

Untuk kelompok D tinggal sisanya saja, jadi tidak perlu repot menghitung. Tetapi
yang perlu diingat adalah dengan
! cara! ini! setiap kasus dihitung sebanyak 3!= 6 kali.
15 11 7
4 4 4
Jadi, jawabannya adalah
3!
5. Diberikan segitiga siku-siku ABC, dengan AB sebagi sisi miringnya. Jika keliling dan
luasnya berturut-turut 624 dan 6864. Panjang sisi miring segitiga tersebut adalah ...
Jawaban : 290
Dari keterangan soal diperoleh,

m
co
a + b + c = 624 ⇔ a + b = 624 − c

dan

a.
ab
= 6864
2

ik
Dengan rumus phytagoras diperoleh

at
c 2 = a2 + b 2
= (a + b)2 − 2ab em
= (624 − c)2 − 4 · 6864
at
= c2 − 2 · 624c + 6242 − 4 · 6864
m
an

maka diperoleh c = 290.

6. Banyaknya tripel bilangan bulat (x, y, z) yang memenuhi


at

x2 + y 2 + z 2 − xy − yz − zx = x3 + y 3 + z 3
at

adalah ...
.c

Jawaban : tak hingga


w

Jika x = k, y = 1 − k dan z = 0 dengan k ∈ Z maka diperoleh,


w

x2 + y 2 + z 2 − xy − yz − zx = k 2 + (1 − k)2 − k(1 − k)
//w

= k 2 + 1 − 2k + k 2 − k + k 2
= 3k 2 − 3k + 1
s:

= k 3 + 1 + 3k 2 − 3k − k 3
= k 3 + (1 − k)3
tp

= x3 + y 3 + z 3
ht

ini berarti (k, 1−k, 0) adalah penyelesaian dari x2 +y 2 +z 2 −xy −yz −zx = x3 +y 3 +z 3 .
Oleh karena itu, (k, 1−k, 0) dengan k ∈ Z dan semua permutasinya adalah penyelesaian
dari x2 + y 2 + z 2 − xy − yz − zx = x3 + y 3 + z 3 yang tentu saja jumlahnya ada takhingga.

7. Diberikan suatu lingkaran dengan diameter AB = 30. Melalui A dan B berturut-


turut ditarik tali busur AD dan BE berpotongan di titik C. Jika AC = 3AD dan
BC = 4BE, maka luas segitiga ABC adalah ...
Jawaban : 540
Perhatikan sketsa gambar di bawah ini!

26
Shared by: www.catatanmatematika.com

Tutur Widodo Pembahasan OSK Matematika SMA 2012

E
D

m
A

co
a.
Perlu diperhatikan bahwa ∠ADB = ∠CDB = ∠AEB = ∠AEC = 90◦ . Misal, AD = x

ik
dan BE = y maka AC = 3x, CD = 2x, BC = 4y dan CE = 3y. Dengan teorema
Phytagoras pada segitiga ABD dan segitiga BCD diperoleh

at
302 − x2 = (4y)2 − (2x)2 ⇔ 900 − x2 = 16y 2 − 4x2
em
⇔ 900 = 16y 2 − 3x2
at
Demikian pula dengan teorema Phytagoras pada segitiga ABE dan segitiga ACE diper-
oleh
m

302 − y 2 = (3x)2 − (3y)2 ⇔ 900 − y 2 = 9x2 − 9y 2


an

⇔ 900 = 9x2 − 8y 2
at

dengan menggabungkan kedua persamaan di atas didapat,


at

16y 2 − 3x2 = 9x2 − 8y 2 ⇔ 24y 2 = 12x2 ⇔ x2 = 2y 2


.c

sehingga kita peroleh


w


900 = 16y 2 − 3x2 = 16y 2 − 6y 2 = 10y 2 ⇔ y= 90
w
//w

Oleh karena itu,



AE 2 = 900 − y 2 = 900 − 90 = 810 ⇔ AE = 810
s:

Jadi,
tp

1
ht

Luas segitiga ABC = BC · AE


2
1 √
= · 4y · 810
2
√ √
= 2 · 90 810
√ √
= 2 · 3 10 · 9 10 = 540

8. Misalkan a, b, c, d, dan e adalah bilangan-bilangan bulat sehingga 2a 3b 4c 5d 6e juga meru-


pakan bilangan bulat. Jika diketahui bahwa nilai mutlak dari a, b, c, d dan e tidak lebih
dari 2012 maka nilai terkecil yang mungkin dari a + b + c + d + e adalah ...
Jawaban : -2012

27
Shared by: www.catatanmatematika.com

Tutur Widodo Pembahasan OSK Matematika SMA 2012

Perhatikan,
2a 3b 4c 5d 6e = 2a 3b 22c 5d (2 · 3)e = 2a+2c+e 3b+e 5d

Agar a + b + c + d + e minimal, maka haruslah a + 2c + e = 0, b + e = 0 dan d = 0. Dari


a + 2c + e = 0 dan b + e = 0 diperoleh persamaan b = a + 2c. Karena nilai minimum
b yang mungkin adalah −2012 maka agar a + b + c + d + e minimum pilih a = −2012
dan c = 0. Sehingga a + b + c + d + e = a = −2012.
√ √
9. Jika ( 2012 + 2011)2 = n + r dengan n merupakan bilangan asli dan 0 ≤ r < 1, maka

m
r = ...
√ √
Jawaban : ( 2012 + 2011)2 − 8045

co
√ √ √
( 2012 + 2011)2 = 2012 + 2011 + 2 2012 · 2011

a.
√ √
Perhatikan bahwa 2011 < 2012 · 2011 < 2012 sehingga 2012 · 2011 = 2011 + k.

ik
Akan ditunjukkan bahwa k < 21 . Andaikan k ≥ 21 maka berakibat

at
2012 · 2011 = (2011 + k)2
1
≥ (2011 + )2
2
em 1
= 20112 + 2011 +
4
at
> 2011 · 2012
m

1
yang jelas salah. Oleh karena itu, terbukti k < 2
sehingga 0 ≤ 2k < 1.
an

√ √ √
( 2012 + 2011)2 = 2012 + 2011 + 2 2012 · 2011
at

= 4023 + 4022 + 2k
= 8045 + r
at

√ √
sehingga r = ( 2012 + 2011)2 − 8045
.c

10. Tentukan semua nilai b sehingga untuk semua x paling tidak salah satu dari f (x) =
w

x2 + 2012x + b atau g(x) = x2 − 2012x + b positif.


w

Jawaban : b > 0
//w

Jumlahkan kedua fungsi, diperoleh

f (x) + g(x) = 2x2 + 2b


s:

sehingga untuk sebarang nilai x jika b > 0 maka f (x) + g(x) selalu bernilai positif. Ini
tp

berarti paling tidak salah satu dari f (x) atau g(x) bernilai positif. Selanjutnya tinggal
ht

dibuktikan, untuk b ≤ 0 terdapat x = t sehingga f (t) ≤ 0 dan g(t) ≤ 0. Untuk itu


pilih t = 0 sehingga

f (t) = f (0) = b ≤ 0 dan g(t) = g(0) = b ≤ 0

Jadi, terbukti nilai b yang memenuhi adalah b > 0.

11. Jumlah semua bilangan bulat x sehingga 2 log(x2 − 4x − 1) merupakan bilangan bulat
adalah ...
Jawaban : 4
Agar 2 log(x2 − 4x − 1) bernilai bulat maka x2 − 4x − 1 = 2n untuk suatu bilangan

28
Shared by: www.catatanmatematika.com

Tutur Widodo Pembahasan OSK Matematika SMA 2012

bulat n. Karena x2 − 4x − 1 bernilai bulat maka n ≥ 0. Perhatikan juga,

x2 − 4x − 1 = 2n ⇔ x2 − 4x + 4 − 1 = 2n + 4
⇔ (x − 2)2 = 2n + 5

tetapi karena (x − 2)2 ≡ 0, 1, atau 4 mod 8 dan untuk n ≥ 3, 2n + 5 ≡ 5 mod 8 maka


n ≤ 2. Jadi, nilai yang memenuhi n = 0, 1, 2. Mudah dicek hanya nilai n = 2 yang
memenuhi dengan memperoleh persamaan kuadrat x2 − 4x − 5 = 0. Jadi, x1 + x2 = 4.

m
12. Ada berapa faktor positif dari 27 35 53 72 yang merupakan kelipatan 6?
Jawaban : 420

co
Karena 27 35 53 72 = 26 34 53 72 ·6, maka banyaknya faktor positif 27 35 53 72 yang merupakan
kelipatan 6 sama dengan banyaknya faktor positif dari 26 34 53 72 yaitu ada

a.
(6 + 1) x (4 + 1) x (3 + 1) x (2 + 1) = 420.

ik
13. Suatu set soal terdiri dari 10 soal pilihan B atau S dan 15 soal pilihan ganda dengan

at
4 pilihan. Seorang siswa menjawab semua soal dengan menebak jawaban secara acak.
Tentukan Probabilitas ia menjawab dengan benar hanya 2 soal?
Jawaban : em
Jika 2 soal benar tersebut berasal dari soal tipe B atau S maka peluangnya adalah
at
 10 !  
15
1 10 3
· ·
2 2 4
m

Jika 2 soal benar tersebut berasal dari soal tipe pilihan ganda maka peluangnya adalah
an

 10  2  13 !
1 1 3 15
· ·
at

2 4 4 2
at

Jika 1 soal benar tersebut berasal dari soal tipe B atau S dan 1 soal benar berasal dari
pilihan ganda maka peluangnya adalah
.c

 10 !    !
14
w

1 10 1 3 15
· ·
2 1 4 4 1
w

Jadi, secara keseluruhan peluang menjawab tepat 2 soal benar adalah


//w

 10 !  !   !    !
15  10  2  13 10 14
1 10 3 1 1 3 15 1 10 1 3 15
· · + · · + · ·
s:

2 2 4 2 4 4 2 2 1 4 4 1
tp

14. Diberikan segitiga ABC dengan keliling 3, dan jumlah kuadrat sisi-sisinya sama dengan
5. Jika jari-jari lingkaran luarnya sama dengan 1, maka jumlah ketiga garis tinggi dari
ht

segitiga ABC tersebut adalah ...


Jawaban : 1
Perhatikan gambar di bawah ini!

29
Shared by: www.catatanmatematika.com

Tutur Widodo Pembahasan OSK Matematika SMA 2012

t1
a
b
t3
t2 B

m
c
A

co
Misalkan sisi - sisi segitiga tersebut adalah a, b, c maka diperoleh

a.
a+b+c=3

ik
at
dan
a2 + b2 + c2 = 5

selain itu kita punya identitas


em
(a + b + c)2 = a2 + b2 + c2 + 2(ab + bc + ac)
at
m

sehingga diperoleh
an

9 = (a+b+c)2 = a2 +b2 +c2 +2(ab+bc+ac) = 5+2(ab+bc+ac) ⇔ ab+bc+ac = 2

Misalkan pula R jari - jari lingkaran luar dari segitiga ABC maka diketahui R = 1.
at

Dari aturan sinus diperoleh


at

a b c
+ + = 2R = 2
.c

sin A sin B sin C

Oleh karena itu, jika t1 , t2 , t3 berturut - turut adalah garis tinggi yang ditarik dari titik
w

C, A, B maka didapatkan
w

t1 + t2 + t3 = b sin A + c sin B + a sin C


//w

a b c
=b· +c· +a·
2 2 2
s:

1
= (ab + bc + ac)
2
tp

1
= ·2=1
2
ht

15. Jika hasil kali tiga bilangan ganjil berurutan sama dengan 7 kali jumlah ketiga bilangan
itu, maka jumlah kuadrat ketiga bilangan itu adalah ...
Jawaban : 83
Misal tiga bilangan tersebut adalah t − 2, t, t + 2 dengan t bilangan ganjil. Sehingga
diperoleh,
(t − 2)t(t + 2) = 7 · 3t ⇔ t2 − 25 = 0

Jika t = 5 maka tiga bilangan tersebut adalah 3, 5, 7 sehingga 32 + 52 + 72 = 83


Jika t = −5 maka tiga bilangan tersebut adalah −7, −5, −3 sehingga (−3)2 + (−5)2 +
(−7)2 = 83.

30
Shared by: www.catatanmatematika.com

Tutur Widodo Pembahasan OSK Matematika SMA 2012

16. Diketahui 4ABC sama kaki dengan panjang AB = AC = 3, BC = 2, titik D pada sisi
AC dengan panjang AD = 1. Tentukan luas 4ABD.

Jawaban : 2 3 2
Dengan Heron formula diperoleh,
√ √
Luas 4ABC = 4·1·1·2=2 2

Selain itu, kita punya


Luas 4ABD AD 1
= =

m
Luas 4ABC AC 3
sehingga diperoleh, √

co
2 2
Luas 4ABD =
3

a.
17. Suatu dadu ditos enam kali. Tentukan Probabilitas jumlah mata yang muncul 27.
1666

ik
Jawaban :
66
Untuk mencari banyak kemungkinan jumlah mata dadu yang muncul berjumlah 27

at
equivalen dengan mencari banyaknya penyelesaian dari persamaan x1 + x2 + x3 + x4 +
x5 + x6 = 27 dimana xi bilangan bulat dan 1 ≤ xi ≤ 6 untuk setiap i = 1, 2, 3, 4, 5, 6.
em
Yang setara dengan mencari koefisien x27 dari (x + x2 + x3 + x4 + x5 + x6 )6 . Perhatikan,

x + x2 + x3 + x4 + x5 + x6 = x(1 + x + x2 + x3 + x4 + x5 )
at
= x(1 + x + x2 + x3 (1 + x + x2 ))
m

= x(1 + x + x2 )(1 + x3 )
an

sehingga
(x + x2 + x3 + x4 + x5 + x6 )6 = x6 (1 + x3 )6 (1 + x + x2 )6
at

Dengan Binom Newton didapat,


at

6
.c

X
3 6
(1 + x ) = x3i
i=0
w

dan
w

6
X
2 6
(x2 )6−i (x + 1)i
//w

(1 + x + x ) =
i=0
6 i
!
X X
= x12−2i xj
s:

i=0 j=0
tp

6 X
X i
= x12−2i+j
i=0 j=0
ht

Oleh karena itu didapat

• Koefisien x9 dari (x3 + 1)6 adalah 20


• Koefisien x12 dari (x3 + 1)6 adalah 15
• Koefisien x15 dari (x3 + 1)6 adalah 6
• Koefisien x18 dari (x3 + 1)6 adalah 1

selain itu diperoleh juga,

• Koefisien x12 dari (x2 + x + 1)6 adalah 1

31
Shared by: www.catatanmatematika.com

Tutur Widodo Pembahasan OSK Matematika SMA 2012

• Koefisien x9 dari (x2 + x + 1)6 adalah 50


• Koefisien x6 dari (x2 + x + 1)6 adalah 141
• Koefisien x6 dari (x2 + x + 1)6 adalah 50

Jadi, koefisien x27 dari x6 (1 + x3 )6 (1 + x + x2 )6 adalah


(20 x 1)+(15 x 50)+(6 x 141)+(1 x 50)=1666
1666
Jadi, peluang diperoleh jumlah mata yang muncul sama dengan 27 adalah
66
18. Diberikan segitiga ABC dengan sisi-sisi : AB = x + 1, BC = 4x − 2 dan CA = 7 − x.

m
Tentukan nilai dari x sehingga segitiga ABC merupakan segitiga sama kaki.

co
Jawaban : 95
Karena x + 1, 4x − 2 dan 7 − x membentuk sisi - sisi segitiga maka berlaku,

a.
4
(x + 1) + (4x − 2) > 7 − x sehingga x >
3

ik
5
(x + 1) + (7 − x) > 4x − 2 sehingga x <
2

at
(7 − x) + (4x − 2) > x + 1 sehingga x > −2

Oeh karena itu,


em
4
• Jika x + 1 = 4x − 2 diperoleh x = 1 yang jelas tidak mungkin sebab < x < 52 .
at
3
4
• Jika x + 1 = 7 − x diperoleh x = 3 yang jelas tidak mungkin sebab 3
< x < 52 .
m

• Jika 7 − x = 4x − 2 diperoleh x = 59 .
an

19. Misalkan terdapat 5 kartu dimana setiap kartu diberi nomor yang berbeda yaitu 2, 3, 4,
5, 6. Kartu-kartu tersebut kemudian dijajarkan dari kiri ke kanan secara acak sehingga
at

berbentuk barisan. Berapa probabilitas bahwa banyaknya kartu yang dijajarkan dari
kiri ke kanan dan ditempatkan pada tempat ke- i akan lebih besar atau sama dengan
at

i untuk setiap i dengan 1 ≤ i ≤ 5


.c

2
Jawaban : 15
Susunan yang paling sederhana adalah 2, 3, 4, 5, 6
w

Untuk memenuhi kondisi pada soal maka masing - masing angka 2, 3, 4, dan 5 hanya
w

bisa digeser ke kanan satu langkah saja. Cara ini ada sebanyak 24 = 16.
Sedangkan untuk kemungkinan angka digeser ke kiri tidak perlu kita perhatikan, sebab
//w

jika kita menggeser angka ke kiri maka pasti ada angka yang harus digeser ke kanan
sehingga sudah masuk perhitungan yang pertama di atas. Oleh karena itu, besar
s:

probabilitas adalah 16
5!
2
= 15 .
tp

20. N lingkaran digambar pada sebuah bidang datar demikian sehingga terdapat enam titik
dimana keenam titik tersebut terdapat pada paling sedikit tiga lingkaran. Berapa N
ht

terkecil yang memenuhi kondisi tersebut?


Jawaban : 5
Jika kita menggambar 3 lingkaran pada didang datar maka maksimal akan terbentuk
6 titik potong, seperti gambar berikut

32
Shared by: www.catatanmatematika.com

Tutur Widodo Pembahasan OSK Matematika SMA 2012

C
D
E
F

m
co
Karena melalui sebarang 3 titik yang tidak segaris dapat dibentuk sebuah lingkaran

a.
yang melalui ketiga titik tersebut, maka dengan membuat dua lingkaran yang masing
- masing melalui 3 titik A, B, C, D, E, F akan terbentuk 5 lingkaran dimana terdapat

ik
6 titik yang masing - masing terdapat pada 3 lingkaran, sesuai apa yang diminta.

at
em
at
m

B
an

C
at

D
E
F
at
.c

A
w
w
//w
s:

Disusun oleh : Tutur Widodo


tp

Apabila ada saran, kritik maupun masukan


ht

silakan kirim via email ke


tutur.w87@gmail.com
Terima kasih.
My blog : http://mathematic-room.blogspot.com

33
Shared by: www.catatanmatematika.com

Olimpiade Sains Nasional Bidang Matematika SMA/MA


Seleksi Tingkat Kota/Kabupaten
Tahun 2013
Waktu: 120 menit

m
Petunjuk: Untuk masing-masing soal, tulis jawab akhirnya saja (tanpa penjabaran) di lembar jawab
yang disediakan.

co
1. Misalkan a dan b bilangan asli dengan a > b. Jika √94 + 2√2013 = √𝑎 + √𝑏, maka nilai a – b
adalah ...

a.
2. Diberikan segitiga ABC dengan luas 10. Titik D, E, dan F berturut-turut terletak pada sisi-sisi

ik
AB, BC, dan CA dengan AD = 2, DB = 3. Jika segitiga ABE dan segiempat DBEF mempunyai

at
luas yang sama, maka luasnya sama dengan ...

3. Misalkan p dan q bilangan prima. Jika diketahui persamaan x2014 – px2013 + q = 0 mempunyai

em
akar-akar bilangan bulat, maka nilai p + q adalah ...

𝑘𝑥 3
4. Jika fungsi f didefinisikan oleh f(x) = 2𝑥+3 , 𝑥 ≠ − , k konstanta, memenuhi 𝑓(𝑓 (𝑥 )) = 𝑥 untuk
at
2
3
setiap bilangan real x, kecuali 𝑥 ≠ − 2, maka nilai k adalah ...
m
5. Koefisien dari x2013 pada ekspansi
an

(1 + 𝑥)4016 + 𝑥(1 + 𝑥)4015 + 𝑥 2 (1 + 𝑥)4014 + ⋯ + 𝑥 2013 (1 + 𝑥)2013


adalah ...
at

2 2
6. Jika 𝑥 − 𝑦 = 1 dan 𝑦 − 𝑥 = 2, maka (x + y)2 = ...
at
.c

7. Suatu dadu ditos 6 kali. Banyak cara memperoleh jumlah mata yang muncul 28 dengan tepat
satu dadu muncul mata 6 adalah ...
w

8. Misalkan P adalah titik interior dalam daerah segitiga ABC sehingga besar PAB = 10, PBA
w

= 20, PCA = 30, dan PAC = 40. Besar ABC adalah ...
//w

9. Sepuluh kartu ditulis angka satu sampai sepuluh (setiap kartu hanya terdapat satu angka dan
tidak ada dua kartu yang memiliki angka yang sama). Kartu-kartu tersebut dimasukkan ke dalam
s:

kotak dan diambil satu secara acak. Kemudian sebuah dadu dilempar. Probabilitas dari hasil kali
angka pada kartu dan angka pada dadu menghasilkan bilangan kuadrat adalah ...
tp

10. Enam orang siswa akan duduk pada tiga meja bundar, dimana setiap meja akan diduduki oleh
ht

minimal satu siswa. Banyaknya cara untuk melakukan hal tersebut adalah ...

11. Suatu partikel bergerak pada bidang Cartesius dari titik (0, 0). Setiap langkah bergerak satu
satuan searah sumbu X positif dengan probabilitas 0,6 atau searah sumbu Y positif dengan
probabilitas 0,4. Setelah sepuluh langkah, probabilitas partikel tersebut sampai pada titik (6,4)
dengan melalui (3,4) adalah ...

34
Shared by: www.catatanmatematika.com
12. Diberikan segitiga ABC, dengan panjang sisi AB = 30. Melalui AB sebagai diameter, dibuat
1
sebuah lingkaran, yang memotong sisi AC dan sisi BC berturut-turut di D dan E. Jika 𝐴𝐷 = 3 𝐴𝐶
1
dan 𝐵𝐸 = 4 𝐵𝐶, maka luas segitiga ABC sama dengan ...

13. Banyaknya nilai α dengan 0 < α < 90 yang memenuhi persamaan
1
(1 + cos α)(1 + cos 2α)(1 + cos 4α) = 8
adalah ...

m
14. Diberikan segitiga lancip ABC dengan O sebagai pusat lingkaran luarnya. Misalkan M dan N

co
berturut-turut pertengahan OA dan BC. Jika ABC = 4OMN dan ACB = 6OMN, maka
besarnya OMN = ...

a.
15. Tentukan semua bilangan tiga digit yang memenuhi syarat bahwa bilangan tersebut sama dengan

ik
penjumlahan dari faktorial setiap digitnya.

at
16. Diberikan himpunan
𝑥 2 − 2𝑥 + 7

em
𝑆 = {𝑥 ∈ ℤ | 2𝑥 − 1 ∈ ℤ}
Banyaknya himpunan bagian dari S adalah ...
at
17. Untuk x > 0, y > 0, didefinisikan f(x, y) adalah nilai terkecil diantara 𝑥,
𝑦
2
2 1
+ , dan . Nilai
𝑥 𝑦
m
terbesar yang mungkin dicapai oleh f(x, y) adalah ...
an

18. Nilai k terkecil, sehingga jika sembarang k bilangan dipilih dari {1, 2, ... , 30}, selalu dapat
ditemukan 2 bilangan yang hasil kalinya merupakan bilangan kuadrat sempurna adalah ...
at

19. Diketahui x1, x2 adalah dua bilangan bulat berbeda yang merupakan akar-akar dari persamaan
kuadrat x2 + px + q + 1 = 0. Jika p dan p2 + q2 adalah bilangan-bilangan prima, maka nilai
at

terbesar yang mungkin dari 𝑥12013 + 𝑥22013 adalah ...


.c

20. Misalkan x menyatakan bilangan bulat terbesar yang lebih kecil atau sama dengan x dan x
w

menyatakan bilangan bulat terkecil yang lebih besar atau sama dengan x. Tentukan semua x yang
memenuhi x + x = 5.
w
//w
s:
tp
ht

35
Shared by: www.catatanmatematika.com

Solusi Olimpiade Matematika Tk Kabupaten/Kota 2013

1. �94 + 2√2013 = √𝑎 + √𝑏
2013 = 61 ⋅ 33 dan 94 = 61 + 33
�94 + 2√2013 = √61 + √33
Maka, 𝑎 = 61 dan 𝑏 = 33

m
∴ Jadi, nilai 𝑎 − 𝑏 adalah 28.

co
2. Misalkan H adalah perpotongan AE dan DF. Misalkan juga [XYZ] menyatakan luas segitiga XYZ.

a.
ik
at
em
Karena [ABE] = [ABEF] maka [ADH] = [EFH]
Karena [ADH] = [EFH] maka [ADF] = [AEF].
Karena ∆ADF dan ∆AEF memiliki alas yang sama dan luas keduanya juga sama maka tinggi

at
keduanya harus sama. Jadi, DE akan sejajar AC.
Karena DE sejajar AC maka ∆DBE sebangun dengan ∆ABC
m
Jadi, BE : EC = 3 : 2
[ABE] : [ABC] = 3 : 5
[ABE] = 6
an

∴ Jadi, luas segitiga ABE sama dengan 6.


at

3. x2014 − px2013 + q = 0
q = x2013(p − x)
at

Maka x = ±1
• Jika x = −1
.c

q = −p − 1
p + q = −1 yang tidak mungkin terpenuhi kesamaan sebab p dan q prima.
w

• Jika x = 1
q=p−1
w

p−q=1
Dua bilangan prima berselisih 1 hanya p = 3 dan q = 2.
//w

∴ Jadi, p + q = 5.
s:

𝑘𝑥
4. 𝑓(𝑥) =
2𝑥+3
𝑓�𝑓(𝑥)� = 𝑥
tp

𝑘𝑥
𝑘 �2𝑥 + 3�
=𝑥
ht

𝑘𝑥
2 �2𝑥 + 3� + 3
𝑘 2 = 2𝑘𝑥 + 3(2𝑥 + 3)
(𝑘 + 3)(𝑘 − 2𝑥 − 3) = 0
SMA Negeri 5 Bengkulu Eddy Hermanto, ST

36
Shared by: www.catatanmatematika.com

Olimpiade Matematika Tk Kabupaten/Kota 2013


𝑘 = −3 atau 𝑘 = 2𝑥 + 3
Karena 𝑘 adalah konstanta maka 𝑘 = −3.
∴ Jadi, nilai 𝑘 adalah −3.

m
5. Nampaknya ada kesalahan dalam soal. Soal seharusnya adalah menentukan koefisien dari x2013
pada ekspansi

co
(1 + x)4016 + x(1 + x)4015 + x2(1 + x)4014 + ⋅⋅⋅ + x2013(1 + x)2003
Maka koefisien x2013 adalah �40162013
� + �4015
2012
� + �4014
2011
� + ⋯ + �20030
�.
�2013� + �2012� + �2011� + ⋯ + � 0 � = �2003� + �2003� + �2003� + ⋯ + �2003
4016 4015 4014 2003 4016 4015 4014

a.
2003
�.
Rumus :
𝑚 𝑚+1 𝑚+2 𝑚+𝑛−1 𝑚+𝑛

ik
� �+� �+� �+ ⋯+ � �=� �
𝑚 𝑚 𝑚 𝑚 𝑚+1
Bukti (dengan induksi matematika) :

at
• Jika n = 1
𝑚 𝑚+1

em
� �=� �=1
𝑚 𝑚+1
• Andaikan benar untuk n = k
𝑚 𝑚+1 𝑚+2 𝑚+𝑘−1 𝑚+𝑘
� �+� �+� �+ ⋯+ � �=� �
𝑚 𝑚 𝑚 𝑚 𝑚+1

at
𝑚 𝑚+1 𝑚+2 𝑚+𝑘−1 𝑚+𝑘 𝑚+𝑘 𝑚+𝑘 𝑚+𝑘+1
� �+� �+� �+ ⋯+ � �+� �=� �+� �=� �
𝑚 𝑚 𝑚 𝑚 𝑚 𝑚+1 𝑚 𝑚+1
m
Terbukti benar untuk n = k + 1
Maka �4016
2003
� + �4015
2003
� + �4014
2003
� + ⋯ + �2003
2003
� = �4017
2004
� = �4017
2013

an
2013 𝟒𝟎𝟏𝟕
∴ Jadi, koefisien x pada ekspansi tersebut adalah �𝟐𝟎𝟎𝟒�.
at

2 2
6. − =1
𝑥 𝑦
at

2(𝑦 − 𝑥) = 𝑥𝑦
𝑦−𝑥 =2
𝑥𝑦 = 4
.c

(𝑥 + 𝑦)2 − (𝑦 − 𝑥)2 = 4𝑥𝑦


(𝑥 + 𝑦)2 = (2)2 + 4(4) = 20
w

∴ Jadi, (𝑥 + 𝑦)2 = 20
w

7. Semua kemungkinan susunan jumlah mata dadu sama dengan 28 dengan angka 6 muncul tepat
//w

sekali adalah :
• Susunan dadu (6,5,5,5,5,2)
6!
Banyaknya susunan = = 30
s:

4!
• Susunan dadu (6,5,5,5,4,3)
6!
Banyaknya susunan = = 120
tp

3!
• Susunan dadu (6,5,5,4,4,4)
6!
ht

Banyaknya susunan = = 60
3!2!
Maka banyaknya semua kemungkinan adalah 30 + 120 + 60 = 210
∴ Jadi, banyak cara memperoleh jumlah mata 28 dengan tepat satu dadu muncul 6 = 210.

SMA Negeri 5 Bengkulu Eddy Hermanto, ST

37
Shared by: www.catatanmatematika.com

Olimpiade Matematika Tk Kabupaten/Kota 2013

8. ∠PAB = 10o, ∠PBA = 20o, ∠PCA = 30o, dan ∠PAC = 40o.

m
co
a.
ik
at
em
∠APB = 150o dan ∠APC = 110o. Maka ∠BPC = 100o. Misalkan ∠PBC = x maka ∠PCB = 80o − x.
Dengan dalil sinus pada ∆APB didapat
sin 20𝑜
𝐴𝑃 = sin 150𝑜 𝐴𝐵 ⋅⋅⋅⋅⋅⋅⋅⋅⋅⋅⋅⋅⋅⋅⋅⋅⋅⋅⋅⋅⋅⋅⋅ (1)

at
Dengan dalil sinus pada ∆APC didapat
sin 30𝑜
𝐴𝑃 = 𝐴𝐶 ⋅⋅⋅⋅⋅⋅⋅⋅⋅⋅⋅⋅⋅⋅⋅⋅⋅⋅⋅⋅⋅⋅⋅ (2)
sin 110𝑜
m
Dari persamaan (1) dan (2) didapat
𝐴𝐵 sin 30𝑜 sin 150𝑜
= sin 20𝑜 sin 110𝑜 ⋅⋅⋅⋅⋅⋅⋅⋅⋅⋅⋅⋅⋅⋅⋅⋅⋅⋅⋅⋅⋅⋅⋅⋅⋅⋅⋅ (3)
an
𝐴𝐶
∠ABC = ∠PBA + ∠PBC = 20o + x dan ∠ACB = ∠ACP + ∠PCB = 110o − x
Dengan dalil sinus pada ∆ABC didapat
at

𝐴𝐵 sin(110𝑜−𝑥)
= sin(20𝑜 +𝑥)
⋅⋅⋅⋅⋅⋅⋅⋅⋅⋅⋅⋅⋅⋅⋅⋅⋅⋅⋅⋅⋅⋅⋅⋅⋅⋅⋅⋅⋅⋅ (4)
𝐵𝐶
Dari persamaan (3) dan (4) didapat
at

sin (20o + x) sin 30o sin 150o = sin (110o − x) sin 20o sin 110o
Mengingat sin 110o = cos 20o maka
.c

sin (20o + x) = 2 sin (110o − x) sin 40o


sin (20o + x) = 2 sin (110o − x) cos 50o = sin (160o − x) + sin (60o − x)
w

Mengingat bahwa sin (160o − x) = sin (20o + x) maka


sin (60o − x) = 0
w

Jadi, x = 60o
∠ABC = 20o + x = 80o
//w

∴ Jadi, ∠ABC = 80o.


s:

9. Misalkan (a,b) adalah kejadian munculnya angka a pada pengambilan kartu dan angka b pada
pelemparan dadu. Agar hasil kali kedua angka merupakan bilangan kuadrat maka kemungkinan
tp

semua kejadian adalah (1,1), (1,4), (2,2), (3,3), (4,1), (4,4), (5,5), (6,6), (8,2), (9,1), (9,4) yang
1 1 1
banyaknya ada 11. Peluang masing-masing kejadian adalah ∙ = .
10 6 60
ht

11
Maka peluang seluruh kejadian = .
60
𝟏𝟏
∴ Jadi, peluang seluruh kejadian =
𝟔𝟎

SMA Negeri 5 Bengkulu Eddy Hermanto, ST

38
Shared by: www.catatanmatematika.com

Olimpiade Matematika Tk Kabupaten/Kota 2013

10. Kemungkinan susunan keenam siswa adalah :


• Susunannya adalah 4, 1, 1.
�64��21��11�(4 − 1)! = 180

m
Terdapat perhitungan ganda pada perhitungan di atas. Contoh : A, B, C. D berada di meja I,
E di meja II dan F di meja III dianggap berbeda dengan A, B, C. D berada di meja I, F di meja
II dan E di meja III padahal seharusnya sama. Maka perhitungan tersebut harus dibagi 2!.

co
�64��21��11�(4−1)!
Jadi, banyaknya susunan = = 90
2!
• Susunannya adalah 3, 2, 1.

a.
�63��32��11�(3 − 1)! (2 − 1)! = 120
• Susunannya adalah 2, 2, 2.

ik
�62��42��22�
= 15

at
3!
Jadi, banyaknya susunan seluruhnya = 90 + 120 + 15 = 225.
∴ Jadi, susunan keenam siswa tersebut adalah 225.

em
11. Banyaknya cara melangkah dari titik (0,0) ke (3,4) adalah 7C3 = 35.
Banyaknya cara melangkah dari titk (3,4) ke titik (6,4) adalah 3C0 = 1.

at
Banyaknya langkah ke kanan dari titik (0,0) ke titik (6,4) ada sebanyak 6 dan langkah ke atas ada
sebanyak 4.
m
Maka peluang kejadian = 35 ⋅ 1 ⋅ (0,6)6 ⋅ (0,4)4.
𝟖𝟏𝟔𝟒𝟖
∴ Jadi, peluang kejadian = 35 ⋅ 1 ⋅ (0,6)6 ⋅ (0,4)4 = 𝟗
an
𝟓

12. Karena titik D dan E terletak pada setengah lingkaran maka ∠AEB = ∠ADB = 90o.
at
at
.c
w
w
//w

Misalkan panjang AC = 3x dan BC = 4y. Maka AD = x ; DC = 2x ; BE = y dan EC = 3y


s:

Pada ∆AEB berlaku :


AB2 = BE2 + AE2
tp

AE2 = 900 − y2 ⋅⋅⋅⋅⋅⋅⋅⋅⋅⋅⋅⋅⋅⋅⋅⋅⋅⋅ (1)


Pada ∆AEC berlaku :
AC2 = AE2 + EC2
ht

AE2 = 9x2 − 9y2 ⋅⋅⋅⋅⋅⋅⋅⋅⋅⋅⋅⋅⋅⋅⋅⋅⋅⋅ (2)


Dari persamaan (1) dan (2) didapat
9x2 − 8y2 = 900 ⋅⋅⋅⋅⋅⋅⋅⋅⋅⋅⋅⋅⋅⋅⋅⋅⋅⋅⋅⋅⋅⋅⋅⋅⋅⋅⋅⋅⋅⋅⋅⋅⋅⋅⋅⋅⋅⋅ (3)
SMA Negeri 5 Bengkulu Eddy Hermanto, ST

39
Shared by: www.catatanmatematika.com

Olimpiade Matematika Tk Kabupaten/Kota 2013


Pada ∆BAD berlaku :
AB2 = AD2 + BD2
BD2 = 900 − x2 ⋅⋅⋅⋅⋅⋅⋅⋅⋅⋅⋅⋅⋅⋅⋅⋅⋅⋅ (4)
Pada ∆BCD berlaku :
BC2 = BD2 + CD2

m
BD2 = 16y2 − 4x2 ⋅⋅⋅⋅⋅⋅⋅⋅⋅⋅⋅⋅⋅⋅⋅⋅⋅⋅ (5)
Dari persamaan (4) dan (5) didapat

co
16y2 − 3x2 = 900 ⋅⋅⋅⋅⋅⋅⋅⋅⋅⋅⋅⋅⋅⋅⋅⋅⋅⋅⋅⋅⋅⋅⋅⋅⋅⋅⋅⋅⋅⋅⋅⋅⋅⋅⋅⋅⋅⋅ (6)
Dari persamaan (3) dan (6) didapat
x2 = 180 sehingga x = 6√5 serta

a.
y2 = 90 sehingga y = 3√10
AC = 3x = 18√5

ik
BD2 = 16y2 − 4x2 = 16(90) − 4(180) = 720 sehingga BD = 12√5

at
1
Luas ∆ABC = AC ⋅ BD = 9√5 ⋅ 12√5 = 540
2
∴ Jadi, luas segitiga ABC sama dengan 540.

em
1
13. (1 + cos 𝛼)(1 + cos 2𝛼)(1 + cos 4𝛼) =
8
1

at
�1 − cos2 𝛼�(1 + cos 2𝛼)(1 + cos 4𝛼) = 8 (1 − cos 𝛼)
1
Mengingat bahwa 1 − cos2α = (1 − cos 2α) dan dengan melakukan terus menerus didapat
2
m
(1 − cos 8𝛼) = (1 − cos 𝛼)
cos 8α = cos α
an
8α = α + k ⋅ 360o atau 8α = −α + k ⋅ 360o
• 7α = k ⋅ 360o
Karena 0 < α < 90o maka ada 1 nilai α yang memenuhi.
at

• 9α = k ⋅ 360o
α = k ⋅ 40o
at

Karena 0 < α < 90o maka ada 2 nilai α yang memenuhi.


Maka banyaknya nilai α yang memenuhi ada 1 + 2 = 3.
.c

∴ Jadi, banyaknya nilai α yang memenuhi ada 3.


w

14. Misalkan ∠OMN = α maka ∠ABC = 4α dan ∠ACB = 6α


w

Karena N pertengahan BC maka ∠CNO = 90o.


//w
s:
tp
ht

Sudut pusat = 2 kali sudut keliling.

SMA Negeri 5 Bengkulu Eddy Hermanto, ST

40
Shared by: www.catatanmatematika.com

Olimpiade Matematika Tk Kabupaten/Kota 2013


∠AOB = 2∠ACB = 12α sehingga ∠OBA = ∠OAB = 90o − 6α.
∠AOC = 2∠ABC = 8α
Karena ∠ABC = 4α maka ∠OBC = ∠OCB = 4α − (90o − 6α) = 10α − 90o.
Maka ∠CON = 90o − (10α − 90o) = 180o − 10α
∠MON = ∠AOC + ∠CON = (8α) + (180o − 10α) = 180o − 2α

m
Karena ∠MON = 180o − 2α dan ∠OMN = α maka ∠ONM = α
𝑅
Maka ∆OMN sama kaki dengan OM = ON = dengan R adalah jari-jari lingkaran luar ∆ABC.

co
2
𝑅
Karena ON = maka ∠OBC = 30 = 10α − 90o
o
2
α = 12o.

a.
∴ Jadi, besarnya ∠OMN sama dengan 12o.

ik
15. Misalkan bilangan tersebut adalah 100a + 10b + c maka 100a + 10b + c = a! + b! + c!

at
Karena 0! = 1, 1! = 1, 2! = 2, 3! = 6, 4! = 24, 5! = 120, 6! = 720 dan 7! = 5040 maka jelas bahwa
a, b, c ≤ 6.

em
Jika salah satu dari a, b dan c = 6 maka a! + b! + c! > 720 sedangkan 100a + 10b + c ≤ 666.
Maka a, b, c ≤ 5.
100a + 10b + c = a! + b! + c!
100a − a! = b! + c! − (10b + c)

at
Maksimum b! + c! − (10b + c) = 5! + 5! = 240
• Jika a = 5 maka 100a − a! = 380 > 240 (tidak memenuhi)
m
• Jika a = 4 maka 100a − a! = 376 > 240 (tidak memenuhi)
• Jika a = 3 maka 100a − a! = 294 > 240 (tidak memenuhi)
an
• Jika a = 2 maka 100a − a! = 198
b! + c! − (10b + c) = 198
Karena 4! + 4! = 48 < 198. Maka sedikitnya salah satu dari b atau c = 5
at

Misalkan b = 5
b! + c! − (10b + c) = 5! + c! − 50 − c
at

198 = 70 + c! − c
c! − c = 128. Tidak ada nilai c yang memenuhi.
.c

Jika c = 5
b! + c! − (10b + c) = b! + 5! − 10b − 5
198 = 115 + b! − 10b.
w

b! − 10b = 83. Tidak ada nilai b yang memenuhi.


w

• Jika a = 1 maka 100a − a! = 99


b! + c! − (10b + c) = 99
//w

99 − b! + 10b = c! − c
Jika b = 0 maka c! − c = 98 (tidak ada nilai c memenuhi)
Jika b = 1 maka c! − c = 108 (tidak ada nilai c memenuhi)
Jika b = 2 maka c! − c = 117 (tidak ada nilai c memenuhi)
s:

Jika b = 3 maka c! − c = 123 (tidak ada nilai c memenuhi)


Jika b = 4 maka c! − c = 115. Nilai c yang memenuhi adalah c = 5
tp

Jika b = 5 maka c! − c = 29 (tidak ada nilai c memenuhi)


Bilangan tersebut adalah 145.
ht

∴ Jadi, semua bilangan yang memenuhi adalah 145.

SMA Negeri 5 Bengkulu Eddy Hermanto, ST

41
Shared by: www.catatanmatematika.com

Olimpiade Matematika Tk Kabupaten/Kota 2013


𝑥2 −2𝑥+7
16. 𝑆 = �𝑥 ∈ 𝑍� ∈ 𝑍�
2𝑥−1
(2x − 1)  (x − 2x + 7) sehingga (2x − 1)  (2x2 − 4x + 14) = (x(2x − 1) − 3x + 14)
2

Maka (2x − 1)  (−3x + 14) sehingga (2x − 1)  (−6x + 28) = −3(2x − 1) + 25


Akibatnya (2x − 1)  25

m
• Jika 2x − 1 = −1
x = 0 yang memenuhi (2x − 1)  (x2 − 2x + 7)

co
• Jika 2x − 1 = 1
x = 1 yang memenuhi (2x − 1)  (x2 − 2x + 7)
• Jika 2x − 1 = −5

a.
x = −2 yang memenuhi (2x − 1)  (x2 − 2x + 7)
• Jika 2x − 1 = 5

ik
x = 3 yang memenuhi (2x − 1)  (x2 − 2x + 7)
• Jika 2x − 1 = −25

at
x = −12 yang memenuhi (2x − 1)  (x2 − 2x + 7)
• Jika 2x − 1 = 25

em
x = 13 yang memenuhi (2x − 1)  (x2 − 2x + 7)
Banyaknya nilai x ∈ Z yang memenuhi ada sebanyak 6.
∴ Jadi, banyaknya himpunan bagian dari 𝑆 adalah 26.

17. Misalkan saja 𝑎 = 𝑥 dan 𝑏 =


1

at
sehingga 𝑎 > 0 dan 𝑏 > 0
𝑦
m
1 2
𝑓(𝑥, 𝑦) = 𝑓(𝑎, 𝑏) = min(𝑎, 𝑏, + )
2𝑏 𝑎
an
1 2
Jika 𝑎 = 𝑏 = +
2𝑏 𝑎
𝑎(2𝑎) = 5
1 2 √10
at

𝑎=𝑏= + =
2𝑏 𝑎 2
√10 √10
• Jika 𝑎 ≤ 2 atau 𝑏 ≤ 2
at

√10
Maka 𝑓(𝑥, 𝑦) ≤ 2
.c

√10 √10
• Jika 𝑎 > dan 𝑏 >
2 2
1 2 1 4 √10
Maka 𝑓(𝑥, 𝑦) = + < 10 + 10 =
w

2𝑏 𝑎 √ √ 2
√10 √10
Maka 𝑓(𝑥, 𝑦) ≤ 2 dengan tanda kesamaan terjadi jika 𝑎 = 𝑏 = .
w

2
√𝟏𝟎
∴ Jadi, nilai terbesar yang mungkin dicapai oleh 𝑓(𝑥, 𝑦) adalah .
𝟐
//w

18. Misalkan A = {10, 11, 13, 14, 15, 17, 19, 21, 22, 23, 26, 29, 30}
s:

B = {1, 4, 9, 16, 25}


C = {2, 8, 18}
D = {3, 12, 27}
tp

E = {5, 20}
G = {6, 24}
ht

H = {7, 28}
A adalah himpunan yang jika dikalikan salah satu anggotanya dengan anggota himpunan A
maupun anggota himpunan lainnya maka tidak akan menghasilkan bilangan kuadrat.

SMA Negeri 5 Bengkulu Eddy Hermanto, ST

42
Shared by: www.catatanmatematika.com

Olimpiade Matematika Tk Kabupaten/Kota 2013


Himpunan B, C, D, E, F, G dan H adalah himpunan yang jika salah satu anggotanya dikalikan
dengan anggota dari himpunannya sendiri akan menghasilkan bilangan kuadrat sempurna.
Maka jika seluruh anggota A, digabungkan dengan masing-masing satu anggota dari himpunan B,
C, D, E, F, G dan H maka tidak akan ada 2 anggota yang jika dikalikan akan menghasilkan
bilangan kuadrat. Banyaknya anggota himpunan ini ada 13 + 6(1) = 19.

m
Tetapi jika satu anggota lagi dipilih dari himpunan manapun maka akan ada 2 anggota dari
himpunan tersebut yang jika dikalikan akan menghasilkan bilangan kuadrat sempurna.

co
∴ Jadi, nilai k terkecil yang memenuhi adalah 20.

a.
19. x2 + px + q + 1 = 0 memiliki akar-akar x1 dan x2.
p = −(x2 + x2)

ik
q = x1x2 − 1
p2 + q2 = (x1 + x2)2 + (x1x2 − 1)2 = (x12 + 1)(x22 + 1)

at
Karena p2 + q2 maka salah satu x1 atau x2 sama dengan 0. Tanpa mengurangi keumuman misalkan
x1 = 0.

em
Maka q = −1
p2 + 1 merupakan bilangan prima.
Jika p ganjil maka p2 + 1 prima genap yang hanya dicapai jika p = ±1. Tetapi p juga harus prima.
Maka tidak ada p ganjil yang memenuhi.

at
Jika p genap maka p = 2 yang memenuhi p2 + 1 bilangan prima.
Maka x2 = −p = −2
m
∴ Jadi, x12013 + x22013 = −22013.
an

20. x + x = 5


Jika x bulat maka x = x sehingga tidak mungkin x + x = 5.
at

Jika x tidak bulat maka x − x = 1 yang dapat dicapai jika x = 3 dan x = 2.
Nilai x yang memenuhi hanya jika 2 < x < 3.
∴ Jadi, nilai x yang memenuhi adalah 2 < x < 3.
at
.c
w
w
//w
s:
tp
ht

SMA Negeri 5 Bengkulu Eddy Hermanto, ST

43
Shared by: www.catatanmatematika.com

Olimpiade Sains Nasional Bidang Matematika SMA/MA


Seleksi Tingkat Kota/Kabupaten
Tahun 2014
Waktu: 120 menit

Petunjuk: Untuk masing-masing soal, tulis jawab akhirnya saja (tanpa penjabaran) di lembar
jawab yang disediakan.

m
1. Garis berat AD pada segitiga ABC memotong garis berat CF di P. serta perpanjangan BP

co
memotong ABC di E. Jika diketahui segitiga ABC lancip dan AB = 6, maka panjang DE
adalah ....

a.
2. Diberikan tiga bilangan bulat positif berurutan. Jika bilangan pertama tetap, bilangan kedua

ik
ditambah 10 dan bilangan ketiga ditambah bilangan prima, maka ketiga bilangan ini
membentuk deret ukur. Bilangan ketiga dari bilangan bulat berurutan adalah ....

at
em
1 1
3. Misalkan a, b adalah bilangan riil sedemikian sehingga a + b = + = 6 . Nilai dari
a b
a b
+ + 1980 adalah .... at
b a
m
2014
1 k
4. Nilai dari +∑ adalah ....
2015! k =1 (k + 1)!
an

16 sin 2 x + 9
Untuk 0 < x < π, nilai minimum dari
at

5. adalah ….
sin x
at

6. Misalkan S adalah himpunan bilangan asli yang digitnya tidak berulang dan dipilih dari 1, 3,
.c

5, 7. Jumlah digit satuan dari semua anggota S adalah ….


w

7. Misalkan x, y, z > 1 dan w > 0. Jika logx w = 4, logy w = 5, dan logxyz w = 2, maka nilai logz w
w

adalah ….
//w

8. Terdapat tiga meja bundar yang identik. Setiap meja harus dapat ditempuh minimal satu
siswa. Banyaknya cara mendudukkan enam siswa pada meja-meja tersebut adalah ….
s:

9. Diberikan persegi ABCD dengan panjang sisi 1 satuan. Titik E dan F berturut-turut berada
tp

pada sisi BC dan CD sehingga AEF samasisi. Dibuat pula persegi yang melewati B yang sisi-
sisinya sejajar dengan ABCD dengan salah satu titik sudutnya berada pada ruas garis AE,
ht

a− b
namun bukan A bukan pula E. Jika panjang sisi persegi yang lebih kecil adalah
c
dengan a, b, c bilangan bulat positif dan b tidak habis dibagi suatu bilangan kuadat sempurna
lebih dari 1, maka nilai a + b + c adalah ….

44
Shared by: www.catatanmatematika.com
10. Suatu perusahaan permen memproduksi empat macam rasa permen. Permen dijual dalam
bungkus, setiap bungkus berisi 10 permen dengan setiap rasa permen ada dalam bungkus.
Banyaknya macam variasi isi bungkusan permen adalah ….

11. Bilangan-bilangan 1111, 5276, 8251, dan 9441 bersisa sama jika dibagi N. Nilai N terbesar
yang memiliki sifat tersebut adalah ….

12. Ada sebanyak 6! permutasi dari huruf-huruf OSNMAT. Jika semua permutasi tersebut

m
diurutkan secara abjad dari A ke Z, maka OSNMAT pada urutan ke ….

co
13. Segitiga ABC merupakan segitiga sama kaki dengan panjang AB = AC = 10 cm. Titik D
terletak pada garis AB sejauh 7 cm dari A dan E titik pada garis AC yang terletak sejauh 4 cm

a.
𝑎
dari A. Dari A ditarik garis tinggi dan memotong BC di F. Jika bilangan rasional 𝑏

ik
menyatakan perbandingan luas segi empat ADFE terhadap luas segitiga ABC dalam bentuk
yang paling sederhana, maka nilai a + b adalah ….

at
em
14. Hasil kali semua akar real dari persamaan 2x2 + 3x + 4 = 2 2 x 2 + 3 x + 12 adalah ….

15. Diberikan segitiga ABC dengan AB = 360, BC = 240, dan AC = 180. Garis bagi dalam dan
at
garis bagi luar dari ∠CAB memotong BC dan perpanjangan BC berturut-turut di P dan Q.
Jari-jari lingkaran yang melalui titik-titik A, P, dan Q adalah ….
nm

16. Diberikan fungsi kuadrat f(x) = ax2 + bx + c yang didefinisikan pada himpunan bilangan real
a+c
ta

dengan b > 0. Jika f(x) selalu tak negatif, maka nilai terkecil yang mungkin untuk
b
adalah ….
a
at

17. Semua pasangan bilangan prima (p, q) yang memenuhi persamaan (7p – q)2 = 2(p – 1)q2
adalah ….
.c
w

18. Diberikan segitiga ABC yang sisi-sisinya tidak sama panjang sehingga panjang garis berat AN
dan BP berturut-turut 3 dan 6. Jika luas segitiga ABC adalah 3 15 , maka panjang garis berat
w

ketiga CM adalah ….
//w

19. Diketahui bahwa


20! + 14! = 243290a0953b4931200.
s:

Nilai a dan b adalah ….


tp

20. Semua bilangan bulat n sehingga n4 – 51n2 + 225 merupakan bilangan prima adalah ….
ht

45
Shared by: www.catatanmatematika.com

Solusi Olimpiade Matematika Tk Kabupaten/Kota 2014

1. Ketiga garis berat bertemu di satu titik. Karena AD dan CF garis berat maka BE juga garis berat.

m
co
𝐶𝐸 𝐶𝐷

a.
Karena = dan ∠ECD = ∠ACB maka ∆CED ≅ ∆CAB.
𝐶𝐴 𝐶𝐵
𝐶𝐸 𝐶𝐷 1 𝐴𝐵
Karena = = maka 𝐷𝐸 = =3

ik
𝐶𝐴 𝐶𝐵 2 2
∴ Jadi, panjang DE = 3.

at
2. Misalkan bilangan pertama adalah x maka bilangan kedua adalah x + 1 dan bilangan ketiga x + 2.

em
x, x + 11, x + 2 + p membentuk deret ukur (geometri) dengan p adalah bilangan prima. Maka
(x + 11)2 = x(x + 2 + p)
22x + 121 = (2 + p)x
Maka x121

at
• Jika x = 1
Maka p = 141 = 3 ⋅ 47 (tidak memenuhi p bilangan prima)
m
• Jika x = 11
33 = 2 + p sehingga p = 31
an
• Jika x = 121
23 = 2 + p sehingga p = 21 (tidak memenuhi p bilangan prima)
Maka nlai x yang memenuhi adalah x = 11 dengan p = 31.
at

∴ Jadi, bilangan ketiga dari bilangan bulat berurutan adalah 13.


at

1 1
3. Diketahui 𝑎 + 𝑏 = + = 6
𝑎 𝑏
.c

1 1 𝑎+𝑏
+ = =6
𝑎 𝑏 𝑎𝑏
Karena 𝑎 + 𝑏 = 6 maka 𝑎𝑏 = 1
w

𝑎 𝑏 𝑎2 + 𝑏 2 (𝑎 + 𝑏)2 − 2𝑎𝑏 62 − 2(1)


+ + 1980 = + 1980 = + 1980 = + 1980 = 34 + 1980 = 2014
w

𝑏 𝑎 𝑎𝑏 𝑎𝑏 1
𝑎 𝑏
∴ Jadi, nilai dari + + 1980 adalah 2014.
//w

𝑏 𝑎

𝑘 1 1
4. (𝑘+1)!
= − (𝑘+1)!
s:

𝑘!
2014 2014
𝑘 1 1 1 1 1 1 1 1 1 1
� =�� − � =� − � + � − � + ⋯+ � − �= −
tp

(𝑘 + 1)! 𝑘! (𝑘 + 1)! 1! 2! 2! 3! 2014! 2015! 1! 2015!


𝑘=1 𝑘=1
2014
1 𝑘 1 1 1
ht

+� = + − =1
2015! (𝑘 + 1)! 2015! 1! 2015!
𝑘=1
𝟏 𝒌
∴ Jadi, + ∑𝟐𝟎𝟏𝟒
𝒌=𝟏 =𝟏
𝟐𝟎𝟏𝟓! (𝒌+𝟏)!

SMA Negeri 5 Bengkulu Eddy Hermanto, ST

46
Shared by: www.catatanmatematika.com

Olimpiade Matematika Tk Kabupaten/Kota 2014

5. Berdasarkan ketaksamaan AM-GM didapat


16 sin2 𝑥 + 9 9 9
= 16 sin 𝑥 + ≥ 2�16 sin 𝑥 ∙ = 24
sin 𝑥 sin 𝑥 sin 𝑥

m
9
Tanda kesamaan terjadi jika 16 sin 𝑥 = yang memenuhi syarat 0 < 𝑥 < 𝜋.
sin 𝑥
2

co
16 sin 𝑥+9
∴ Jadi, nilai minimum dari adalah 24.
sin 𝑥

a.
6. Anggota S terdiri dari 1 digit ada 4 yang setiap digit muncul sebagai satuan 1 kali.
Anggota S terdiri dari 2 digit ada 4 ⋅ 3 = 12 yang setiap digit muncul sebagai satuan 3 kali.

ik
Anggota S terdiri dari 3 digit ada 4 ⋅ 3 ⋅ 2 = 24 yang setiap digit muncul sebagai satuan 6 kali.
Anggota S terdiri dari 4 digit ada 4! = 24 yang setiap digit muncul sebagai satuan 6 kali.

at
Jadi, masing-masing digit akan muncul sebanyak 16 kali.
Jumlah digit satuan dari semua anggota S = 16 ⋅ (1 + 3 + 5 + 7) = 256

em
∴ Jadi, Jumlah digit satuan dari semua anggota S adalah 256.

7. Alternatif 1 :

at
1
log𝑥 𝑤 = 4 sehingga log𝑤 𝑥 = . 4
1
log𝑦 𝑤 = 5 sehingga log𝑤 𝑦 = .
m
5
1
log𝑥𝑦𝑧 𝑤 = 2 sehingga log𝑤 𝑥𝑦𝑧 = 2.
an
1
log𝑤 𝑥 + log𝑤 𝑦 + log𝑤 𝑧 = log𝑤 𝑥𝑦𝑧 sehingga log𝑤 𝑧 = 20.
∴ Jadi, nilai 𝐥𝐨𝐠 𝒛 𝒘 = 𝟐𝟎
at

Alternatif 2 :
log𝑥 𝑤 = 4 sehingga 𝑥 = 𝑤 1/4
at

log𝑦 𝑤 = 5 sehingga 𝑦 = 𝑤 1/5.


log𝑥𝑦𝑧 𝑤 = 2 sehingga 𝑥𝑦𝑧 = 𝑤 1/2.
.c

�𝑤 1/4 ��𝑤 1/5 �𝑧 = 𝑤 1/2


w

1 1 1
� − − �
𝑧 = 𝑤 2 4 5 = 𝑤 1/20
∴ Jadi, nilai 𝐥𝐨𝐠 𝒛 𝒘 = 𝟐𝟎
w
//w

8. Kemungkinan susunan keenam siswa adalah :


• Susunannya adalah 4, 1, 1.
�64��21��11�(4 − 1)! = 180
s:

Terdapat perhitungan ganda pada perhitungan di atas. Contoh : A, B, C. D berada di meja I,


E di meja II dan F di meja III dianggap berbeda dengan A, B, C. D berada di meja I, F di meja
tp

II dan E di meja III padahal seharusnya sama. Maka perhitungan tersebut harus dibagi 2!.
�6��2��11�(4−1)!
Jadi, banyaknya susunan = 4 1 = 90
ht

2!
• Susunannya adalah 3, 2, 1.
�63��32��11�(3 − 1)! (2 − 1)! = 120

SMA Negeri 5 Bengkulu Eddy Hermanto, ST

47
Shared by: www.catatanmatematika.com

Olimpiade Matematika Tk Kabupaten/Kota 2014

• Susunannya adalah 2, 2, 2.
�62��42��22�
3!
= 15
Jadi, banyaknya susunan seluruhnya = 90 + 120 + 15 = 225.

m
∴ Jadi, susunan keenam siswa tersebut adalah 225.

co
9. Misalkan panjang sisi persegi yang melalui B adalah 𝑦.

a.
ik
at
em
Alternatif 1 :
Karena simetris maka ∠EAB = ∠FAD = 15o.

at
√3 − 1
tan 15𝑜 = tan(60𝑜 − 45𝑜 ) = = 2 − √3
√3 + 1
𝐺𝐻 𝑦
m
= = tan 15𝑜 = 2 − √3
𝐴𝐻 1 − 𝑦
an
𝑦�1 + 2 − √3� = 2 − √3
2 − √3 3 − √3 𝑎 − √𝑏
𝑦= = =
3 − √3 6 𝑐
at

Maka 𝑎 = 3 ; 𝑏 = 3 dan 𝑐 = 6
∴ Jadi, nilai 𝑎 + 𝑏 + 𝑐 adalah 12.
at

Alternatif 2 :
.c

Karena simetris maka 𝐵𝐸 = 𝐷𝐹 = 𝑥.


𝐴𝐹 = 𝐸𝐹
12 + 𝑥 2 = (1 − 𝑥)2 + (1 − 𝑥)2
w

𝑥 2 − 4𝑥 + 1 = 0
w

𝑥 = 2 − √3
∆𝐸𝐴𝐵 ≅ ∆𝐺𝐴𝐻
//w

𝐺𝐻 𝐸𝐵
=
𝐴𝐻 𝐴𝐵
𝑦 2 − √3
=
s:

1−𝑦 1
𝑦�1 + 2 − √3� = 2 − √3
tp

2 − √3 3 − √3 𝑎 − √𝑏
𝑦= = =
3 − √3 6 𝑐
ht

Maka 𝑎 = 3 ; 𝑏 = 3 dan 𝑐 = 6
∴ Jadi, nilai 𝑎 + 𝑏 + 𝑐 adalah 12.

SMA Negeri 5 Bengkulu Eddy Hermanto, ST

48
Shared by: www.catatanmatematika.com

Olimpiade Matematika Tk Kabupaten/Kota 2014

10. Persoalan setara dengan menyelesaian persamaan


x1 + x2 + x3 + x4 = 10 dengan x1, x2, x3, x4 ∈ N.
Misalkan a = x1 − 1 ; b = x2 − 1 ; c = x3 − 1 ; d = x4 − 1 maka
a + b + c + d = 6 dengan a, b, c, d ∈ N0.

m
6+4−1
Maka banyaknya tupel bilangan (a, b, c, d) yang memenuhi = � � = 84.
3

co
∴ Jadi, banyaknya macam variasi isi bungkusan permen adalah 84.

a.
11. 1111 ≡ 5276 ≡ 8251 ≡ 9441 ≡ k (mod N)
9441 − 8251 = 1190 = 2 ⋅ 595 ≡ 0 (mod N) ; 9441 − 5276 = 4165 = 7 ⋅ 595 ≡ 0 (mod N)

ik
9441 − 1111 = 8330 = 14 ⋅ 595 ≡ 0 (mod N) ; 8251 − 5276 = 2975 = 5 ⋅ 595 ≡ 0 (mod N)
8251 − 1111 = 7140 = 12 ⋅ 595 ≡ 0 (mod N) ; 5276 − 1111 = 4165 = 7 ⋅ 595 ≡ 0 (mod N)

at
FPB (1190, 4165, 8330, 2975, 7140) = 595
Maka nilai N terbesar yang memenuhi adalah 595.

em
∴ Jadi, nilai N terbesar yang memiliki sifat tersebut adalah 595

12. Urutan abjad adalah A, M, N, O, S, T.

at
Maka NTSOMA berada pada urutan 3 ⋅ 5! = 360.
ONTSMA berada pada urutan 360 + 3 ⋅ 4! = 432.
m
OSMTNA berada pada urutan 432 + 2 ⋅ 3! = 444.
Urutan berikutnya adalah OSNAMT, OSNATM, OSNMAT, OSNMTA, OSNTAM, OSNTMA.
an
Maka OSNMAT berada pada urutan ke-447.
∴ Jadi, OSNMAT pada urutan ke-447.
at

13. AF adalah garis tinggi.


at
.c
w
w

Karena AB = AC maka F adalah pertengahan BC sehingga BF = FC.


//w

1 1 3 1 3
[𝐵𝐷𝐹] = ∙ 𝐵𝐷 ∙ 𝐵𝐹 ∙ sin ∠𝐴𝐵𝐶 = ∙ ∙ 𝐴𝐵 ∙ ∙ 𝐵𝐶 ∙ sin ∠𝐴𝐵𝐶 = [𝐴𝐵𝐶]
2 2 10 2 20
1 1 6 1 3
[𝐶𝐸𝐹] = ∙ 𝐶𝐸 ∙ 𝐶𝐹 ∙ sin ∠𝐴𝐶𝐵 = ∙ ∙ 𝐴𝐶 ∙ ∙ 𝐵𝐶 ∙ sin ∠𝐴𝐶𝐵 = [𝐴𝐵𝐶]
s:

2 2 10 2 10
11
[𝐴𝐷𝐹𝐸] = [𝐴𝐵𝐶] − [𝐵𝐷𝐹] − [𝐶𝐸𝐹] = [𝐴𝐵𝐶]
tp

20
[𝐴𝐷𝐹𝐸] 11
=
ht

[𝐴𝐵𝐶] 20
Maka 𝑎 = 11 dan 𝑏 = 20
∴ Jadi, nilai 𝑎 + 𝑏 adalah 31.

SMA Negeri 5 Bengkulu Eddy Hermanto, ST

49
Shared by: www.catatanmatematika.com

Olimpiade Matematika Tk Kabupaten/Kota 2014

14. 2𝑥2 + 3𝑥 + 4 = 2�2𝑥2 + 3𝑥 + 12


Misalkan 𝑝 = �2𝑥2 + 3𝑥 + 12
Karena akar suatu bilangan real tidak mungkin negatif maka 𝑝 ≥ 0.

m
𝑝2 − 8 = 2𝑝
(𝑝 − 4)(𝑝 + 2) = 0

co
Karena 𝑝 ≥ 0 maka 𝑝 = 4
2𝑥2 + 3𝑥 + 12 = 42
2𝑥2 + 3𝑥 − 4 = 0

a.
𝐶
Hasil kali semua akar = = −2
𝐴
∴ Jadi, hasil kali semua akar real sama dengan −2.

ik
at
15. AB = 360 ; AC = 180 dan BC = 240

em
at
m
𝐴𝐵 𝐵𝑃 360 𝐵𝑃
Karena AP garis bagi maka = sehingga =
𝐴𝐶 𝑃𝐶 180 240−𝐵𝑃
an
Maka BP = 160 dan PC = 80
𝐴𝐵 𝐵𝑄 360 240+𝐶𝑄
Karena AQ garis bagi maka = sehingga =
𝐴𝐶 𝐶𝑄 180 𝐶𝑄
Maka CQ = 240
at

Maka PQ = PC + CQ = 80 + 240 = 320


2 ⋅ ∠CAQ + 2 ⋅ ∠CAP = 180o
at

∠PAQ = ∠CAQ + ∠CAP = 90o


Karena ∠PAQ = 90o maka PQ adalah diameter lingkaran yang melalui titik A, P dan Q.
.c

∴ Jadi, jari-jari lingkaran yang melalui titik-titik A, P, dan Q adalah 160.


w

16. 𝑓(𝑥) = 𝑎𝑥2 + 𝑏𝑥 + 𝑐


w

Penyelesaian soal berikut didasrkan pada soal asli yang telah disesuaikan. Pada soal aslinya
dinyatakan bahwa 𝑓(𝑥) selalu positif dan itu diganti menjadi 𝑓(𝑥) tidak pernah negatif. Selain
//w

itu dalam soal aslinya tertulis 𝑏 ≠ 0 dan diganti menjadi 𝑏 > 0.


Syarat 𝑓(𝑥) tidak pernah negatif adalah 𝑎 > 0 dan 𝑏 2 − 4𝑎𝑐 ≤ 0
Karena 4𝑎𝑐 ≥ 𝑏 2 > 0 maka 𝑐 > 0
𝑎𝑐 1
s:


𝑏2 4
Dengan ketaksamaan AM-GM didapat
tp

𝑎+𝑐 𝑎 𝑐 𝑎 𝑐 𝑎𝑐
= + ≥ 2� ∙ = 2� 2 = 1
𝑏 𝑏 𝑏 𝑏 𝑏 𝑏
ht

𝑎 𝑐
Tanda kesamaan terjadi jika = atau 𝑎 = 𝑐 dan 𝑏 2 = 4𝑎𝑐 yang setara dengan 𝑏 = 2𝑎 = 2𝑐.
𝑏 𝑏
𝑎+𝑐
∴ Jadi, nilai terkecil yang mungkin untuk adalah 1.
𝑏

SMA Negeri 5 Bengkulu Eddy Hermanto, ST

50
Shared by: www.catatanmatematika.com

Olimpiade Matematika Tk Kabupaten/Kota 2014

17. (7𝑝 − 𝑞)2 = 2(𝑝 − 1)𝑞 2


49𝑝2 − 14𝑝𝑞 + 𝑞 2 = 2(𝑝 − 1)𝑞 2
Maka 𝑞49𝑝2
Karena 𝑝 dan 𝑞 prima maka ada 2 kasus :

m
• Kasus 1, 𝑞 = 7
𝑝2 − 2𝑝 + 1 = 2𝑝 − 2

co
(𝑝 − 3)(𝑝 − 1) = 0
Karena 𝑝 prima maka 𝑝 = 3.
• Kasus 2, 𝑞𝑝

a.
Maka 𝑞 = 𝑝
36𝑝2 = 2(𝑝 − 1)𝑝2

ik
𝑝 = 𝑞 = 19
∴ Jadi, semua pasangan bilangan prima (𝑝, 𝑞) yang memenuhi adalah (𝟑, 𝟕) dan (𝟏𝟗, 𝟏𝟗).

at
em
18. Misalkan panjang BC = 2a, AC = 2b dan AB = 2c
Misalkan garis berat dari C memotong sisi AB di Q dan perpotongan ketiga garis berat di G.
Misalkan juga panjang garis berat CQ = 3k sehingga GQ = k

at
m
an
at

1 3
[𝐴𝐵𝑃] = ∙ 3√15 = √15
at

2 2
Karena PG : GB = 1 : 2 maka [𝐴𝐵𝐺] = √15
.c

Perhatikan ∆ABG.
2𝑐+2+4
𝑠= =𝑐+3
2
w

Dengan rumus Heron didapat


[𝐴𝐵𝐺] = �(𝑐 + 3)(3 − 𝑐)(𝑐 + 1)(𝑐 − 1) = √15
w

𝑐 4 − 10𝑐 2 + 9 = −15
(𝑐 2 − 4)(𝑐 2 − 6) = 0
//w

• Jika 𝑐 = √6
Maka 𝐴𝐵 = 2√6
2 2
22 + �√6� − 𝑘 2 22 + �2√6� − 42
s:

cos ∠𝐺𝐴𝐵 = =
2 ∙ 2 ∙ √6 2 ∙ 2 ∙ 2√6
tp

𝑘=2
Karena GM = GP maka AB = AC. Kontradiksi dengan panjang ketiga sisi berbeda.
• Jika 𝑐 = 2
ht

Maka 𝐴𝐵 = 4
22 + 22 − 𝑘 2 22 + 42 − 42
cos ∠𝐺𝐴𝐵 = =
2∙2∙2 2∙2∙4
SMA Negeri 5 Bengkulu Eddy Hermanto, ST

51
Shared by: www.catatanmatematika.com

Olimpiade Matematika Tk Kabupaten/Kota 2014


𝑘 = √6
𝐶𝑀 = 3𝑘 = 3√6
∴ Jadi, panjang garis berat ketiga, 𝐶𝑀 adalah 𝟑√𝟔.

m
19. 20! + 14! = 243290a0953b4931200
20! + 14! habis dibagi 9 maupun 11.

co
9(2+4+3+2+9+0+a+0+9+5+3+b+4+9+3+1+2+0+0) = 56 + a + b
a + b = 7 atau a + b = 16
11(2−4+3−2+9−0+a−0+9−5+3−b+4−9+3−1+2−0+0) = 14 + a − b

a.
Maka a − b = −3 atau a − b = 8
Berdasarkan paritas (a + b dan a − b memiliki paritas yang sama) maka ada 2 kasus :

ik
• Kasus 1, a + b = 7 dan a − b = −3
Maka didapat a = 2 dan b = 5

at
• Kasus 2, a + b = 16 dan a − b = 8
Maka didapat a = 12 dan b = 4 yang tidak memenuhi syarat bahwa a adalah digit bilangan.

em
Maka a = 2 dan b = 5
∴ Jadi, nilai a dan b adalah a = 2 dan b = 5.

at
20. 𝑛4 − 51𝑛2 + 225 = (𝑛2 + 15)2 − 81𝑛2 = (𝑛2 − 9𝑛 + 15)(𝑛2 + 9𝑛 + 15)
Jika 𝑛 = 𝑘 memenuhi untuk suatu 𝑘 ∈ 𝑁0 maka 𝑛 = −𝑘 juga memenuhi. Maka tanpa mengurangi
m
keumuman dapat dimisalkan 𝑛 ≥ 0.
Karena 𝑛4 − 51𝑛2 + 225 prima dan 𝑛2 + 9𝑛 + 15 ≥ 𝑛2 − 9𝑛 + 15 maka 𝑛2 − 9𝑛 + 15 = 1
an
𝑛2 − 9𝑛 + 14 = 0
(𝑛 − 2)(𝑛 − 7) = 0
𝑛 bulat yang memenuhi adalah 𝑛 = 2 atau 7
at

∴ Jadi, semua bilangan bulat 𝑛 yang memenuhi adalah −𝟕, −𝟐, 𝟐, 𝟕.


at
.c
w
w
//w
s:
tp
ht

SMA Negeri 5 Bengkulu Eddy Hermanto, ST

52
Shared by: www.catatanmatematika.com

Olimpiade Sains Nasional Bidang Matematika SMA/MA


Seleksi Tingkat Kota/Kabupaten
Tahun 2015
Waktu: 120 menit

Petunjuk: Untuk masing-masing soal, tulis jawab akhirnya saja (tanpa penjabaran) di lembar jawab
yang disediakan.

m
1. Banyaknya faktor bulat positif dari 2015 adalah ....

co
2. Suatu dadu ditos enam kali. Probabilitas jumlah mata yang muncul 9 adalah ....

a.
7x  3
3. Jika (f o g)(x) = dan g(x) = 2x – 4, maka nilai f(2) adalah ....

ik
5x  9

at
4. Diberikan trapesium ABCD, dengan AB sejajar DC dan AB = 84 serta DC = 25. Jika trapesium
ABCD memiliki lingkaran dalam yang menyinggung keempat sisinya, keliling trapesium ABCD

em
adalah ....

5. Diketahui barisan bilangan real a1, a2, … , an, … merupakan barisan geometri. Jika a1 + a4 =
at
20, maka nilai minimal dari
nm
a1 + a2 + a3 + a4 + a5 + a6
adalah ….
a

6. Bilangan bulat x jika dikalikan 11 terletak diantara 1500 dan 2000. Jika x dikalikan 7 terletak
antara 970 dan 1275. Jika x dikalikan 5 terletak antara 960 dan 900. Banyaknya bilangan x
at

sedemikian yang habis dibagi 3 sekaligus habis dibagi 5 ada sebanyak ….


at

7. Suatu sekolah mempunyai lima kelompok belajar siswa kelas 11. Kelompok-kelompok belajar
.c

itu berturut-turut mengirimkan 2, 2, 2, 3, dan 3 siswa untuk suatu pertemuan. Mereka akan
duduk melingkar sehingga setiap siswa memiliki paling sedikit satu teman dari kelompok
w

belajar yang sama yang duduk disampingnya. Banyaknya cara melakukan hal tersebut adalah
w

….
//w

8. Diberikan segitiga ABC dengan sudut ABC = 90o. Lingkaran L1 dengan AB sebagai diameter
sedangkan lingkaran L2 dengan BC sebagai diameternya. Kedua lingkaran L1 dan L2
240
s:

berpotongan di B dan P. Jika AB = 5, BC = 12 dan BP = x, maka nilai dari adalah ….


x
tp

9. Diketahui bilangan real positif a dan b memenuhi persamaan


ht

a4 + a2b2 + b4 = 6 dan a2 + ab + b2 = 4
Nilai dari a + b adalah ….

53
Shared by: www.catatanmatematika.com
10. Diketahui susunan 4 × 5 titik yang jarak ke kanan sama dan jarak ke bawah sama. Ada berapa
segitiga (dengan luas positif) yang titik-titik sudutnya adalah ketiga titik pada susunan tersebut?

m
11. Bilangan x adalah bilangan bulat positif terkecil yang membuat
31n + x . 96n

co
merupakan kelipatan 2015 untuk setiap bilangan asli n. Nilai x adalah ….

a.
12. Semua bilangan bulat n yang memenuhi
n8  n 7  n 6  2n5  2n 4  2n3  2n 2  2017

ik
p(n) 
n2  n 1

at
bulat adalah ….

em
13. Diketahui a, b, c akar dari persamaan x3 – 5x2 – 9x + 10 = 0. Jika sukubanyak P(x) = Ax3 + Bx2
+ Cx – 2015 memenuhi P(a) = b + c, P(b) = a + c, P(c) = a + b, maka nilai dari A + B + C
adalah …. at
nm
14. Pada segitiga ABC, garis tinggi AD, garis bagi BE dan garis berat CF berpotongan di satu titik.
Jika panjang AB = 4 dan BC = 5, dan CD = m2/n2 dengan m dan n relatif prima, maka nilai dari
m – n adalah ….
a

15. Banyaknya bilangan asli n ≤ 2015 yang dapat dinyatakan dalam bentuk n = a + b dengan a, b
at

bilangan asli yang memenuhi a – b bilangan prima dan ab bilangan kuadrat sempurna adalah
….
at
.c

16. Tiga titik berbeda B, C, dan D terletak segaris dengan C diantara B dan D. Titik A adalah suatu
titik yang tidak terletak digaris BD dan memenuhi |AB| = |AC| = |CD|. Jika diketahui
w

1 1 1
 
| CD | | BD | | CD |  | BD |
w

maka besar sudut BAC adalah ….


//w

17. Masing-masing kotak pada papan catur berukuran 3 × 3 dilabeli dengan satu angka, yaitu 1, 2,
s:

atau 3. Banyaknya penomoran yang mungkin sehingga jumlah angka pada masing-masing baris
dan masing-masing kolom habis dibagi oleh 3 adalah ….
tp

18. Pada segilima beraturan ABCDE, diagonal-diagonalnya berpotongan di F, G, H, I dan J.


ht

misalkan S1 menyatakan luas segilima ABCDE dan S2 menyatakan luas segilima FGHIJ. Jika
S1 m  n
 , dengan k, m, n bilangan bulat positif dan n tidak memiliki faktor kuadrat selain
S2 k
1, maka nilai dari k + m + n adalah ….

54
Shared by: www.catatanmatematika.com
19. Suatu permutasi a1, a2, …, a10 dari {1, 2, …, 10} dikatakan sebagai suatu permutasi yang
hampir naik jika terdapat tepat satu indeks i sehingga ai1 > ai. Banyaknya permutasi hampir
naik yang mungkin adalah ….

20. Untuk setiap bilangan real a, didefinisikan f(a) sebagai nilai maksimal dari
2
sin x  a
3  sin x
Nilai maksimal dari f(a) adalah ….

m
co
a.
ik
at
em
at
a nm
at
at
.c
w
w
//w
s:
tp
ht

55
Shared by: www.catatanmatematika.com

Solusi Olimpiade Matematika Tk Kabupaten/Kota 2015

1. 2015 = 5 ⋅ 13 ⋅ 31
Banyaknya faktor positif = 2 ⋅ 2 ⋅ 2 = 8
∴ Jadi, banyaknya faktor bulat positif dari 2015 adalah 8.

m
2. Semua kemungkinan jumlah keenam dadu sama dengan 9 adalah (1,1,1,1,1,4), (1,1,1,1,2,3),

co
(1,1,1,2,2,2).
Maka ada 3 kasus :
a. Kasus 1, jika susunannya adalah (1,1,1,1,1,4).

a.
6!
Banyaknya permutasi adalah = 6
5!
b. Kasus 2, jika susunannya adalah (1,1,1,1,2,3).

ik
6!
Banyaknya permutasi adalah = 30
4!

at
c. Kasus 3, jika susunannya adalah (1,1,1,2,2,2).
6!
Banyaknya permutasi adalah = 20

em
3!3!
Jadi, banyaknya cara = 56.
𝟓𝟔
∴ Jadi, probabilitas jumlah mata yang muncul 9 adalah 𝟔 .
𝟔

3. 𝑔(𝑥) = 2𝑥 − 4
7𝑥 + 3
at
m
(𝑓𝑜𝑔)(𝑥) = 𝑓�𝑔(𝑥)� =
5𝑥 − 9
an
Alternatif 1 :
𝑔(3) = 2(3) − 4 = 2
7(3) + 3
at

𝑓(2) = 𝑓�𝑔(3)� = =4
5(3) − 9
at

Alternatif 2 :
7𝑥 + 3
𝑓(2𝑥 − 4) =
.c

5𝑥 − 9
𝑦+4
Misalkan 𝑦 = 2𝑥 − 4 maka 𝑥 = .
2
w

𝑦+4
7� � + 3 7𝑦 + 34
𝑓(𝑦) = 2 =
w

𝑦+4 5𝑦 + 2
5� �−9
2
Yang setara dengan
//w

7𝑥 + 34
𝑓(𝑥) =
5𝑦 + 2
7(2) + 34
s:

𝑓(2) = =4
5(2) + 2
∴ Jadi, nilai f(2) adalah 4.
tp
ht

4. Jika titik P di luar lingkaran dan garis yang ditarik dari titik P menyinggung lingkaran tersebut di titik Q
dan R maka PQ = PR.

SMA Negeri 5 Bengkulu Eddy Hermanto, ST

56
Shared by: www.catatanmatematika.com

Olimpiade Matematika Tk Kabupaten/Kota 2015

m
co
Dari gambar di atas didapat DG = DH ; CG = CF ; BF = BE ; AE = AH
Keliling = AE + AH + BE + BF + CF + CG + DG + DH = 2 (DG + CG + AE + BE)
Keliling = 2(DC + AB) = 2(25 + 84)

a.
∴ Keliling trapesium = 218

ik
5. Misalkan 𝑎1 , 𝑎2 , 𝑎3 , ⋅⋅⋅⋅ merupakan barisan geometri dengan rasio 𝑟 dan 𝑎1 = 𝑎.

at
𝑎1 + 𝑎3 = 𝑎(1 + 𝑟 3 ) = 20
𝑎(𝑟 6 − 1)
𝑎1 + 𝑎2 + 𝑎3 + 𝑎4 + 𝑎5 + 𝑎6 = = 𝑎(𝑟 3 + 1)(𝑟 2 + 𝑟 + 1) = 20(𝑟 2 + 𝑟 + 1)

em
𝑟 −2 1
4𝐴𝐶−𝐵 3
Karena (𝐴𝑥 2 + 𝐵𝑥 + 𝐶)𝑚𝑖𝑛 = 4𝐴 untuk 𝐴 > 0 maka (𝑟 2 + 𝑟 + 1)𝑚𝑖𝑛 = .
4
3
Maka nilai minimum dari 𝑎1 + 𝑎2 + 𝑎3 + 𝑎4 + 𝑎5 + 𝑎6 adalah 20 ∙ = 15.
4

at
∴ Jadi, nilai minimum dari 𝑎1 + 𝑎2 + 𝑎3 + 𝑎4 + 𝑎5 + 𝑎6 adalah 𝟏𝟓.
m
6. 1500 < 11x < 2000 sehingga 136 < x < 182
970 < 7x < 1275 sehingga 138 < x < 183
an

690 < 5x < 900 sehingga 138 < x < 180


Maka 138 < x < 180
Bilangan yang habis dibagi 3 dan 5 maka bilangan tersebut habis dibagi 15.
at

Bilangan yang habis dibagi 15 ada 2 yaitu 150 dan 165.


∴ Jadi, banyaknya bilangan yang memenuhi ada 2.
at
.c

7. Karena setiap siswa memiliki paling sedikit satu teman dari kelompok belajar yang sama yang
duduk disampingnya maka setiap siswa dalam kelompok belajar yang sama akan duduk
w

berdekatan. Jika setiap kelompok dinyatakan sebagai obyek maka akan ada 5 obyek yang duduk
membentuk lingkaran serta ada permutasi susunan duduk siswa pada masing-masing obyek.
w

Banyaknya cara melakukan = (5 − 1)! ⋅ 2! ⋅ 2! ⋅ 2! ⋅ 3! ⋅ 3! = 6912.


∴ Jadi, banyaknya cara melakukan hal tersebut adalah 6912.
//w

8. Misalkan titik R terletak pada sisi AC sehingga BR tegak lurus AC.


s:
tp
ht

SMA Negeri 5 Bengkulu Eddy Hermanto, ST

57
Shared by: www.catatanmatematika.com

Olimpiade Matematika Tk Kabupaten/Kota 2015

Karena ∠ARB = 90o maka lingkaran berdiameter AB akan melalui titik R.


Karena ∠BRC = 90o maka lingkaran berdiameter BC akan melalui titik R.
Jadi, titik R = P.
AC ⋅ BP = AB ⋅ BC

m
13 ⋅ BP = 5 ⋅ 12
60

co
𝐵𝑃 = 𝑥 =
13
240
= 52
𝑥

a.
240
∴ Jadi, nilai dari adalah 52.
𝑥

ik
9. 𝑎4 + 𝑎2 𝑏 2 + 𝑏 4 = 6

at
𝑎2 + 𝑎𝑏 + 𝑏 2 = 4
𝑎4 + 𝑏 4 + 𝑎2 𝑏 2 + 2𝑎2 𝑏 2 + 2𝑎𝑏(𝑎2 + 𝑏 2 ) = 42

em
2𝑎2 𝑏 2 + 2𝑎𝑏(4 − 𝑎𝑏) = 10
Misalkan 𝑦 = 𝑎𝑏 > 0
5
𝑎𝑏 =

at
4
5 21
(𝑎 + 𝑏)2 = 𝑎2 + 𝑏 2 + 𝑎𝑏 + 𝑎𝑏 = 4 + =
4 4
m
√21
𝑎+𝑏 =
2
an
√𝟐𝟏
∴ Jadi, nilai 𝑎 + 𝑏 adalah .
𝟐
at

10. Segitiga dibentuk dari 3 titik. Maka banyaknya segitiga = 20C3 = 1140.
Agar luas segitiga positif maka ketiga titik tidak boleh berada pada satu garis lurus. Maka akan
at

dicari banyaknya 3 titik yang berada pada satu garis lurus.


• Pada arah horisontal
.c

Ada 4 buah 5 titik berada pada satu garis lurus.


Banyaknya 3 titik yang berada pada satu garis lurus = 4 ⋅ 5C3 = 40.
w

• Pada arah vertikal


Ada 5 buah 4 titik berada pada satu garis lurus.
w

Banyaknya 3 titik yang berada pada satu garis lurus = 5 ⋅ 4C3 = 20.
• Pada arah diagonal
//w

* 4 titik berada pada satu garis lurus


Ada 2 buah dengan gradien 1 dan ada 2 buah dengan gradien −1.
Maka banyaknya 3 titik yang berada pada satu garis lurus = 4 ⋅ 4C3 = 16.
s:

* 3 titik berada pada satu garis lurus


Ada 2 buah dengan gradien 1, ada 2 buah dengan gradien −1, ada 2 buah dengan gradien
tp

1/2 dan ada 2 buah dengan gradien −1/2.


Maka banyaknya 3 titik yang berada pada satu garis lurus = 8 ⋅ 3C3 = 8.
ht

Maka banyaknya segitiga dengan luas positif = 1140 − 40 − 20 − 16 − 8 = 1056.


∴ Jadi, banyaknya segitiga dengan luas positif adalah 1056.

SMA Negeri 5 Bengkulu Eddy Hermanto, ST

58
Shared by: www.catatanmatematika.com

Olimpiade Matematika Tk Kabupaten/Kota 2015

11. 31n + x ⋅ 96n habis dibagi 2015 = 5 ⋅ 13 ⋅ 31 maka


31n + x ⋅ 96n ≡ 1n + x ⋅ 1n (mod 5) ≡ 1 + x (mod 5)
Jadi, x ≡ −1 (mod 5)
31n + x ⋅ 96n ≡ 5n + x ⋅ 5n (mod 13)

m
Jadi, x ≡ −1 (mod 13)
31n + x ⋅ 96n ≡ x ⋅ 3n (mod 31)

co
FPB (3, 31) = 1 maka x ≡ 0 (mod 31)
Maka x = 31a dengan a ∈ N
31a ≡ −1 (mod 13)

a.
5a ≡ −1 (mod 13)
a = 13b + 5 dengan b ∈ N

ik
x = 31(13b + 5) = 403b + 155
403b + 155 ≡ −1 (mod 5)

at
403b ≡ −1 (mod 5)
3b ≡ −1 (mod 5)

em
Maka b = 5c + 3
x = 403b + 65 = 403(5c + 3) + 155 = 2015c + 1364 dengan c ∈ N
∴ Jadi, nilai terkecil x yang memenuhi adalah 1364.

12. n3 + 1 = (n + 1)(n2 − n + 1)
at
m
Maka n2 − n + 1 membagi n3 + 1
Misalkan y = n8 + n7 + n6 + 2n5 + 2n4 + 2n3 + 2n2 + 2017
an
y = n5(n3 + 1) + n4(n3 + 1) + n3(n3 + 1) + n2(n3 + 1) + n(n3 + 1) + n3 + 1 + n2 − n + 1 + 2015
Maka haruslah n2 − n + 1 membagi 2015.
1 2 3
at

𝑛2 − 𝑛 + 1 = �𝑛 − � +
2 4
Maka n2 − n + 1 ≥ 1
at

Ada 8 kasus :
• Jika n2 − n + 1 = 1
.c

Maka n = 0 atau n = 1
• Jika n2 − n + 1 = 5
w

Tidak ada n bulat yang memenuhi.


• Jika n2 − n + 1 = 13
w

Maka n = 4 atau n = −3
• Jika n2 − n + 1 = 31
//w

Maka n = 6 atau n = −5
• Jika n2 − n + 1 = 65
Tidak ada n bulat yang memenuhi.
s:

• Jika n2 − n + 1 = 155
Tidak ada n bulat yang memenuhi.
• Jika n2 − n + 1 = 403
tp

Tidak ada n bulat yang memenuhi.


• Jika n2 − n + 1 = 2015
ht

Tidak ada n bulat yang memenuhi.


∴ Jadi, semua n bulat yang memenuhi adalah −5, −3, 0, 1, 4, 6.

SMA Negeri 5 Bengkulu Eddy Hermanto, ST

59
Shared by: www.catatanmatematika.com

Olimpiade Matematika Tk Kabupaten/Kota 2015

13. x3 – 5x2 – 9x + 10 = 0 akar-akarnya a, b dan c.


a+b+c=5
P(x) = Ax3 + Bx2 + Cx – 2015
P(a) = b + c = 5 − a

m
Aa3 + Ba2 + Ca – 2015 = 5 − a
Aa3 + Ba2 + (C + 1)a – 2020 = 0 ⋅⋅⋅⋅⋅⋅⋅⋅⋅⋅⋅⋅ (1)

co
P(b) = a + c = 5 − b
Ab3 + Bb2 + Cb – 2015 = 5 − a
Ab3 + Bb2 + (C + 1)b – 2020 = 0 ⋅⋅⋅⋅⋅⋅⋅⋅⋅⋅⋅⋅ (2)

a.
P(c) = a + b = 5 − c
Ac3 + Bc2 + Cc – 2015 = 5 − a

ik
Ac3 + Bc2 + (C + 1)c – 2020 = 0 ⋅⋅⋅⋅⋅⋅⋅⋅⋅⋅⋅⋅ (3)
Berdasarkan (1), (2) dan (3) maka Ax3 + Bx2 + (C + 1)x − 2020 = 0 juga akan memiliki akar-akar a,

at
b dan c.
Dengan membandingkan persamaan di atas dengan persamaan x3 – 5x2 – 9x + 10 = 0 didapat A =

em
−202 ; B = −202(−5) = 1010 dan C + 1 = −202(−9) = 1818 sehingga C = 1817
A + B + C = −202 + 1010 + 1817 = 2625
∴ Jadi, nilai A + B + C adalah 2625.

14. Karena CF adalah garis berat maka AF = FB = 2


at
m
an
at
at
.c

Karena BE adalah garis bagi maka


w

𝐶𝐸 𝐵𝐶 5
= =
𝐸𝐴 𝐴𝐵 4
w

Ketiga garis bertemu di satu titik maka sesuai dali Ceva didapat
𝐴𝐹 𝐵𝐷 𝐶𝐸
//w

∙ ∙ =1
𝐹𝐵 𝐷𝐶 𝐸𝐴
2 𝐵𝐷 5
∙ ∙ =1
2 𝐷𝐶 4
𝐵𝐷 4
s:

Maka =
𝐷𝐶 5
Misalkan BD = 4x maka CD = 5x
tp

5
BD + CD = 5 maka x =
9
25 𝑚2
Maka panjang 𝐶𝐷 = = 𝑛2
ht

9
Didapat m = 5 dan n = 3.
∴ Jadi, nilai m − n adalah 2.

SMA Negeri 5 Bengkulu Eddy Hermanto, ST

60
Shared by: www.catatanmatematika.com

Olimpiade Matematika Tk Kabupaten/Kota 2015

15. n = a + b dengan n ≤ 2015 dan n, a, b ∈ N.


Jelas bahwa b < a
a + b = n ≤ 2015
2b < n ≤ 2015 maka b ≤ 1007

m
Andaikan FPB(a, b) = d
Maka a = dp dan b = dq

co
a − b = d(p − q) merupakan bilangan prima. Maka d = 1
Karena ab kuadrat sempurna sedangkan FPB (a, b) = 1 maka haruslah a dan b masing-masing
kuadrat sempurna.

a.
Misalkan a = m2 dan b = t2
t2 ≤ 1007 sehingga t ≤ 31

ik
a − b = m2 − t2 = (m + t)(m − t) adalah bilangan prima.
Maka m − t = 1

at
a − b = (t + 1)2 − t2 = 2t + 1 ≤ 63 adalah bilangan pima ganjil.
Bilangan prima ganjil ≤ 63 adalah 3, 5, 7, 11, 13, 17, 19, 23, 29, 31, 37, 41, 43, 47, 53, 59 dan

em
61. Banyaknya nilai b yang memenuhi ada 17.
Maka banyaknya nilai n yang memenuhi ada 17.
∴ Jadi, banyaknya nilai n yang memenuhi ada 17.

at
16. Misalkan panjang BC = 2y dan AB = AC = CD = x. Titik E pertengahan BC sehingga BE = EC = y.
m
∠BAE = ∠CAE
cos 3α = 4 cos3α − 3 cos α
an
sin (36o) = sin (90o − 54o) = cos 54o
2 sin 18o cos 18o = 4 cos318o − 3 cos 18o
2 sin 18o = 4 − 4 sin218o − 3
at

4 sin218o + 2 sin 18o − 1 = 0


√5 − 1
sin 18𝑜 =
at

4
.c
w
w
//w

1 1 1
s:

− =
|𝐶𝐷| |𝐵𝐷| |𝐵𝐷| + |𝐶𝐷|
|𝐵𝐷|2 − |𝐶𝐷|2 = |𝐵𝐷||𝐶𝐷|
tp

(2𝑦 + 𝑥)2 − 𝑥 2 = (2𝑦 + 𝑥)(𝑥)


𝑦 2 𝑦
ht

4� � +2� � −1 = 0
𝑥 𝑥
𝑦 −2 + �22 − 4(4)(−1) √5 − 1
= =
𝑥 8 4
SMA Negeri 5 Bengkulu Eddy Hermanto, ST

61
Shared by: www.catatanmatematika.com

Olimpiade Matematika Tk Kabupaten/Kota 2015


𝑦 √5 − 1
sin ∠𝐵𝐴𝐸 = =
𝑥 4
∠𝐵𝐴𝐸 = 18𝑜
Maka ∠BAC = 36o.
∴ Jadi, besar ∠BAC adalah 36o.

m
co
17. Misalkan bilangan-bilangan pada baris pertama adalah a, b dan c. Pada baris kedua adalah d, e,
f dan baris ketiga g, h, i.
Jika a = b maka agar memenuhi a + b + c habis dibagi 3 maka a = b = c.

a.
Jika a ≠ b maka agar memenuhi a + b + c habis dibagi 3 maka a, b, c semuanya berbeda dengan
a, b, c ∈ {1, 2, 3}.

ik
Maka masing-masing ada 3 kemungkinan untuk nilai a dan b. Nilai c menyesuaikan sehingga
hanya ada 1 kemungkinan.

at
Maka masing-masing ada 3 kemungkinan untuk nilai d dan e. Nilai f menyesuaikan sehingga
hanya ada 1 kemungkinan.

em
Jelas nilai g, h, i hanya menyesuaikan dengan bilangan-bilangan di atasnya. Jadi, masing-masing
hanya ada 1 kemungkinan.
Cukup membuktikan bahwa jika a + b + c, d + e + f, a + d + g, b + e + h dan c + f + i masing-
masing habis dibagi 3 maka g + h + i juga habis dibagi 3.

at
g = 3k − a − d, h = 3m − b − e dan i = 3n − c − f
g + h + i = 3(k + m + n) − (a + b + c) − (d + e + f) yang habis dibagi 3.
m
Jadi, banyaknya kemungkinan yang memenuhi ada 3 x 3 x 3 x 3 = 81.
∴ Jadi, banyaknya penomoran yang memenuhi adalah 81.
an

18. cos 3α = 4 cos3α − 3 cos α


at

sin (36o) = sin (90o − 54o) = cos 54o


2 sin 18o cos 18o = 4 cos318o − 3 cos 18o
2 sin 18o = 4 − 4 sin218o − 3
at

4 sin218o + 2 sin 18o − 1 = 0


√5 − 1
.c

sin 18𝑜 =
4
w
w
//w
s:
tp

360𝑜
∠AOB = = 72o sehingga ∠ABC = 108o.
5
Maka ∠BAC = 36o.
ht

𝐴𝐶 sin 108𝑜 sin 72𝑜


= 𝑜
= 𝑜
= 2 cos 36𝑜
𝐴𝐵 sin 36 sin 36
∠ABK = 54o sehingga ∠FBK = 54o − 36o = 18o.

SMA Negeri 5 Bengkulu Eddy Hermanto, ST

62
Shared by: www.catatanmatematika.com

Olimpiade Matematika Tk Kabupaten/Kota 2015


𝐹𝐺 𝐹𝐾 tan 18𝑜 sin 18𝑜 sin 36𝑜
= = = ∙
𝐴𝐶 𝐴𝐾 tan𝑜54𝑜 cos 𝑜
18𝑜 cos 36𝑜
𝐹𝐺 2 ∙ sin 18 ∙ sin 36
= = (2 ∙ sin 18𝑜 )2
𝐴𝐵 cos 18𝑜
Segilima ABCDE dan FGHIJ sebangun maka perbandingan luas dapat dinyatakan sebagai kuadrat

m
perbandingan sisi-sisinya.
𝑆1 𝐴𝐵 2 1 4
2 4
7 + 3√5

co
=� � =� 𝑜
� =� � =
𝑆2 𝐹𝐺 2 ∙ sin 18 √5 − 1 2
𝑆1 𝟕+𝟑√𝟓
∴ Jadi, nilai 𝑆2
adalah 𝟐
.

a.
ik
19. Misalkan p, q ∈ {1, 2, 3, ⋅⋅⋅, 10} dengan p < q serta ai-1 = q dan ai = p.
Maka semua bilangan kurang dari p akan berada di kiri q dan semua bilangan lebih dari q akan

at
berada di kanan p.
Semua bilangan di antara p dan q bisa berada di kiri q maupun di kanan p.

em
Maka persoalannya setara dengan banyaknya cara memilih bilangan di antara p dan q untuk
ditaruh di kiri q.
Misalkan n adalah banyaknya bilangan di antara p dan q dengan n ∈ {0 1, 2, ⋅⋅⋅, 8}
Maka banyaknya cara memilih bilangan = nC0 + nC1 + nC2 + ⋅⋅⋅ + nCn = 2n.

at
Banyaknya pasangan (p, q) untuk n = 0, 1, 2 ⋅⋅⋅, 8 berturut-turut adalah 9, 8, 7, ⋅⋅⋅, 1.
Banyaknya permutasi hampir naik = 9 ⋅ 20 + 8 ⋅ 21 + 7 ⋅ 22 + ⋅⋅⋅ + 1 ⋅ 28 = 1013.
m
∴ Jadi, banyaknya permutasi hampir naik adalah 1013.
an

2
20. 𝑓(𝑎) adalah nilai maksimum dari �sin 𝑥 + + 𝑎� untuk 𝑎 ∈ 𝑅.
sin 𝑥+3
−1 ≤ sin 𝑥 ≤ 1
at

Misalkan 𝑡 = 3 + sin 𝑥 maka 2 ≤ 𝑡 ≤ 4


2 2
�sin 𝑥 + + 𝑎� = �𝑡 + + 𝑎 − 3�
at

sin 𝑥 + 3 𝑡
2 9 2 3
Dengan 3 ≤ 𝑡 + ≤ sehingga 0 ≤ 𝑡 + − 3 ≤
𝑡 2 𝑡 2
.c

3 3 3
Untuk 𝑎 ≥ − maka 𝑓(𝑎) = �𝑎 + � =𝑎+
4 2 2
3 3
w

Karena linier maka 𝑎 + minimum ketika 𝑎 = −


2 4
3
Untuk 𝑎 ≤ − maka 𝑓(𝑎) = |𝑎 + 0| = −𝑎
w

4
3
Karena linier maka −𝑎 minimum ketika 𝑎 = −
4
//w

𝟑
∴ Jadi, nilai minimum 𝑓(𝑎) adalah .
𝟒
s:
tp
ht

SMA Negeri 5 Bengkulu Eddy Hermanto, ST

63
Shared by: www.catatanmatematika.com

Tutur Widodo Soal OSK Matematika SMA 2016

Soal Olimpiade Sains Tingkat Kabupaten/Kota 2016


Bidang Matematika
Waktu: 120 menit

om
Petunjuk : Untuk masing-masing soal tulis jawab akhirnya saja (tanpa penjabaran)
di lembar jawab yang disediakan.

c
a.
1. Jika a, b, c, d, e merupakan bilangan asli dengan a < 2b, b < 3c, c < 4d, d < 5e dan e < 100,
maka nilai maksimum dari a adalah ...

ik
2. Rudi membuat bilangan asli dua digit. Probabilitas bahwa kedua digit bilangan tersebut

at
merupakan bilangan prima dan bilangan tersebut bersisa 3 jika dibagi 7 adalah ...

em
3. Pada segitiga ABC, titik M terletak pada BC sehingga AB = 7, AM = 3, BM = 5 dan
M C = 6. Panjang AC adalah ...
√ √
4. Diberikan a dan b bilangan real dengan
...
a− at b = 20. Nilai maksimum dari a − 5b adalah
m
5. Pada segitiga ABC, titik X, Y dan Z berturut-turut terletak pada sinar BA, CB dan AC
an

sehingga BX = 2BA, CY = 2CB dan AZ = 2AC. Jika luas 4ABC adalah 1, maka luas
4XY Z adalah ...
at

6. Banyaknya bilangan asli n yang memenuhi sifat hasil jumlah n dan suatu pembagi positif n
yang kurang dari n sama dengan 2016 adalah ...
at

7. Misalkan a adalah bilangan real sehingga polinomial p(x) = x4 + 4x + a habis dibagi oleh
.c

(x − c)2 untuk suatu bilangan real c. Nilai a yang memenuhi adalah ...
w

8. Anak laki-laki dan anak perempuan yang berjumlah 48 orang duduk melingkar secara acak.
w

Banyaknya minimum anak perempuan sehingga pasti ada enam anak perempuan yang duduk
berdekatan tanpa diselingi anak laki-laki adalah ...
//w

9. Misalkan (a, b, c, d, e, f ) adalah sebarang pengurutan dari (1, 2, 3, 4, 5, 6). Banyaknya pengu-
rutan sehingga a + c + e > b + d + f adalah ...
s:

10. Misalkan n1 , n2 , n3 , · · · bilangan-bilangan asli yang membentuk barisan aritmatika. Banyaknya


tp

nilai di himpunan {1, 2, 3, · · · , 1000} yang mungkin menjadi nilai nn2 − nn1 adalah ...
ht

11. Segitiga ABC mempunyai panjang sisi AB = 20, AC = 21 dan BC = 29. Titik D dan E
terletak pada segmen garis BC, dengan BD = 8 dan EC = 9. Besar ∠DAE adalah ...
derajat.

12. Bilangan real t sehingga terdapat dengan tunggal tripel bilangan real (x, y, z) yang memenuhi
x2 + 2y 2 = 3z dan x + y + z = t adalah ...

64
Shared by: www.catatanmatematika.com

Tutur Widodo Soal OSK Matematika SMA 2016

13. Palindrom adalah bilangan yang sama dibaca dari depan atau dari belakang. Sebagai contoh
12321 dan 32223 merupakan palindrom. Palindrom 5 digit terbesar yang habis dibagi 303
adalah ...
1 1
14. Diberikan barisan {an } dan {bn } dengan an = √ dan bn = q untuk
n n 1 + n + 1 + n1
1


om
setiap bilangan asli n. Misalkan Sn = a1 b1 + a2 b2 + · · · + an bn . Banyaknya bilangan asli n
dengan n ≤ 2016 sehingga Sn merupakan bilangan rasional adalah ...

c
15. Diberikan persegi ABCD dengan panjang sisi 1. Titik K dan L berturut-turut terletak

a.
pada segmen garis BC dan DC sehingga keliling dari 4KCL adalah 2. Luas minimum dari
4AKL adalah ...

ik
16. Banyaknya pasangan terurut bilangan asli (a, b, c) dengan a, b, c ∈ {1, 2, 3, 4, 5} sehingga

at
max{a, b, c} < 2 min{a, b, c}

em
adalah ...
at
17. Banyaknya bilangan asli n ∈ {1, 2, 3, · · · , 1000} sehingga terdapat bilangan real positif x yang
memenuhi x2 + bxc2 = n adalah ...
m
18. Misalkan x, y, z bilangan real positif yang memenuhi
an

3 logx (3y) = 3 log3x (27z) = log3x4 (81yz) 6= 0


at

Nilai dari x5 y 4 z adalah ...


at

19. Diberikan empat titik pada satu lingkaran Γ dalam urutan A, B, C, D. Sinar garis AB dan
DC berpotongan di E, dan sinar garis AD dan BC berpotongan di F . Misalkan EP dan
.c

F Q menyinggung lingkaran Γ berturut-turut di P dan Q. Misalkan pula bahwa EP = 60


w

dan F Q = 63, maka panjang EF adalah ...


w

20. Pada sebuah bidang datar, terdapat 16 garis berbeda dan n titik potong berbeda. Nilai
minimal n sehingga dapat dipastikan terdapat 3 kelompok garis yang masing-masing memuat
//w

garis-garis berbeda yang saling sejajar adalah ...


s:
tp

Ditulis ulang oleh : Tutur Widodo


ht

Apabila ada saran, kritik maupun masukan


silakan kirim via email ke
tutur.w87@gmail.com
Website :
http://www.tuturwidodo.com
http://www.pintarmatematika.net

65
Shared by: www.catatanmatematika.com

Tutur Widodo Solusi OSK Matematika SMA 2016

Solusi Olimpiade Sains Tingkat Kabupaten/Kota 2016


Bidang Matematika
1. Jika a, b, c, d, e merupakan bilangan asli dengan a < 2b, b < 3c, c < 4d, d < 5e dan e < 100,
maka nilai maksimum dari a adalah ...

om
Jawaban : 11847

c
e ≤ 99 =⇒ d < 495

a.
d ≤ 494 =⇒ c < 1976

ik
c ≤ 1975 =⇒ b < 5925
b ≤ 5924 =⇒ a < 11848

at
em
Jadi, nilai maksimum a adalah 11847.

2. Rudi membuat bilangan asli dua digit. Probabilitas bahwa kedua digit bilangan tersebut
merupakan bilangan prima dan bilangan tersebut bersisa 3 jika dibagi 7 adalah ...
Jawaban :
1
45
at
m
Misalkan bilangan yang dibuat Rudi adalah 10a + b. Diketahui bahwa
an

10a + b ≡ 3 mod 7 ⇔ 3a + b ≡ 3 mod 7

karena a, b ∈ {2, 3, 5, 7} maka tinggal dibagi kasus


at

• a = 2, diperoleh 6 + b ≡ 3 mod 7 ⇔ b ≡ 4 mod 7. Tidak ada nilai b yang memenuhi.


at

• a = 3, diperoleh 9 + b ≡ 3 mod 7 ⇔ b ≡ 1 mod 7. Tidak ada nilai b yang memenuhi.


.c

• a = 5, diperoleh 15 + b ≡ 3 mod 7 ⇔ b ≡ 2 mod 7. Diperoleh b = 2.


• a = 7, diperoleh 21 + b ≡ 3 mod 7 ⇔ b ≡ 3 mod 7. Diperoleh b = 3.
w

Jadi, ada dua bilangan yang memiliki sifat kedua digit penyusunnya berupa bilangan prima
w

dan bilangan tersebut bersisa 3 jika dibagi 7 yaitu 52 dan 73. Sehingga peluangnya adalah
2 1
//w

= .
90 45
3. Pada segitiga ABC, titik M terletak pada BC sehingga AB = 7, AM = 3, BM = 5 dan
s:

M C = 6. Panjang AC adalah ...



Jawaban : 3 3
tp

Dengan dalil Stewart diperoleh


ht

AB 2 × M C + AC 2 × BM = AM 2 × BC + BC × BM × M C
⇔ 49 × 6 + AC 2 × 5 = 9 × 11 + 11 × 5 × 6
⇔ 5AC 2 = 135

⇔ AC = 3 3

66
Shared by: www.catatanmatematika.com

Tutur Widodo Solusi OSK Matematika SMA 2016

√ √
4. Diberikan a dan b bilangan real dengan a− b = 20. Nilai maksimum dari a − 5b adalah
...
Jawaban : 500
√ √ √
a − b = 20 =⇒ a = b + 40 b + 400, sehingga
√ √

om
a − 5b = b + 40 b + 400 − 5b = −4( b − 5)2 + 500

Oleh karena itu, nilai maksimum dari a − 5b adalah 500, dicapai ketika a = 625 dan b = 25.

c
5. Pada segitiga ABC, titik X, Y dan Z berturut-turut terletak pada sinar BA, CB dan AC

a.
sehingga BX = 2BA, CY = 2CB dan AZ = 2AC. Jika luas 4ABC adalah 1, maka luas

ik
4XY Z adalah ...
Jawaban : 7

at
Perhatikan gambar berikut!

em
Z

A
at B
m
X Y
an

Kita punya
at

[ABC] = [ABY ] = [AXY ]


at

[ABC] = [BCZ] = [BZY ]


.c

[ABC] = [ACX] = [CZX]


w

Sehingga [XY Z] = 7[ABC] = 7.


w

6. Banyaknya bilangan asli n yang memenuhi sifat hasil jumlah n dan suatu pembagi positif n
yang kurang dari n sama dengan 2016 adalah ...
//w

Jawaban : 34
Misalkan a < n adalah faktor positif dari n sehingga a + n = 2016. Perhatikan bahwa a
membagi 2016. Sehingga a adalah faktor positif dari 2016. Karena 2016 = 25 × 32 × 7 maka
s:

faktor positif dari 2016 ada sebanyak 6 × 3 × 2 = 36. Dan karena n = 2016 − a ≥ 1 serta
tp

a < n maka a 6= 2016 dan a 6= 1008. Sehingga banyaknya bilangan asli n yang memenuhi
ada 36 − 2 = 34.
ht

7. Misalkan a adalah bilangan real sehingga polinomial p(x) = x4 + 4x + a habis dibagi oleh
(x − c)2 untuk suatu bilangan real c. Nilai a yang memenuhi adalah ...
Jawaban : a = 3
Jelas c 6= 0. Karena (x − c)2 faktor dari p(x) maka diperoleh
 a
x4 + 4x + a = (x2 − 2cx + c2 ) x2 + bx + 2
c

67
Shared by: www.catatanmatematika.com

Tutur Widodo Solusi OSK Matematika SMA 2016

dengan menjabarkan ruas kanan diperoleh


a  
4 4 3 2

2 2 2a
x + 4x + a = x + (b − 2c)x + − 2bc + c x + bc − x+a
c2 c

Oleh karena itu,

om
b − 2c = 0 =⇒ b = 2c
a
− 2bc + c2 = 0 =⇒ a = 3c4

c
c2

a.
2a
bc2 − = 4 =⇒ c3 = −1 =⇒ c = −1
c

ik
sehingga a = 3c4 = 3.

at
8. Anak laki-laki dan anak perempuan yang berjumlah 48 orang duduk melingkar secara acak.
Banyaknya minimum anak perempuan sehingga pasti ada enam anak perempuan yang duduk

em
berdekatan tanpa diselingi anak laki-laki adalah ...
Jawaban : 41
at
Misalkan n menyatakan jumlah anak laki-laki dan misalkan pula tempat duduk diantara dua
laki-laki yang berdekatan kita sebut sebagai ruang. Jika n ≥ 8 maka ada minimal 8 ruang
m
yang bisa ditempati oleh anak perempuan. Sementara itu, jumlah anak perempuan maksimal
ada 40. Jadi, kita dapat mengatur anak perempuan tersebut ke dalam ruang-ruang sehingga
an

tiap ruang maksimal ada 5 anak perempuan.


Jika n = 7 maka ada 7 ruang yang bisa ditempati oleh 41 anak perempuan. Berdasarkan
at

PHP pasti ada setidaknya satu ruang yang ditempati oleh setidaknya 6 anak perempuan.
at

Jadi, jumlah anak perempuan minimum ada 41.

9. Misalkan (a, b, c, d, e, f ) adalah sebarang pengurutan dari (1, 2, 3, 4, 5, 6). Banyaknya pengu-
.c

rutan sehingga a + c + e > b + d + f adalah ...


w

Jawaban : 360
Karena 1 + 2 + 3 + 4 + 5 + 6 = 21, dan a + c + e > b + d + f maka 6 ≤ b + d + f ≤ 10. WLOG
w

a < c < e dan b < d < f .


//w

• Jika b + d + f = 6 maka (b, d, f ) = (1, 2, 3) dan (a, c, e) = (4, 5, 6).


• Jika b + d + f = 7 maka (b, d, f ) = (1, 2, 4) dan (a, c, e) = (3, 5, 6).
s:

• Jika b + d + f = 8 maka
tp

– (b, d, f ) = (1, 2, 5) dan (a, c, e) = (3, 4, 6),


– (b, d, f ) = (1, 3, 4) dan (a, c, e) = (2, 5, 6)
ht

• Jika b + d + f = 9 maka
– (b, d, f ) = (1, 2, 6) dan (a, c, e) = (3, 4, 5),
– (b, d, f ) = (1, 3, 5) dan (a, c, e) = (2, 4, 6),
– (b, d, f ) = (2, 3, 4) dan (a, c, e) = (1, 5, 6)
• Jika b + d + f = 10 maka

68
Shared by: www.catatanmatematika.com

Tutur Widodo Solusi OSK Matematika SMA 2016

– (b, d, f ) = (1, 3, 6) dan (a, c, e) = (2, 4, 5),


– (b, d, f ) = (1, 4, 5) dan (a, c, e) = (2, 3, 6),
– (b, d, f ) = (2, 3, 5) dan (a, c, e) = (1, 4, 6)

Jadi, pasangan (a, b, c, d, e, f ) yang memenuhi ada sebanyak 10 × 3! × 3! = 360.

om
10. Misalkan n1 , n2 , n3 , · · · bilangan-bilangan asli yang membentuk barisan aritmatika. Banyaknya
nilai di himpunan {1, 2, 3, · · · , 1000} yang mungkin menjadi nilai nn2 − nn1 adalah ...

c
Jawaban : 31

a.
Misalkan n1 = a dan beda barisan aritmatika tersebut adalah b dengan a, b > 0.

ik
nn2 − nn1 = na+b − na = a + (a + b − 1)b − (a + (a − 1)b) = b2

at
karena 312 < 1000 < 322 maka banyaknya nilai yang mungkin dari nn2 − nn1 adalah 31.

em
11. Segitiga ABC mempunyai panjang sisi AB = 20, AC = 21 dan BC = 29. Titik D dan E
terletak pada segmen garis BC, dengan BD = 8 dan EC = 9. Besar ∠DAE adalah ...
derajat.
Jawaban : 45◦
at
Buat garis melalui B sejajar AC yang memotong perpanjangan AD di G. Demikian pula,
m
buat garis melalui C sejajar AB yang memotong perpanjangan AE di F , seperti gambar
an

berikut

C F
at
at

E
.c
w

G
D
w
//w

A B
s:

8
Dengan memanfaatkan kesebangunan antara 4BDG dan 4ADC diperoleh BG = × 21 =
21
tp

8. Dengan cara serupa diperoleh pula CF = 9. Misalkan ∠CAF = β dan ∠BAG = α. Maka
diperoleh
8 2 9 3
ht

tan α = = dan tan β = =


20 5 21 7

69
Shared by: www.catatanmatematika.com

Tutur Widodo Solusi OSK Matematika SMA 2016

sehingga didapat

tan ∠DAE = tan(90 − (α + β))


= cot(α + β))
1

om
=
tan(α + β))
1 − tan α tan β
=
tan α + tan β

c
1− 2 × 3
= 2 5 3 7

a.
5 + 7
35 − 6

ik
= =1
14 + 15

at
Jadi, ∠DAE = 45◦ .
Alternatif solusi (Kredit to Pak Eddy) : Perhatikan bahwa 4ABE dan 4ADC adalah

em
segitiga samakaki. Dengan mengingat bahwa ∠ABC + ∠BCA = 90◦ , diperoleh

∠DAE = ∠BAE + ∠CAD − 90◦ at


180◦ − ∠ABC 180◦ − ∠BCA
= + − 90◦
m
2 2
360◦ − ∠ABC − ∠BCA
= − 90◦
2
an

= 135◦ − 90◦
= 45◦
at
at

12. Bilangan real t sehingga terdapat dengan tunggal tripel bilangan real (x, y, z) yang memenuhi
x2 + 2y 2 = 3z dan x + y + z = t adalah ...
9
.c

Jawaban : t = −
8
w

3 2 3 2 27
   
2 2
3t = 3x + 3y + 3z = 3x + 3y + x + 2y = x + +2 y+ −
2 4 8
w

27 9
Agar memiliki penyelesaian tunggal maka haruslah 3t = − ⇔ t=−
//w

8 8
13. Palindrom adalah bilangan yang sama dibaca dari depan atau dari belakang. Sebagai contoh
12321 dan 32223 merupakan palindrom. Palindrom 5 digit terbesar yang habis dibagi 303
s:

adalah ...
tp

Jawaban : 47874
Misalkan palindrom lima digit tersebut adalah n = abcba = 10001a + 1010b + 100c. Karena
ht

habis dibagi 303 = 3 × 101 maka

n = 10001a + 1010b + 100c ≡ 2a − c ≡ 0 mod 101

dan
n = 10001a + 1010b + 100c ≡ 2a + 2b + c ≡ 0 mod 3

70
Shared by: www.catatanmatematika.com

Tutur Widodo Solusi OSK Matematika SMA 2016

karena 2a − c ≡ 0 mod 101 dan −9 ≤ 2a − c ≤ 18 maka 2a − c = 0 =⇒ c = 2a. Agar n


maksimal pilih a = 4. Akibatnya

2a + 2b + c ≡ 0 mod 3 ⇔ 16 + 2b ≡ 0 mod 3 ⇔ b ≡ 1 mod 3

om
maka nilai b terbesar adalah b = 7. Jadi, n = 47874.
1 1
14. Diberikan barisan {an } dan {bn } dengan an = √ dan bn = q untuk
n n 1+ n + 1+ 1
 1

c
n
setiap bilangan asli n. Misalkan Sn = a1 b1 + a2 b2 + · · · + an bn . Banyaknya bilangan asli n

a.
dengan n ≤ 2016 sehingga Sn merupakan bilangan rasional adalah ...

ik
Jawaban : 43
Perhatikan bahwa

at
1 1
ak bk = √ ×

em
q
k k 1 + k1 + 1 +
 1
k
1
=√ √
k(k + 1) + k k + 1
=p
1

at 1
k(k + 1) k + 1 + k

m
1 √ √
=p ( k + 1 − k)
k(k + 1)
an

1 1
=√ −√
k k+1
at

sehingga
at

     
1 1 1 1 1 1 1
Sn = √ −√ + √ −√ + ··· + √ −√ =1− √
1 2 2 3 n n+1 n+1
.c

Agar Sn bernilai rasional maka n + 1 harus berupa bilangan kuadrat. Mengingat 2 ≤ n + 1 ≤


w

2017, dan 442 < 2017 < 452 , maka nilai n yang mungkin ada sebanyak 43.
w

15. Diberikan persegi ABCD dengan panjang sisi 1. Titik K dan L berturut-turut terletak
//w

pada segmen garis BC dan DC sehingga keliling dari 4KCL adalah 2. Luas minimum dari
4AKL adalah ...

Jawaban : 2 − 1
s:

Perhatikan gambar berikut!


tp
ht

71
Shared by: www.catatanmatematika.com

Tutur Widodo Solusi OSK Matematika SMA 2016

D L C

om
K

c
A B

a.
ik
Misalkan CK = a dan CL = b dengan 0 < a, b < 1. Karena keliling 4KCL = 2 diperoleh

at
p p
a+b+ a2 + b2 = 2 ⇔ a + b + (a + b)2 − 2ab = 2

em
misalkan a + b = x dan ab = y dengan 0 < x < 2 dan 0 < y < 1 diperoleh
p
x+ x2 − 2y = 2 ⇔ x2 − 2y = 4 − 4x + x2 ⇔ y = 2x − 2
at √
Selain itu berdasarkan AM-GM diperoleh pula a + b ≥ 2 ab ⇔

x ≥ 2 y. Yang berakibat
m

x ≥ 2 2x − 2 ⇔ x2 − 8x + 8 ≥ 0
an

√ √
sehingga x ≤ 4 − 2 2 atau x ≥ 4 + 2 2. Akan tetapi, karena x < 2 maka diperoleh

at

x ≤ 4 − 2 2.
Di lain pihak
at

[AKL] = 1 − [ABK] − [KCL] − [ADL]


.c

1 1 1
= 1 − (1 − a) − ab − (1 − b)
w

2 2 2
1
= (a + b − ab)
w

2
1
= (x − y)
//w

2
1 1 √ √
= (2 − x) ≥ (2 − (4 − 2 2)) = 2 − 1
2 2
s:

√ √
Jadi, luas 4AKL minimal adalah 2 − 1 yang dicapai saat a = b = 2 − 2.
tp

16. Banyaknya pasangan terurut bilangan asli (a, b, c) dengan a, b, c ∈ {1, 2, 3, 4, 5} sehingga
ht

max{a, b, c} < 2 min{a, b, c}

adalah ...
Jawaban : 35
WLOG a ≤ b ≤ c, maka diperoleh c < 2a.

(a) Jika a = 1 maka c = 1 dan b = 1, maka diperoleh pasangan (1, 1, 1).

72
Shared by: www.catatanmatematika.com

Tutur Widodo Solusi OSK Matematika SMA 2016

(b) Jika a = 2 maka


• c = 2 dan b = 2, diperoleh pasangan (2, 2, 2)
• c = 3 dan b = 2, 3, diperoleh pasangan (2, 2, 3) dan (2, 3, 3) ada sebanyak 2 × 3 = 6
pasangan.

om
(c) Jika a = 3 maka
• c = 3 dan b = 3, diperoleh pasangan (3, 3, 3)
• c = 4 dan b = 3, 4, diperoleh pasangan (3, 3, 4) dan (3, 4, 4) ada sebanyak 2 × 3 = 6

c
pasangan.

a.
• c = 5 dan b = 3, 4, 5, diperoleh pasangan (3, 3, 5), (3, 4, 5) dan (3, 5, 5) ada sebanyak
3 + 6 + 3 = 12 pasangan.

ik
(d) Jika a = 4 maka

at
• c = 4 dan b = 4, diperoleh pasangan (4, 4, 4)

em
• c = 5 dan b = 4, 5, diperoleh pasangan (4, 4, 5) dan (4, 5, 5) ada sebanyak 2 × 3 = 6
pasangan.
(e) Jika a = 5 maka c = 5 dan b = 5, maka diperoleh pasangan (5, 5, 5).

Jadi, total ada 1 + 7 + 19 + 7 + 1 = 35 pasangan.


at
m
17. Banyaknya bilangan asli n ∈ {1, 2, 3, · · · , 1000} sehingga terdapat bilangan real positif x yang
memenuhi x2 + bxc2 = n adalah ...
an

Jawaban : 516
Perhatikan bahwa x2 = n − bxc2 sehingga x2 adalah bilangan bulat positif. Oleh karena itu,
at


x = a untuk suatu bilangan a bulat positif. Misalkan a = k 2 + m dengan 0 ≤ m ≤ 2k,
maka diperoleh
at

n = k 2 + m + k 2 = 2k 2 + m
.c

Untuk k = 1, 2, 3, · · · , 21 maka nilai n yang mungkin ada sebanyak


w

21
X
(2k + 1) = 483
w

k=1
//w

Sedangkan untuk k = 22 perlu diperhatikan bahwa nilai m yang mungkin hanya m =


0, 1, 2, 3, · · · , 32. Jadi ada 33 nilai n yang mungkin.
s:

Untuk k ≥ 23 akan berakibat n > 1000.


Jadi, total banyaknya kemungkinan nilai n adalah 483 + 33 = 516.
tp

18. Misalkan x, y, z bilangan real positif yang memenuhi


ht

3 logx (3y) = 3 log3x (27z) = log3x4 (81yz) 6= 0

Nilai dari x5 y 4 z adalah ...


1
Jawaban : 8
3
Misalkan
3 logx (3y) = 3 log3x (27z) = log3x4 (81yz) = k

73
Shared by: www.catatanmatematika.com

Tutur Widodo Solusi OSK Matematika SMA 2016

Berdasarkan definisi fungsi logaritma diperoleh

k k
logx (3y) = =⇒ 3y = x 3 =⇒ xk = 33 y 3 ............(1)
3
k k
log3x (27z) = =⇒ 27z = (3x) 3 =⇒ (3x)k = 39 z 3 ............(2)
3

om
log3x4 (81yz) = k =⇒ 81yz = (3x4 )k ...........(3)

Sehingga diperoleh

c
a.
xk · 3k · xk 33 y 3 · 39 z 3 1 1
= ⇔ = 38 y 2 z 2 ⇔ xk =
3k · x4k 34 yz x2k 34 yz

ik
Dari pers.(1) diperoleh

at
1 1
= 27y 3 =⇒ y 4 z = 7
34 yz 3

em
Dari pers.(3) diperoleh
 k
1
4 k 1 1
(3x ) = =⇒ 3x4 = =⇒ x5 =

1
x x at3
m
Jadi, x5 y 4 z =
38
an

19. Diberikan empat titik pada satu lingkaran Γ dalam urutan A, B, C, D. Sinar garis AB dan
DC berpotongan di E, dan sinar garis AD dan BC berpotongan di F . Misalkan EP dan
F Q menyinggung lingkaran Γ berturut-turut di P dan Q. Misalkan pula bahwa EP = 60
at

dan F Q = 63, maka panjang EF adalah ...


at

Jawaban : 87
Misalkan ER adalah garis singgung (lain) yang ditarik dari titik E. Misalkan O adalah pusat
.c

lingkaran Γ, dan G perpotongan antara EO dan P R, seperti terlihat pada gambar berikut
w

A
w

Q
//w

P O
D
s:

C
tp

B R
ht

Jelas bahwa P G = GR. Perhatikan pula bahwa P, R, F segaris. Hal ini karena P R adalah
polar dari E, sementara itu F juga terletak pada polar E.

74
Shared by: www.catatanmatematika.com

Tutur Widodo Solusi OSK Matematika SMA 2016

Selanjutnya dengan dalil phytagoras pada 4EP G dan 4EF G diperoleh

EP 2 − P G2 = EF 2 − F G2
⇔ EP 2 − P G2 = EF 2 − (GR + RF )2
⇔ EP 2 − P G2 = EF 2 − (P G + RF )2

om
⇔ EP 2 − P G2 = EF 2 − P G2 − 2 × P G × RF − RF 2
⇔ EP 2 = EF 2 − RF (2 × P G + RF )

c
⇔ EP 2 = EF 2 − RF × P F )

a.
⇔ EP 2 = EF 2 − F Q2

ik
p
⇔ EF = 602 + 632 = 87

at
Alternatif solusi (Kredit to Pak Eddy) : Misalkan lingkaran luar 4EBC memotong (lagi)

em
EF di M .

at
Q
m
P
D
an

C
at

B
at

F
.c

M
E
w
w

Berdasarkan teorema Miquel, maka F DCM adalah segiempat talibusur. Oleh karena itu,
//w

dengan power of the point diperoleh

F M × F E = F C × F B = F Q2
s:

dan
tp

EM × EF = EC × ED = EP 2
ht

75
Shared by: www.catatanmatematika.com

Tutur Widodo Solusi OSK Matematika SMA 2016

Dengan menjumlahkam kedua persamaan di atas didapatkan

F M × F E + EM × EF = F Q2 + EP 2
EF (F M + M E) = F Q2 + EP 2
EF 2 = F Q2 + EP 2

om
p
EF = F Q2 + EP 2
p
EF = 632 + 602 = 87

c
a.
20. Pada sebuah bidang datar, terdapat 16 garis berbeda dan n titik potong berbeda. Nilai
minimal n sehingga dapat dipastikan terdapat 3 kelompok garis yang masing-masing memuat

ik
garis-garis berbeda yang saling sejajar adalah ...

at
em
at Disusun oleh : Tutur Widodo
Apabila ada saran, kritik maupun masukan
m
silakan kirim via email ke
tutur.w87@gmail.com
an

Website :
http://www.tuturwidodo.com
at

http://www.pintarmatematika.net
at
.c
w
w
//w
s:
tp
ht

76
Shared by: www.catatanmatematika.com

PEMBAHASAN SOAL
OLIMPIADE SAINS MATEMATIKA SMA
TINGKAT KABUPATEN
14 MARET 2017

m
By Pak Anang (http://pak-anang.blogspot.com)

co
1. Diketahui dan . Nilai dari adalah ….

a.
Pembahasan:
Perhatikan untuk mencari bentuk jalur yang kita tempuh adalah mencari terlebih dahulu

ik
bentuk .

at
Perhatikan karena , maka , sehingga

em
Padahal karena , maka
at , sehingga
a nm
at
at
.c
w
w
//w
s:
tp
ht

Pembahasan OSK Matematika SMA 2017 by Pak Anang (http://pak-anang.blogspot.com)

77
Shared by: www.catatanmatematika.com

2. Empat siswa Adi, Budi, Cokro, dan Dion bertanding balap sepeda. Kita hanya diberikan sebagian
informasi sebagai berikut:
(a) setiap siswa sampai di garis finish pada waktu yang berlainan
(b) Adi bukan juara pertama
(c) Cokro kalah dari Budi
Dengan hanya mengetahui informasi ini saja, banyaknya susunan juara pertama, kedua, ketiga,
dan keempat adalah ….

m
Pembahasan:

co
Perhatikan informasi yang diberikan pada soal adalah sebagai berikut:
(a) setiap siswa sampai di garis finish pada waktu yang belainan, artinya setiap posisi juara
ditempati oleh satu orang saja.

a.
(b) Adi bukan juara pertama, artinya juara pertama tidak mungkin ditempati oleh Adi. Adi hanya
bisa menempati posisi pada juara kedua, ketiga, atau keempat.

ik
(c) Cokro kalah dari Budi, artinya jika Cokro menjadi juara pertama, maka kemungkinan Budi
adalah juara kedua, ketiga, atau keempat. Sedangkan jika Cokro juara kedua, maka

at
kemungkinan Budi hanya menjadi juara ketiga atau keempat saja. Sedang jika Cokro juara
ketiga, maka Budi pastilah menjadi juara keempat. Syarat ini tidak memungkinkan untuk
Cokro menjadi juara keempat.

em
Sehingga, dari informasi tersebut, kita misalkan posisi masing-masing juara sebagai berikut:
1. Posisi Adi.

2. Posisi Cokro dan Budi


at
Kemungkinan posisi Adi adalah memilih satu tempat dari 3, sehingga .
nm
Kemungkinan posisi Cokro dan Budi adalah memilih dua tempat dari 3 tempat secara
kombinasi, karena posisinya sudah pasti Cokro kalah dari Budi, sehingga .
3. Posisi Dion.
Posisi Dion sudah tidak perlu ditentukan karena hanya tersisa satu tempat lagi, sehingga .
a

Jadi, banyaknya cara menentukan susunan juara pertama, kedua, ketiga, dan keempat adalah:
at
at

Cara Alternatif:
Manual dengan meletakkan masing-masing orang sesuai dengan informasi pada soal.
.c

Sehingga diperoleh 9 kemungkinan, yaitu:


w

No Juara 1 Juara 2 Juara 3 Juara 4


1. Cokro Adi Budi Dion
w

2. Cokro Adi Dion Budi


3. Dion Adi Cokro Budi
//w

4. Cokro Budi Adi Dion


5. Cokro Dion Adi Budi
6. Dion Cokro Adi Budi
s:

7. Cokro Budi Dion Adi


8. Cokro Dion Budi Adi
tp

9. Dion Cokro Budi Adi


ht

Pembahasan OSK Matematika SMA 2017 by Pak Anang (http://pak-anang.blogspot.com)

78
Shared by: www.catatanmatematika.com

3. Banyaknya bilangan asli yang memenuhi untuk semua bilangan asli adalah ….

Pembahasan:
Perhatikan , maksudnya adalah faktor dari dimana adalah bilangan asli.

Pemfaktoran dari bentuk , diperoleh:

m
co
a.
Mudah diperiksa bahwa bentuk memuat bentuk perkalian dari 3 bilangan asli berurutan,

ik
dimana 3 bilangan asli berurutan pasti habis dibagi 6.
Sehingga , untuk semua bilangan asli pasti juga habis dibagi oleh 6.

at
Namun, setelah diperiksa lebih lanjut, ternyata 7 juga menjadi faktor dari .

em
Misal , maka dan .
Kedua nilai dan adalah juga kelipatan 7 karena .
at
Misal adalah kelipatan 7, maka akan dibuktikan
nm
adalah juga kelipatan 7.

Perhatikan,
a
at
at
.c
w
w
//w
s:

Jadi jelas bahwa 7 adalah salah satu faktor dari .


tp

Jadi, karena faktor adalah 1, 2, 3, 6, 7, 14, 21, dan 42, maka terdapat 8 buah bilangan
asli yang merupakan faktor dari .
ht

Atau menggunakan rumus banyak faktor bulat positif, maka karena , sehingga
banyak faktor bulat positif dari 42 adalah buah.

Pembahasan OSK Matematika SMA 2017 by Pak Anang (http://pak-anang.blogspot.com)

79
Shared by: www.catatanmatematika.com

4. Pada sebuah lingkaran dengan pusat , talibusur berjarak 5 dari titik dan talibusur
berjarak dari titik . Jika panjang jari-jari lingkaran 10, maka adalah ….

Pembahasan:
Ilustrasi soal terlihat pada gambar berikut:

m
co
a.
ik
at
em
(i) at (ii)

Ternyata tali busur AC ada dua buah yang sesuai kriteria pada soal, yaitu berjarak dari titik .
nm
Sehingga, titik kita beri indeks masing-masing untuk membedakannya, yaitu dan .

Kasus 1.
a

Pada gambar (i), titik kita pilih berada “di atas” B.


at

Perhatikan , misal , maka:


at

sin sin
.c

sin
w

Perhatikan , misal , maka:


w

sin sin
//w

sin
s:

Padahal, sudut keliling .


tp

Sehingga sudut pusat .

Jadi, dengan menggunakan aturan cosinus, diperoleh:


ht

Pembahasan OSK Matematika SMA 2017 by Pak Anang (http://pak-anang.blogspot.com)

80
Shared by: www.catatanmatematika.com

Kasus 2.
Pada gambar (ii), titik kita pilih berada “di bawah” B.

Perhatikan , misal , maka:

sin sin

m
sin

co
Perhatikan , misal , maka:
sin sin

a.
sin

ik
at
Padahal, sudut keliling .

em
Sehingga sudut pusat .

Jadi, dengan menggunakan aturan cosinus, diperoleh:


s at
a nm

Komentar terhadap soal:


at

Menurut pandangan saya, soal ini multitafsir dan cenderung ambigu, karena memiliki dua
penyelesaian….. Menurut kabar yang beredar kun inya adalah , yang artinya
at

jawabannya tunggal.
.c

Padahal, dari hasil pengamatan pada soal dan ilustrasi soal pada gambar, dapat ditemukan dua
nilai panjang yang dimaksud pada s al….
w

Jadi sebaiknya soal perlu diberi keterangan lebih lanjut agar jawaban benar tunggal dan tidak bias
w

atau ambigu…..
//w
s:
tp
ht

Pembahasan OSK Matematika SMA 2017 by Pak Anang (http://pak-anang.blogspot.com)

81
Shared by: www.catatanmatematika.com

5. Jika , maka nilai dari adalah ….

Pembahasan:
Misal,

m
Maka,

co
Sehingga diperoleh,

a.
Apabila kedua persamaan dikuadratkan, maka

ik
dan,

at
em
Jadi, diperoleh,

at
a nm
at
at
.c
w

Cara Alternatif:
w

Perhatikan,
//w
s:

tambah kedua ruas dengan


tp
ht

Pembahasan OSK Matematika SMA 2017 by Pak Anang (http://pak-anang.blogspot.com)

82
Shared by: www.catatanmatematika.com

6. Pada suatu kotak ada sekumpulan bola berwarna merah dan hitam yang secara keseluruhannya
kurang dari 1000 bola. Misalkan diambil dua bola. Peluang terambilnya dua bola merah adalah
dan peluang terambilnya dua bola hitam adalah dengan . Selisih terbesar yang
mungkin dari banyaknya b la merah dan hitam adalah ….

Pembahasan:
Misal,

m
banyaknya bola merah
banyaknya bola hitam

co
Sehingga, apabila banyak bola merah dan bola hitam secara keseluruhan dimisalkan dan banyak
keseluruhan kurang dari 1000, maka

a.
ik
Peluang terambil dua bola merah dan dua bola hitam adalah

at
em
Perhatikan, pada soal diketahui , sehingga:

at
a nm
at
at
.c
w
w
//w

Padahal, , sehingga:
s:
tp
ht

Jadi, nilai terbesar adalah ,


Sedangkan karena , jadi jumlah terbesar , maka diperoleh .

Sehingga selisih terbesar yang mungkin dari banyaknya bola merah dan hitam adalah

Pembahasan OSK Matematika SMA 2017 by Pak Anang (http://pak-anang.blogspot.com)

83
Shared by: www.catatanmatematika.com

Cara Alternatif:
Misal,
banyaknya bola merah
banyaknya bola hitam

Peluang terambil dua bola merah dan dua bola hitam adalah

m
co
Perhatikan, pada soal diketahui , sehingga:

a.
ik
at
em
at
a nm

Padahal, banyak keseluruhan bola adalah kurang dari 1000, maka


at

Sehingga, dengan sedikit manipulasi dari dapat diperoleh,


at
.c
w
w
//w

Jadi, diperoleh sedangkan selisih terbesar yang mungkin dari banyaknya


bola merah dan hitam adalah .
s:
tp
ht

Pembahasan OSK Matematika SMA 2017 by Pak Anang (http://pak-anang.blogspot.com)

84
Shared by: www.catatanmatematika.com

7. Misalkan menyatakan faktor prima terbesar dari dan menyatakan faktor prima
terkecil dari . Banyaknya bilangan asli … sehingga adalah ….

Pembahasan:
Karena , maka dua bilangan prima yang selisihnya 1 adalah
bilangan 2 dan 3.
Jadi jelas bahwa dan

m
Sehingga, bilangan yang dimaksud adalah , dengan dan adalah bilangan asli.

co
Padahal bilangan … , maka
- untuk , diperoleh tiga buah nilai yang mungkin adalah
- untuk , diperoleh dua buah nilai yang mungkin adalah

a.
- untuk , diperoleh dua nilai yang mungkin adalah
- untuk , diperoleh satu nilai yang mungkin adalah

ik
- untuk , diperoleh satu nilai yang mungkin adalah

at
Jadi, ada buah bilangan asli yang memenuhi.

Cara Alternatif:

em
Dicoba dengan cara manual, karena , maka dua bilangan prima
yang selisihnya 1 adalah bilangan 2 dan 3.
Jadi jelas bahwa dan

Sehingga, bilangan yang dimaksud adalah


at
, dengan dan adalah bilangan asli.
nm
Jadi, bilangan tersebut adalah:
-
-
a

-
-
at

-
-
at

-
-
.c

-
w

Jadi, ada buah bilangan asli yang memenuhi.


w

TRIK SUPERKILAT:
Perhatikan jelas bahwa , dengan bilangan asli.
//w

Maka adalah bilangan kelipatan 2 dan 3 yang tidak habis dibagi bilangan prima , dimana
, jadi .
Sehingga misal banyaknya adalah dapat dirumuskan sebagai:
s:
tp

Jadi, ada buah bilangan asli yang memenuhi.


ht

Pembahasan OSK Matematika SMA 2017 by Pak Anang (http://pak-anang.blogspot.com)

85
Shared by: www.catatanmatematika.com

8. Semua titik sudut suatu persegi dengan panjang sisi terletak pada batas dari juring lingkaran
berjari-jari yang sudut pusatnya . Jika persegi diletakkan secara simetris dalam juring, maka
nilai adalah ….

m
co
Pembahasan:

a.
Perhatikan gambar disamping,

ik
Misal,
jari-jari lingkaran

at
sisi persegi

em
Perhatikan ,
tan
tan
at
a nm
at
at

Sehingga diperoleh koordinat .


Dan karena , maka koordinat .
.c
w

Pandang persamaan lingkaran dengan pusat yang melalui adalah


, maka
w
//w
s:
tp
ht

Pembahasan OSK Matematika SMA 2017 by Pak Anang (http://pak-anang.blogspot.com)

86
Shared by: www.catatanmatematika.com

TRIK SUPERKILAT:
Perhatikan gambar disamping,

Jika persegi berada di dalam juring dengan sudut pusat ,


maka adalah segitiga sama sisi.
Sehingga, .

m
Perhatikan, ,

co
Pada berlaku aturan kosinus sebagai berikut:
s

a.
ik
at
em
at
a nm
at
at
.c
w
w
//w
s:
tp
ht

Pembahasan OSK Matematika SMA 2017 by Pak Anang (http://pak-anang.blogspot.com)

87
Shared by: www.catatanmatematika.com

9. Misalkan bilangan real positif yang memenuhi . Nilai minimum dari adalah
….

Pembahasan:
Misal,

m
Berdasarkan ketaksamaan diperoleh:

co
a.
Tanda kesamaan terjadi saat , sehingga dan

ik
Padahal , sehingga

at
Berdasarkan ketaksamaan diperoleh:

em
Tanda kesamaan terjadi saat , sehingga dan at
nm
Sehingga, nilai minimum dari adalah

min
a
at
at
.c
w
w
//w
s:
tp
ht

Pembahasan OSK Matematika SMA 2017 by Pak Anang (http://pak-anang.blogspot.com)

88
Shared by: www.catatanmatematika.com

10. Sebuah hotel mempunyai kamar bernomor 000 sampai dengan 999. Hotel tersebut menerapkan
aturan aneh sebagai berikut: jika suatu kamar berisi tamu, dan sembarang dua digit nomor kamar
tersebut dipertukarkan tempatnya, maka diperoleh nomor kamar yang sama atau nomor kamar
yang tidak berisi tamu. Maksimal banyak kamar yang berisi tamu adalah ….

Pembahasan:
Perhatikan aturan aneh pada hotel tersebut: “Jika suatu kamar berisi tamu, dan sembarang dua

m
digit nomor kamar tersebut dipertukarkan tempatnya”, maka diperoleh:
 nomor kamar yang sama.

co
Jika nomor kamarnya sama, maka sudah pasti nomor kamar tersebut berisi tamu.
 nomor kamar yang tidak berisi tamu.
Artinya, permutasi dari digit kamar menghasilkan nomor kamar yang tidak berisi tamu.

a.
Padahal, kamar hotel bernomor 000 sampai dengan 999. Artinya ada 1000 buah kamar hotel.

ik
Kamar hotel tersebut dapat diklasifikasikan berdasarkan jenis digitnya, yaitu:
 Nomor kamar yang ketiga digitnya sama.

at
Maka nomor kamar tersebut adalah .
Karena … , maka ada sebanyak 10 buah kamar yang tiga digitnya sama.

em
Karena apabila dipertukarkan dua digitnya mendapat nomor kamar yang sama, maka ada
10 buah kamar yang berisi tamu.

 Nomor kamar dengan dua digit yang sama.


Maka nomor kamar tersebut adalah
Karena … dan misal dipilih
at .
, maka … , maka ada
nm
sebanyak buah kamar.

Perhatikan tabel berikut:


Tamu
a

Jenis nomor kamar Ditukar


Ada Tidak
2 digitnya
at


at

Sehingga ada kamar yang berisi tamu.


.c

 Nomor kamar yang ketiga digitnya berbeda.


w

Maka nomor kamar tersebut adalah .


Karena … dan misal dipilih , maka … dan misal dipilih
w

, maka … , maka ada sebanyak buah kamar


//w

Perhatikan tabel berikut:


Tamu Tamu Tamu
Jenis nomor kamar Ditukar
Ada Tidak
Ditukar
Ada Tidak
Ditukar
Ada Tidak
2 digitnya 2 digitnya 2 digitnya
s:


tp


ht

Berarti, ada tiga kamar yang terisi tamu, yaitu .


Sehingga ada kamar yang berisi tamu.

Jadi, maksimal banyak kamar yang berisi tamu adalah buah kamar.

Pembahasan OSK Matematika SMA 2017 by Pak Anang (http://pak-anang.blogspot.com)

89
Shared by: www.catatanmatematika.com

11. Fungsi memetakan himpunan bilangan bulat tak negatif. Fungsi tersebut memenuhi
dan untuk setiap bilangan asli berbeda dengan , berlaku . Jika diketahui
, maka nilai dari adalah ….

Pembahasan:
Perhatikan bahwa untuk setiap bilangan asli berbeda dengan , maka berlaku
.

m
Perhatikan, pandang bentuk .

co
Jika untuk setiap bilangan asli berbeda dengan , maka diperoleh
… … , dimana:

a.
ik

at
em

Sehingga apabila
at
diurutkan dari kecil ke besar diperoleh
nm

Dan apabila bilangan asli dimana , maka .


Jadi, akan berlaku
a
at

Nah, sekarang perhatikan bentuk faktorisasi prima dari 2016 adalah


Serta pandang bentuk faktorisasi prima dari adalah
at

Juga pada soal diketahui dan


.c

Untuk , maka diperoleh salah satu alternatif susunan , yaitu:


w
w
//w
s:
tp
ht

Dan karena maka berlaku .


Perhatikan juga karena dan , artinya .

Dari alternatif susunan di atas maka dengan mudah dapat dilihat .

Pembahasan OSK Matematika SMA 2017 by Pak Anang (http://pak-anang.blogspot.com)

90
Shared by: www.catatanmatematika.com

12. Diberikan segitiga dengan . Misalkan dan berturut-turut titik tengah


dan . Titik terletak pada sisi sehingga adalah garis bagi sudut . Jika
, maka besarnya sudut sama dengan ….

Pembahasan:
Perhatikan ilustrasi segitiga pada gambar di samping!

m
Misal,

co
Karena garis membagi sudut sama besar, sehingga

a.
ik
Padahal

at
Sehingga,

em
at
nm

Padalah dan merupakan titik tengah berturut-turut sisi dan , sehingga


a
at
at

Sehingga, karena , maka adalah segitiga sama kaki.


Misal
Diperoleh
.c

Begitu pula karena , maka adalah segitiga sama kaki.


w

Misal
Diperoleh
w
//w

Perhatikan , berlaku
s:
tp
ht

Jadi, dapat ditemukan dengan memandang bahwa adalah suatu garis lurus.

Pembahasan OSK Matematika SMA 2017 by Pak Anang (http://pak-anang.blogspot.com)

91
Shared by: www.catatanmatematika.com

Komentar terhadap soal:


Perhatikan, aturan kosinus pada :
s

m
s

co
s

a.
s

ik
Mengingat, dari diperoleh , maka

at
s
s

em
s
s

Sehingga,
at
nm
s

Padahal .
Sehingga, untuk soal di atas akan menjadi benar dan nyata apabila memenuhi
a

.
at

Faktanya, pada soal , yaitu .


at

Kesimpulannya soal tersebut memang dapat dikerjakan secara benar dengan konsep dan
mendapatkan hasil yang seolah-olah benar. Namun, apabila dicermati lebih lanjut maka bentuk
.c

segitiganya tidak dapat digambarkan….


w
w
//w
s:
tp
ht

Pembahasan OSK Matematika SMA 2017 by Pak Anang (http://pak-anang.blogspot.com)

92
Shared by: www.catatanmatematika.com

13. Misalkan suatu polinom berderajat 4 yang memiliki nilai maksimum 2018 di dan
. Jika , maka nilai adalah ….

Pembahasan:
Perhatikan, nilai maksimum di dan , artinya dan .
Karena suatu polinom berderajat 4, maka adalah suatu polinom berderajat 3 yang
memuat faktor dan , serta satu faktor yang lain, misal .

m
Jadi,

co
dapat ditentukan dengan menggunakan anti-turunan dari , sehingga

a.
ik
at
em
Padahal memiliki nilai maksimum 2018 di , artinya , maka

at
nm
Sehingga, karena maka diperoleh

juga memiliki nilai maksimum 2018 di , artinya , maka


a
at
at
.c
w
w

Sehingga, diperoleh
//w

Maka, nilai dapat ditentukan menggunakan


s:
tp
ht

Sehingga, diperoleh
Jadi,

Pembahasan OSK Matematika SMA 2017 by Pak Anang (http://pak-anang.blogspot.com)

93
Shared by: www.catatanmatematika.com

Cara Alternatif:
Misal,
maka,

Karena , maka .
Karena , maka ………

m
Karena maka
Karena , maka …….

co
Karena , maka …….

Sehingga, dari eliminasi pada persamaan dan , diperoleh

a.
…….

ik
Dari eliminasi pada persamaan dan diperoleh
…….

at
Dari eliminasi pada persamaan dan diperoleh
, sehingga

em
Substitusi dan ke persamaan diperoleh
at
nm
Sehingga, karena , , , , dan , diperoleh:
a

Jadi,
at
at

TRIK SUPERKILAT:
.c

Karena grafik dari fungsi pangkat 4 simetris, maka apabila diperhatikan, ambil dua titik puncak
dan , maka fungsi tersebut adalah , sehingga karena nilai
w

dan , maka fungsi tersebut adalah .


w

Uji titik , maka diperoleh:


//w

Sehingga fungsi tersebut adalah .

Jadi, .
s:
tp
ht

Pembahasan OSK Matematika SMA 2017 by Pak Anang (http://pak-anang.blogspot.com)

94
Shared by: www.catatanmatematika.com

14. Terdapat 6 anak, A, B, C, D, E, dan F, akan saling bertukar kado. Tidak ada yang menerima kadonya
sendiri, dan kado dari A diberikan kepada B. Banyaknya cara membagikan kado dengan cara
demikian adalah ….

Pembahasan:
Perhatikan, mula-mula setiap anak membawa kadonya sendiri-sendiri

m
A B C D E F

co
a.
Kado dari A pasti diberikan kepada B, sehingga ilustrasinya sebagai berikut

ik
A B C D E F

at
em
at
nm

Sehingga, sekarang B harus memberikan kadonya ke salah satu dari 5 anak yang lain.
a

Disini ada dua pilihan, yaitu


- B memberikan kado ke A
at

- B memberikan kado ke selain A


at
.c
w
w
//w
s:
tp
ht

Pembahasan OSK Matematika SMA 2017 by Pak Anang (http://pak-anang.blogspot.com)

95
Shared by: www.catatanmatematika.com

Perhatikan ilustrasi berikut:

Kasus 1.1
Misal B memberikan kadonya ke A, maka ilustrasinya sebagai berikut

A B C D E F

m
co
a.
ik
Artinya kita akan mengacak 4 anak agar tidak mendapat kadonya sendiri.

at
Perhatikan,

em
- banyak keseluruhan permutasi yang mungkin adalah .
- apabila ada 1 orang yang dibiarkan tetap, maka ada cara memilih orang dan permutasi
yang mungkin adalah sebanyak , sehingga banyak kemungkinan adalah .
- apabila ada 2 orang yang dibiarkan tetap, maka ada
at cara memilih orang dan permutasi
yang mungkin adalah sebanyak , sehingga banyak kemungkinan adalah .
- apabila ada 3 orang yang dibiarkan tetap, maka ada cara memilih orang dan permutasi
nm
yang mungkin adalah sebanyak , sehingga banyak kemungkinan adalah .
- apabila ada 4 orang yang dibiarkan tetap, maka ada cara memilih orang dan permutasi
yang mungkin adalah sebanyak , sehingga banyak kemungkinan adalah
a

Jadi, menggunakan prinsip inklusi-eksklusi himpunan diperoleh banyak kemungkinannya adalah:


at

.
at

ara

Kasus 1.2
.c

Misal B memberikan kadonya ke selain A, maka ilustrasinya sebagai berikut


w

A B C D E F
w
//w
s:
tp

Artinya, kita akan memilih kemungkinan B memberikan kado ke C, D, E atau F?


ht

Jadi, . ara

Pembahasan OSK Matematika SMA 2017 by Pak Anang (http://pak-anang.blogspot.com)

96
Shared by: www.catatanmatematika.com

Setelah B memberikan kado ke salah satu dari 4 orang tersebut, misalkan kado B diberikan ke C.

Sehingga ilustrasinya menjadi berikut

A B C D E F

m
co
a.
ik
Maka disini akan muncul dua kasus lagi.
Disini ada dua pilihan, yaitu

at
- C memberikan kado ke A
- C memberikan kado ke selain A

em
Kasus 2.1
Misal C memberikan kadonya ke A, maka ilustrasinya sebagai berikut

A B C D
at E F
a nm
at
at

Artinya, kita akan mengacak 3 anak agar tidak mendapat kadonya sendiri.
.c

Perhatikan,
w

- banyak keseluruhan permutasi yang mungkin adalah .


- apabila ada 1 orang yang dibiarkan tetap, maka ada cara memilih orang dan permutasi
w

yang mungkin adalah sebanyak , sehingga banyak kemungkinan adalah .


- apabila ada 2 orang yang dibiarkan tetap, maka ada cara memilih orang dan permutasi
//w

yang mungkin adalah sebanyak , sehingga banyak kemungkinan adalah .


- apabila ada 3 orang yang dibiarkan tetap, maka ada cara memilih orang dan permutasi
yang mungkin adalah sebanyak , sehingga banyak kemungkinan adalah .
s:

Jadi, menggunakan prinsip inklusi-eksklusi himpunan diperoleh banyak kemungkinannya adalah:


tp

.
ht

ara

Pembahasan OSK Matematika SMA 2017 by Pak Anang (http://pak-anang.blogspot.com)

97
Shared by: www.catatanmatematika.com

Kasus 2.2
Misal C memberikan kadonya ke selain A, maka ilustrasinya sebagai berikut

A B C D E F

m
co
a.
Artinya, kita akan memilih kemungkinan C memberikan kado ke D, E atau F?

ik
Jadi, . ara

at
em
Setelah C memberikan kado ke salah satu dari 3 orang tersebut, misalkan kado C diberikan ke D.

Sehingga ilustrasinya menjadi berikut at


A B C D E F
a nm
at
at
.c

Maka disini akan muncul dua kasus lagi.


Disini ada dua pilihan, yaitu
w

- D memberikan kado ke A
- D memberikan kado ke selain A
w

Kasus 3.1
//w

Misal D memberikan kadonya ke A, maka ilustrasinya sebagai berikut

A B C D E F
s:
tp
ht

Artinya, kita akan mengacak 2 anak agar tidak mendapat kadonya sendiri.

Pembahasan OSK Matematika SMA 2017 by Pak Anang (http://pak-anang.blogspot.com)

98
Shared by: www.catatanmatematika.com

Perhatikan,
- banyak keseluruhan permutasi yang mungkin adalah .
- apabila ada 1 orang yang dibiarkan tetap, maka ada cara memilih orang dan permutasi
yang mungkin adalah sebanyak , sehingga banyak kemungkinan adalah .
- apabila ada 2 orang yang dibiarkan tetap, maka ada cara memilih orang dan permutasi
yang mungkin adalah sebanyak , sehingga banyak kemungkinan adalah .

m
Jadi, menggunakan prinsip inklusi-eksklusi himpunan diperoleh banyak kemungkinannya adalah:
.

co
ara

a.
Kasus 3.2
Misal D memberikan kadonya ke selain A, maka ilustrasinya sebagai berikut

ik
A B C D E F

at
em
at
nm
Artinya, kita akan memilih kemungkinan D memberikan kado ke E atau F?

Jadi, . ara
a
at
at

Setelah D memberikan kado ke salah satu dari 2 orang tersebut, misalkan kado D diberikan ke E.
.c

Sehingga ilustrasinya menjadi berikut


w

A B C D E F
w
//w
s:
tp

Maka disini hanya akan muncul satu kasus saja, yaitu E memberikan kado ke F.
ht

Pembahasan OSK Matematika SMA 2017 by Pak Anang (http://pak-anang.blogspot.com)

99
Shared by: www.catatanmatematika.com

Kasus 4.1
Maka E pasti memberikan kadonya ke F.
Mengapa?
Karena tidak mungkin E memberikan kado ke A, karena F akan mendapatkan kadonya sendiri.
Sehingga ilustrasinya sebagai berikut

A B C D E F

m
co
a.
ik
at
Dan otomatis, F akan member kadonya ke A.
Sehingga ilustrasinya sebagai berikut

em
A B C D E F

at
nm

Jadi, . ara
a

Secara sederhana berikut bagannya dari awal sampai akhir proses pengacakan ini.
at

AB
at
.c

Kasus 1.1 Kasus 1.2


BA B(C, D, E atau F)
w
w

Misal
BC
//w

Kasus 2.1 Kasus 2.2


CA C(D, E atau F)
s:

Misal
tp

CD
ht

Kasus 3.1 Kasus 3.2


DA D(E atau F)

Kasus 4.1
DE dan otomatis
EF

Pembahasan OSK Matematika SMA 2017 by Pak Anang (http://pak-anang.blogspot.com)

100
Shared by: www.catatanmatematika.com

Berarti sampai disini kasus sudah berhenti dan mari kita menghitung ulang seluruh kemungkinan
yang terjadi, yaitu:

. . . . . . .

m
co
a.
Jadi, banyaknya cara membagikan kado adalah 53 cara.

ik
at
Cara Alternatif:

em
Dengan cara manual kita dapat mencari banyaknya cara membagi kado.
Tanpa mengikutkan B yang sudah pasti mendapatkan kado dari A, maka hasil pengacakan yang
mungkin dapat dilihat seperti berikut:
at
Kemungkinan pertama, B memberikan kado ke A, sehingga terjadi pengacakan pada keempat
orang lain yaitu C, D, E, dan F sehingga menghasilkan bentuk sebagai berikut:
nm

24 permutasi yang mungkin dari CDEF adalah sebagai berikut:


a

CDEF DCEF ECDF FCDE


CDFE DCFE ECFD FCED
at

CEDF DECF EDCF FDCE


CEFD DEFC EDFC FDEC
at

CFDE DFCE EFCD FECD


CFED DFEC EFDC FEDC
.c

Maka diperoleh 9 buah kemungkinan pengacakan yang diperbolehkan yaitu yang bertanda biru.
w

Kemungkinan kedua, B memberikan kado ke selain A, berarti ada 4 kemungkinan, yaitu


w

memberikan kado tersebut ke C, D, E atau F.


Anggap B memberikan kado ke C, berarti ada 24 permutasi yang mungkin dari CDEF yang akan
//w

diletakkan pada ADEF, yaitu:

CDEF DCEF ECDF FCDE


s:

CDFE DCFE ECFD FCED


CEDF DECF EDCF FDCE
CEFD DEFC EDFC FDEC
tp

CFDE DFCE EFCD FECD


CFED DFEC EFDC FEDC
ht

Maka diperoleh 11 buah kemungkinan pengacakan yang diperbolehkan yaitu yang bertanda biru.
Sehingga, banyaknya kemungkinan adalah cara.

Jadi banyak kemungkinan seluruhnya adalah cara.

Pembahasan OSK Matematika SMA 2017 by Pak Anang (http://pak-anang.blogspot.com)

101
Shared by: www.catatanmatematika.com

TRIK SUPERKILAT:
Banyak cara membagikan kado adalah derangement (pengacakan) dengan ada tempat terlarang.
Dengan prinsip inklusi-eksklusi dapat dihitung sebagai

ara.

m
co
a.
ik
at
em
at
a nm
at
at
.c
w
w
//w
s:
tp
ht

Pembahasan OSK Matematika SMA 2017 by Pak Anang (http://pak-anang.blogspot.com)

102
Shared by: www.catatanmatematika.com

15. Bilangan asli terbesar sehingga dapat dinyatakan sebagai hasil perkalian dari bilangan
asli berurutan adalah ….

Pembahasan:
Perhatikan, …
buah akt r

m
Ide untuk menjadikan menjadi perkalian bilangan asli berurutan adalah dengan
memotong beberapa perkalian bilangan asli berurutan pertama.

co
Pandang bahwa adalah perkalian dari bilangan asli berurutan, maka bilangan terbesar
dapat diperoleh dengan memotong buah perkalian bilangan asli pertama, yaitu

a.
… … …

ik
buah akt r buah akt r buah akt r
buah akt r buah akt r

at
dimana, adalah hasil perkalian dari beberapa perkalian bilangan asli pertama yang terpotong.

Sehingga, karena … …

em
buah akt r
buah akt r

Jadi, diperoleh bahwa bilangan asli terbesar sehingga at dapat dinyatakan sebagai hasil
perkalian dari bilangan asli berurutan adalah 119.
a nm
at
at
.c
w
w
//w
s:
tp
ht

Pembahasan OSK Matematika SMA 2017 by Pak Anang (http://pak-anang.blogspot.com)

103
Shared by: www.catatanmatematika.com

16. Pada segitiga titik dan berturut-turut adalah titik tengah dan . Jika dan
saling tegak lurus, maka nilai minimum adalah ….

Pembahasan:
Perhatikan gambar di samping.

Karena dan berturut-turut titik tengah dan , maka

m
co
Perhatikan, dan sebangun dan , maka

a.
ik
Misal,

at
maka,

em
dan

Perhatikan,
misal,
, maka diperoleh
at
Perhatikan,
misal,
, maka diperoleh
nm
maka maka
t t t t

tan tan
a

tan tan tan tan


at

tan tan tan tan


at
.c
w
w

Sehingga,
//w

t t

Mengingat, dari diperoleh


s:

kalikan kedua ruas dengan


tp
ht

Jadi, nilai minimum t t

Pembahasan OSK Matematika SMA 2017 by Pak Anang (http://pak-anang.blogspot.com)

104
Shared by: www.catatanmatematika.com

17. Misalkan dan bilangan-bilangan bulat positif. Jajargenjang yang dibatasi oleh garis-garis
dan mempunyai luas 18. Jajargenjang yang
dibatasi oleh garis-garis dan mempunyai luas .
Nilai terkecil yang mungkin untuk adalah ….

Pembahasan:

m
co
a.
ik
at
em
Secara grafik, kita dapat melihat dengan mudah bahwa titik potong dengan sumbu Y untuk
keempat garis adalah:
1. Untuk jajargenjang dengan luas 18, adalah di
2. Untuk jajargenjang dengan luas 72, adalah di
dan
dan
at .
.
nm
Perhatikan bahwa luas jajargenjang menjadi 4 kali lebih besar, maka dengan prinsip
kesebangunan dan perbandingan, maka ukuran panjang sisi jajargenjang menjadi 2 kali lebih
besar dari semula.
a

Sehingga,
at

Perhatikan jarak ke adalah .


Perhatikan jarak ke adalah .
at

Padahal, ukuran sisi jajargenjang menjadi 2 kali lebih besar dari semula, sehingga
.c
w

Secara grafik, kita juga dapat melihat dengan mudah bahwa jajargenjang dipisahkan menjadi dua
bagian sama besar oleh sumbu Y.
w

Perhatikan luas bagian sebelah kanan sumbu Y adalah 9, sehingga:


//w

ehingga diper leh


s:
tp

adi
ht

Perhatikan, bahwa , sehingga dari hubungan juga dapat diperoleh .


Dan pandang bentuk dengan bilangan positif, maka nilai terkecil adalah
masing-masing dan

Sehingga, diperoleh penyelesaian min .


Jadi, nilai minimum .

Pembahasan OSK Matematika SMA 2017 by Pak Anang (http://pak-anang.blogspot.com)

105
Shared by: www.catatanmatematika.com

18. Seratus bilangan bulat disusun mengelilingi lingkaran sedemikian sehingga (menurut arah jarum
jam) setiap bilangan lebih besar daripada hasil penjumlahan dua bilangan sebelumnya. Maksimal
banyaknya bilangan bulat p siti yang terdapat pada lingkaran tersebut adalah ….

Pembahasan:

m
co
a.

ik

at

em
Perhatikan ilustrasi di atas.
at
nm
Terdapat 100 buah bilangan bulat yang mengelilingi lingkaran, sedemikian hingga menurut arah
jarum jam, setiap bilangan lebih besar dari hasil penjumlahan dua bilangan seterusnya.
a

Sehingga dapat dipahami bahwa pada susunan bilangan melingkar tersebut berlaku:
at
at

.
.
.c

.
w
w

Perhatikan pernyataan
//w

Artinya, paling tidak ada satu buah nilai diantara … yang bernilai negatif.

Tanpa mengurangi keumuman, misalkan , perhatikan bahwa berarti paling


s:

tidak ada satu nilai yang negatif diantara atau .


Misalkan juga , maka akibatnya paling tidak juga ada satu nilai yang negatif diantara
tp

atau . Agar bilangan negatif minimum, maka .


Proses tersebut berulang sampai .
ht

Sehingga, diperoleh … dan … .

Jadi, banyaknya bilangan positif ada sebanyak 49 buah.

Pembahasan OSK Matematika SMA 2017 by Pak Anang (http://pak-anang.blogspot.com)

106
Shared by: www.catatanmatematika.com

Cara Alternatif 1:

m

co

a.

ik
at
em
Perhatikan ilustrasi di atas.

Dan ingat sifat barisan Fibonacci, yang nilai sukunya sama dengan jumlah dua suku sebelumnya.
at
Padahal di soal ini nilai suku lebih dari jumlah suku sebelumnya.
nm
Barisan Fib na i misalkan saja … adalah barisan yang m n t n naik.
Artinya, nilainya akan terus menerus membesar. Padahal di soal ini, bilangan tersebut disusun
melingkar, sehingga nilai paling ujung di akhir barisan akan bertemu dengan nilai di awal barisan.
Jelas apabila barisan tersebut monoton naik, ataupun monoton turun tidak mungkin terpilih
a

sebagai barisan yang dimaksudkan pada soal.


at

Sehingga pola barisan pada soal seharusnya adalah bertanda selang-seling. Positif, negatif, positif,
negatif. Dan kita pilih suku positifnya adalah bilangan bulat positif terkecil, yaitu 1.
at

Misal barisan tersebut adalah


.c


w

Sehingga diperoleh sedangkan .


Jadi, diperoleh
w
//w
s:
tp

Pertanyaannya kini adalah “Apakah juga bernilai ”


Agar berlaku , dan maka
ht

Jadi, dari ketiga pertidaksamaan tersebut, jelas sekali bahwa .

Maka bilangan positif yang mungkin yaitu sebanyak 49 buah.

Pembahasan OSK Matematika SMA 2017 by Pak Anang (http://pak-anang.blogspot.com)

107
Shared by: www.catatanmatematika.com

19. Untuk sebarang bilangan asli , misalkan adalah jumlah digit-digit dari dalam penulisan
desimal. Jika , maka nilai maksimum dari adalah ….

Pembahasan:
Perhatikan,
Misal, , maka untuk semua bilangan bulat non negatif
jumlah digit-digit dari adalah 5.

m
Contohnya,
Apabila , maka diperoleh:

co
Sehingga, jumlah digit-digit dari 1121 adalah .
Terbukti, .

a.
Jadi diperoleh bentuk yang apabila dijabarkan akan

ik
menjadi,

at
sebanyak suku sebanyak suku sebanyak suku sebanyak suku

em
sebanyak suku sebanyak suku sebanyak suku

Sehingga, terdapat 7 bentuk suku yang dapat dikelompokkan berdasarkan pangkat dari ,
yaitu:
Cara penjumlahan
menghasilkan bentuk 5
Bentuk
at Koefisien
Banyak suku yang
terbentuk
a nm
at
at
.c
w

Keterangan:
Pandang bentuk sebagai perkalian berulang sebagai berikut:
w

maka,
//w

 Koefisien dari adalah berapa banyak cara dapat terbentuk dari perkalian berulang sebanyak lima kali,
dengan mengacak bentuk yaitu
s:

dan seterusnya…
tp

Sehingga, koefisien dapat dihitung dengan konsep permutasi unsur dengan ada unsur yang sama. Diperoleh koefisien dari
adalah .
ht

 Banyak suku yang terbentuk dari dst adalah berapa banyak cara pangkat dari dapat terbentuk
dari huruf-huruf yang tersedia yaitu .

Yaitu, memilih sebuah huruf secara permutasi untuk dipasangkan dengan 3, dan memilih dua huruf yang lain dipasangkan
dengan 1 dari dua huruf yang tersisa secara kombinasi. Sehingga, buah suku.

Pembahasan OSK Matematika SMA 2017 by Pak Anang (http://pak-anang.blogspot.com)

108
Shared by: www.catatanmatematika.com

Lalu pandang lagi, bahwa apabila dua digit dijumlahkan dan lebih dari 10, maka jumlah digit
mereka akan lebih kecil dari jumlah kedua digit tersebut.

Perhatikan, sebagai contohnya misalkan saja ada dua bilangan yaitu 8 dan 9.
Maka, jumlah kedua digit adalah . Sedangkan, apabila kedua digit dijumlahkan , maka jumlah digit dari adalah
. Jelas bahwa .

Sehingga, jumlah digit-digit dari adalah akan maksimum

m
apabila setiap bentuk memiliki bentuk pangkat yang tidak sama . Artinya:

co
sehingga seluruh bentuk adalah bentuk berbeda nilainya.

Maka, bilangan yang menghasilkan jumlah digit terbesar adalah:

a.
dengan dan
… hanya dilihat jumlah digitnya saja (contoh: 20 hanya dilihat sebagai )

ik
Jadi, jumlah digit maksimum dari adalah:

at
Banyak suku yang
Bentuk Koefisien Jumlah digit
terbentuk

em
at
a nm
at

Jumlah maksimum dari digit-digit 398


at

Jadi, jumlah maksimum dari digit-digit adalah


.c
w
w
//w
s:
tp
ht

Pembahasan OSK Matematika SMA 2017 by Pak Anang (http://pak-anang.blogspot.com)

109
Shared by: www.catatanmatematika.com

20. Diberikan segitiga dengan dan . Misalkan suatu titik pada garis
bagi yang terletak di dalam dan misalkan suatu titik pada sisi (dengan ).
Garis dan memotong dan berturut-turut di dan . Jika dan
, maka nilai adalah ….

Pembahasan:
Perhatikan ilustrasi berikut.

m
Karena membagi sudut sama besar, maka menurut

co
sifat garis bagi berlaku:
dan

a.
Perhatikan,

ik
Misal, dan , maka
Dengan menggunakan sudut luar segitiga diperoleh:

at
em
Perhatian sekali lagi pada dan , berlaku
- kedua sisi berhimpit
- si at garis bagi sudut
- si at sudut luar segitiga at
Jadi, dengan prinsip si-su-su maka kongruen dengan , sehingga,
nm
Perhatikan, karena adalah segitiga sama kaki, maka selain merupakan garis bagi, maka
juga merupakan garis berat, dan garis tinggi .
Oleh karena itu, dan .
a

Perhatikan, , sehingga
at

misal, dan
maka,
at
.c

Dengan menggunakan kesamaan pada kedua persamaan diatas, maka diperoleh:


w
w
//w

Padahal, sebangun , sehingga


s:
tp

Substitusikan ke , sehingga diperoleh:


ht

Pembahasan OSK Matematika SMA 2017 by Pak Anang (http://pak-anang.blogspot.com)

110
Shared by: www.catatanmatematika.com

Perhatikan, pada berlaku,

m
co
a.
ik
at
em
at
a nm
at
at
.c
w
w
//w
s:
tp
ht

Pembahasan OSK Matematika SMA 2017 by Pak Anang (http://pak-anang.blogspot.com)

111
Shared by: www.catatanmatematika.com

PEMBAHASAN SOAL
OLIMPIADE SAINS MATEMATIKA SMA
TINGKAT KABUPATEN/KOTA

m
28 Februari 2018
By Pak Anang (http://pak-anang.blogspot.com)

co
1. Misalkan 𝑎, 𝑏, dan 𝑐 adalah tiga bilangan 𝑏𝑒𝑟𝑏𝑒𝑑𝑎. Jika ketiga bilangan tersebut merupakan
bilangan asli satu digit maka jumlah terbesar akar-akar persamaan (𝑥 − 𝑎)(𝑥 − 𝑏) + (𝑥 − 𝑏)(𝑥 −

a.
𝑐) = 0 yang mungkin adalah ….

ik
Pembahasan:
Perhatikan penjabaran bentuk aljabar tersebut.

at
(𝑥 − 𝑎)(𝑥 − 𝑏) + (𝑥 − 𝑏)(𝑥 − 𝑐) = 0
⇔ 𝑥 − (𝑎 + 𝑏)𝑥 + 𝑎𝑏 + 𝑥 2 − (𝑏 + 𝑐)𝑥 + 𝑏𝑐 = 0
2

em
⇔ 2𝑥 2 − (𝑎 + 2𝑏 + 𝑐)𝑥 + (𝑎𝑏 + 𝑏𝑐) = 0

Sehingga, jika akar-akar dari persamaan kuadrat 𝑥 2 − (𝑎 + 2𝑏 + 𝑐)𝑥 + (𝑎𝑏 + 𝑏𝑐) = 0 adalah 𝑥1
dan 𝑥2 , maka dengan rumus jumlah akar-akar persamaan kuadrat diperoleh:
at
𝑎 + 2𝑏 + 𝑐
𝑥1 + 𝑥2 =
2
nm
Perhatikan juga bahwa 𝑎, 𝑏, dan 𝑐 adalah tiga bilangan satu digit berbeda, sehingga 𝑎 + 2𝑏 + 𝑐 akan
maksimum apabila 𝑏 adalah bilangan terbesar dan 𝑎, 𝑐 masing-masing dipilih bilangan satu digit
berurutan yang lebih kecil dari 𝑏.
a
at

Sehingga apabila 𝑏 = 9 dan masing-masing 𝑎 atau 𝑐 adalah 8 atau 7, diperoleh jumlah terbesar akar-
akar persamaan kuadrat tersebut adalah
at

𝑎 + 2𝑏 + 𝑐 33
𝑥1 + 𝑥2 = =
2 2
.c

TRIK SUPERKILAT:
Perhatikan penjabaran bentuk aljabar tersebut.
w

(𝑥 − 𝑎)(𝑥 − 𝑏) + (𝑥 − 𝑏)(𝑥 − 𝑐) = 0
w

⇔ (𝑥 − 𝑏)(2𝑥 − (𝑎 + 𝑐)) = 0
𝑎+𝑐
⇔ 𝑥1 = 𝑏 atau 𝑥2 =
//w

Sehingga, diperoleh jumlah akar-akarnya adalah


s:

𝑎+𝑐
𝑥1 + 𝑥2 = 𝑏 +
2
tp

𝑎+𝑐
Dengan mudah kita tahu bahwa 𝑏 + 2 akan maksimum apabila 𝑏 merupakan bilangan terbesar
ht

yaitu 9. Jadi, 𝑎 atau 𝑐 adalah 7 atau 8, begitu juga sebaliknya.

𝑎+𝑐 15
𝑥1 + 𝑥2 = 𝑏 + =9+ = 9 + 7,5 = 16,5
2 2

Pembahasan OSK Matematika SMA 2018 by Pak Anang (http://pak-anang.blogspot.com)

112
Shared by: www.catatanmatematika.com

2. Setiap sel dari suatu tabel berukuran 2 × 2 dapat diisi dengan bilangan 1, 2, atau 3. Misalkan 𝑁
adalah banyaknya tabel yang memenuhi kedua sifat berikut sekaligus:
• untuk setiap baris, hasil penjumlahannya genap
• untuk setiap kolom, hasil penjumlahannya genap
Nilai 𝑁 adalah ….

Pembahasan:

m
Perhatikan tabel 2 × 2 berikut!

co
𝑎 𝑏
𝑐 𝑑

a.
Dengan memperhatikan bahwa hasil penjumlahan setiap baris dan kolom adalah genap, maka
diperoleh kedua bilangan pada setiap baris atau kolom memiliki paritas yang sama.

ik
Perhatikan juga bahwa 𝑎, 𝑏, 𝑐, atau 𝑑 hanya dapat diisi dengan bilangan 1, 2, atau 3.

at
Banyak tabel yang memenuhi dapat ditentukan dengan membagi kasus:

em
• untuk 𝑎, 𝑏, 𝑐, 𝑑 bilangan ganjil
maka ada tiga kemungkinan
- keempat bilangan 𝑎, 𝑏, 𝑐, 𝑑 adalah bilangan yang sama, sebanyak 2 𝐶1 = 2 cara.
4!
- diantara bilangan 𝑎, 𝑏, 𝑐, 𝑑 ada tiga bilangan yang sama, sebanyak 3! × 2 𝐶1 = 8 cara.
at 4!
- diantara bilangan 𝑎, 𝑏, 𝑐, 𝑑 ada dua bilangan yang sama, sebanyak 2!2! = 6 cara.
nm
• untuk 𝑎, 𝑏, 𝑐, 𝑑 bilangan genap
maka hanya ada satu kemungkinan yaitu keempat bilangan 𝑎, 𝑏, 𝑐, 𝑑 adalah bilangan 2. Sehingga
ada sebanyak 1 cara.
a

Jadi, total banyak tabel yang memenuhi adalah sebanyak 2 + 8 + 6 + 1 = 17 cara.


at

TRIK SUPERKILAT:
at

Banyak tabel yang memenuhi dapat ditentukan dengan membagi kasus:


• Kasus pertama: 𝑎, 𝑏, 𝑐, 𝑑 adalah bilangan ganjil.
4 4 4 4 4
.c

Banyak kejadian adalah ( ) + ( ) + ( ) + ( ) + ( ) = 24 = 16


0 1 2 3 4
• Kasus kedua: 𝑎, 𝑏, 𝑐, 𝑑 adalah bilangan genap.
w

Hanya ada satu kemungkinan, yaitu 𝑎, 𝑏, 𝑐, 𝑑 adalah bilangan 2.


w

Jadi, total banyak tabel yang memenuhi adalah sebanyak 16 + 1 = 17 cara.


//w
s:
tp
ht

Pembahasan OSK Matematika SMA 2018 by Pak Anang (http://pak-anang.blogspot.com)

113
Shared by: www.catatanmatematika.com

3. Diberikan persegi berukuran 3 × 3 satuan seperti pada gambar. Luas segilima yang diarsir adalah
….

m
co
Pembahasan:
Perhatikan gambar berikut!

a.
𝐶

ik
𝐼 𝐻

at
𝐺

em
𝐽 𝐷 𝐹 𝐸

𝐴 𝐾 𝐵 at
Perhatikan, karena 𝐴𝐵 ∥ 𝐷𝐸, maka ∆𝐶𝐴𝐵 ∼ ∆𝐶𝐷𝐸 sehingga diperoleh perbandingan
nm
𝐶𝐸 𝐷𝐸 𝐶𝐸 2 4
= ⇒ 𝐷𝐸 = × 𝐴𝐵 = × 2 =
𝐶𝐵 𝐴𝐵 𝐶𝐵 3 3
a

Sehingga, karena 𝐷𝐸 = 𝐷𝐹 + 𝐹𝐸, dan 𝐹𝐸 = 1, maka diperoleh


4 1
at

𝐷𝐹 = 𝐷𝐸 − 𝐹𝐸 = − 1 =
3 3
at

Perhatikan, karena ∆𝐷𝐹𝐺 ∼ ∆𝐴𝐵𝐶 sehingga diperoleh perbandingan


1
𝐹𝐺 𝐷𝐹 𝐷𝐹 1
= ⇒ 𝐹𝐺 = × 𝐵𝐶 = 3 × 3 =
.c

𝐵𝐶 𝐴𝐵 𝐴𝐵 2 2
w

1 1 1 1 1
Jadi, [𝐷𝐹𝐺] = 2 ∙ 𝐷𝐹 ∙ 𝐹𝐺 = 2 ∙ 3 ∙ 2 = 12
w

1 11
Sehingga, [𝐷𝐺𝐻𝐼𝐽] = [𝐹𝐻𝐼𝐽] − [𝐷𝐹𝐺] = 1 − 12 = 12.
//w

TRIK SUPERKILAT:
1 1 2 1 1
Perhatikan bahwa ∆𝐷𝐹𝐺 ∼ ∆𝐴𝐵𝐶, sehingga karena 𝐷𝐹 = 6 𝐴𝐵, maka [𝐷𝐹𝐺] = (6) ∙ 2 ∙ 2 ∙ 3 = 12.
s:

1 11
Sehingga [𝐷𝐺𝐻𝐼𝐽] = 1 − 12 = 12.
tp
ht

Pembahasan OSK Matematika SMA 2018 by Pak Anang (http://pak-anang.blogspot.com)

114
Shared by: www.catatanmatematika.com

4. Parabola 𝑦 = 𝑎𝑥 2 − 4 dan 𝑦 = 8 − 𝑏𝑥 2 memotong sumbu koordinat pada tepat empat titik.


Keempat titik tersebut merupakan titik-titik sudut layang-layang dengan luas 24. Nilai 𝑎 + 𝑏 adalah
….

Pembahasan:
Perhatikan, titik potong parabola 𝑦 = 𝑎𝑥 2 − 4 pada sumbu Y adalah di titik (0, −4). Sedangkan, titik
potong parabola 𝑦 = 8 − 𝑏𝑥 2 pada sumbu Y adalah di titik (0, 8).

m
Perhatikan juga, agar dapat diperoleh dua titik lagi sebagai titik-titik sudut layang-layang yang lain,

co
maka titik potong parabola 𝑦 = 𝑎𝑥 2 − 4 dan 𝑦 = 8 − 𝑏𝑥 2 pada sumbu X seharusnya adalah pada
titik yang sama, sehingga dapat disimpulkan kedua kurva berpotongan di sumbu X.

a.
Sehingga, titik potong di sumbu X dapat ditentukan dengan
𝑦1 = 𝑦2

ik
⇔ 𝑎𝑥 − 4 = 8 − 𝑏𝑥 2
2

⇔ (𝑎 + 𝑏)𝑥 2 − 12 = 0

at
12
⇔ 𝑥 = ±√

em
𝑎+𝑏

12 12
Jadi, titik potong kedua parabola pada sumbu X adalah di titik (√𝑎+𝑏 , 0) dan (−√𝑎+𝑏 , 0).
Padahal, luas layang-layang adalah 24, sehingga
1
at
nm
𝐿 = × 𝑑1 × 𝑑2
2
1 12 12
⇔ 24 = × |8 − (−4)| × |√ − (−√ )|
2 𝑎+𝑏 𝑎+𝑏
a
at

1 12
⇔ 24 = × 12 × 2√
2 𝑎+𝑏
at

12
2=√
.c


𝑎+𝑏
12
w

⇔ 4=
𝑎+𝑏
w

⇔𝑎+𝑏 =3
//w
s:
tp
ht

Pembahasan OSK Matematika SMA 2018 by Pak Anang (http://pak-anang.blogspot.com)

115
Shared by: www.catatanmatematika.com

5. Untuk setiap bilangan asli 𝑛 didefinisikan 𝑠(𝑛) sebagai hasil penjumlahan dari semua digit-digit
dari 𝑛. Banyaknya bilangan asli 𝑑 sehingga 𝑑 habis membagi 𝑛 − 𝑠(𝑛) untuk setiap bilangan asli 𝑛
adalah ….

Pembahasan:
Perhatikan, bilangan asli 𝑛 dapat dinyatakan sebagai 𝑛 = 𝑎0 ∙ 100 + 𝑎1 ∙ 101 + 𝑎2 ∙ 102 + ⋯, maka
jika 𝑠(𝑛) didefinisikan sebagai hasil penjumlahan dari semua digit-digit dari 𝑛, maka diperoleh

m
𝑠(𝑛) = 𝑎0 + 𝑎1 + 𝑎2 + ⋯

co
Misal, 𝑝 = 𝑛 − 𝑠(𝑛), maka
𝑝 = 𝑛 − 𝑠(𝑛)
= (𝑎0 ∙ 100 + 𝑎1 ∙ 101 + 𝑎2 ∙ 102 + ⋯ ) − (𝑎0 + 𝑎1 + 𝑎2 + ⋯ )

a.
= 𝑎0 (100 − 1) + 𝑎1 (101 − 1) + 𝑎2 (102 − 1) + ⋯
= 9𝑎1 + 99𝑎2 + 999𝑎3 + ⋯

ik
= 9(𝑎1 + 11𝑎2 + 111𝑎3 + ⋯ )

at
Sehingga, 9|𝑛 − 𝑠(𝑛). Jadi bilangan asli 𝑑 adalah faktor bulat positif dari 9, yaitu 1, 3, dan 9.

em
Jadi, ada sebanyak 3 buah bilangan 𝑑 yang memenuhi.

at
a nm
at
at
.c
w
w
//w
s:
tp
ht

Pembahasan OSK Matematika SMA 2018 by Pak Anang (http://pak-anang.blogspot.com)

116
Shared by: www.catatanmatematika.com

6. Diketahui 𝑥 dan 𝑦 bilangan prima dengan 𝑥 < 𝑦, dan 𝑥 3 + 𝑦 3 + 2018 = 30𝑦 2 − 300𝑦 + 3018. Nilai
𝑥 yang memenuhi ….

Pembahasan:
Perhatikan,
𝑥 3 + 𝑦 3 + 2018 = 30𝑦 2 − 300𝑦 + 3018
⇔ 𝑥 3 + 𝑦 3 − 30𝑦 2 + 300𝑦 − 1000 = 0

m
⇔ 𝑥 3 + (𝑦 − 10)3 = 0

co
Sehingga, diperoleh
𝑥 = −(𝑦 − 10)
⇔ 𝑥 + 𝑦 = 10

a.
Karena, 𝑥, 𝑦 adalah bilangan prima, maka dua buah bilangan prima yang jumlahnya 10 adalah 3 dan

ik
7. Mengingat 𝑥 < 𝑦, sehingga dapat diperoleh 𝑥 = 3 dan 𝑦 = 7.

at
Jadi, 𝑥 yang memenuhi adalah 3.

em
at
a nm
at
at
.c
w
w
//w
s:
tp
ht

Pembahasan OSK Matematika SMA 2018 by Pak Anang (http://pak-anang.blogspot.com)

117
Shared by: www.catatanmatematika.com

7. Diberikan dua bilangan asli dua angka yang selisihnya 10. Diketahui bahwa bilangan yang kecil
merupakan kelipatan 3, sedangkan lainnya merupakan kelipatan 7. Diketahui pula bahwa jumlah
semua faktor prima kedua bilangan tersebut adalah 17. Jumlah dua bilangan tersebut adalah ….

Pembahasan:
Perhatikan, misal kedua bilangan tersebut adalah 𝑥 dan 𝑦, karena 𝑥 adalah bilangan kelipatan 7 dan
𝑦 adalah bilangan kelipatan 3, maka untuk 𝑚 dan 𝑛 adalah suatu bilangan asli, 𝑥 dan 𝑦 dapat

m
dinyatakan sebagai
𝑥 = 7𝑚

co
𝑦 = 3𝑛

Karena selisih kedua bilangan adalah 10, dan 𝑥 > 𝑦, maka 𝑥 − 𝑦 = 10. Ini sama saja dengan

a.
persamaan 7𝑚 − 3𝑛 = 10.

ik
Nilai 𝑚 dan 𝑛 dapat ditentukan menggunakan pembalikan algoritma Euclid, yaitu
7=2×3+1

at
Sehingga,

em
1=7−2×3

Dengan mengalikan 10 kedua ruas diperoleh


10 = 70 − 60

Sehingga, diperoleh 𝑚 = 10 dan 𝑛 = 20.


at
nm
Sehingga, solusi umumnya adalah
𝑚 = 10 − 3𝑡 ⇒ 𝑥 = 70 − 21𝑡
𝑛 = 20 − 7𝑡 ⇒ 𝑦 = 60 − 21𝑡
a
at

Diperoleh pasangan bilangan dua digit 𝑥, 𝑦 yang memenuhi adalah


(𝑥, 𝑦) = {(28, 18), (49, 39), (70, 60), (91, 81)}
at

Perhatikan bahwa jumlah semua faktor prima 𝑥 dan y adalah 17, maka 17 = 3 + 𝑝 + 𝑞 + 7.
Maka 𝑝 + 𝑞 = 7, sehingga bilangan prima 𝑝, 𝑞 yang memenuhi hanyalah 2 dan 5.
.c

Sehingga, jelas diantara pasangan 𝑥, 𝑦 yang memiliki faktor prima 5 hanyalah 𝑥 = 70 dan 𝑦 = 60.
w

Jadi, jumlah kedua bilangan tersebut adalah 𝑥 + 𝑦 = 70 + 60 = 130.


w
//w
s:
tp
ht

Pembahasan OSK Matematika SMA 2018 by Pak Anang (http://pak-anang.blogspot.com)

118
Shared by: www.catatanmatematika.com

TRIK SUPERKILAT 1:
Perhatikan, 𝑥 ≡ 0 (mod 7) dan 𝑦 ≡ 0 (mod 3). Karena selisih kedua bilangan adalah 10, dan 𝑥 > 𝑦,
maka 𝑥 − 𝑦 = 10, sehingga
𝑦 = 𝑥 − 10 ≡ 0 (mod 3) ⇒ 𝑥 ≡ 1 (mod 3) ⇒ 𝑥 = 3𝑡 + 1
Sehingga,
𝑥 = 3𝑡 + 1 ≡ 0 (mod 7) ⇒ 3𝑡 = 6 (mod 7) ⇒ 3𝑡 = 7𝑢 + 6

m
Diperoleh, 𝑥 = 21𝑢 + 7 dan nilai 𝑥 yang memenuhi adalah 𝑥 = {28, 49, 70, 91}

co
Perhatikan bahwa jumlah semua faktor prima 𝑥 dan 𝑦 adalah 17, maka 17 = 3 + 𝑝 + 𝑞 + 7.
Maka 𝑝 + 𝑞 = 7, sehingga bilangan prima 𝑝, 𝑞 yang memenuhi hanyalah 2 dan 5.

a.
Sehingga, diantara pasangan 𝑥, 𝑦 yang memiliki faktor prima 5 hanyalah 𝑥 = 70, akibatnya 𝑦 = 60.

ik
Jadi, jumlah kedua bilangan tersebut adalah 𝑥 + 𝑦 = 70 + 60 = 130.

at
TRIK SUPERKILAT 2 (LOGIKA PRAKTIS):
Perhatikan bahwa jumlah semua faktor prima 𝑥 dan 𝑦 adalah 17, maka 17 = 3 + 𝑝 + 𝑞 + 7.

em
Maka 𝑝 + 𝑞 = 7, sehingga bilangan prima 𝑝, 𝑞 yang memenuhi hanyalah 2 dan 5.

Akibatnya, kemungkinan yang terjadi adalah 𝑥 = 7(2𝑎 ∙ 3𝑏 ∙ 5𝑐 ∙ 7𝑑 ) dan 𝑦 = 3(2𝑘 ∙ 3𝑙 ∙ 5𝑚 ∙ 7𝑛 ) .

LOGIKA:
at
• 5 adalah semestinya menjadi salah satu faktor prima dari salah satu dari 𝑥 atau 𝑦.
nm
• Mengingat 𝑥 − 𝑦 = 10, suatu bilangan kelipatan 5 pasti memiliki selisih 10 dengan bilangan
yang lain, apabila bilangan yang lain juga kelipatan 5. Sehingga, disimpulkan 5 sudah pasti
menjadi faktor dari baik 𝑥 maupun 𝑦.
a

Sehingga dengan cara mendaftar kemungkinan secara manual:


at

𝑥 = 35𝑡 ⇒ 35, 70
𝑦 = 15𝑢 ⇒ 15, 30, 45, 60, 90
at

Jelas yang memenuhi 𝑥 − 𝑦 = 10, adalah 𝑥 = 70 dan 𝑦 = 60.


.c

Jadi, jumlah kedua bilangan tersebut adalah 𝑥 + 𝑦 = 70 + 60 = 130.


w
w
//w
s:
tp
ht

Pembahasan OSK Matematika SMA 2018 by Pak Anang (http://pak-anang.blogspot.com)

119
Shared by: www.catatanmatematika.com

TRIK SUPERKILAT 3 (LOGIKA PRAKTIS):


Perhatikan bahwa jumlah semua faktor prima 𝑥 dan 𝑦 adalah 17, maka 17 = 3 + 𝑝 + 𝑞 + 7.
Maka 𝑝 + 𝑞 = 7, sehingga bilangan prima 𝑝, 𝑞 yang memenuhi hanyalah 2 dan 5.

Akibatnya, kemungkinan yang terjadi adalah 𝑥 = 7(2𝑎 ∙ 3𝑏 ∙ 5𝑐 ∙ 7𝑑 ) dan 𝑦 = 3(2𝑘 ∙ 3𝑙 ∙ 5𝑚 ∙ 7𝑛 ) .

LOGIKA:

m
• 5 adalah semestinya menjadi salah satu faktor prima dari salah satu dari 𝑥 atau 𝑦.
• Mengingat 𝑥 − 𝑦 = 10, suatu bilangan kelipatan 5 pasti memiliki selisih 10 dengan bilangan

co
yang lain, apabila bilangan yang lain juga kelipatan 5. Sehingga, disimpulkan 5 sudah pasti
menjadi faktor dari baik 𝑥 maupun 𝑦.
• Karena 2 belum menjadi faktor dari masing-masing bilangan, maka 2 pasti juga menjadi faktor

a.
dari salah satu bilangan. Dan karena selisihnya 10, merupakan kelipatan 10, berarti bilangan
lain juga kelipatan 2.

ik
• Akibatnya karena 2 dan 5 adalah faktor setiap bilangan, maka keduanya adalah kelipatan 10.

at
Sehingga, kemungkinan yang terjadi hanyalah
𝑥 = 70𝑡 ⇒ 70

em
𝑦 = 30𝑢 ⇒ 30, 60

Jelas yang memenuhi 𝑥 − 𝑦 = 10, adalah 𝑥 = 70 dan 𝑦 = 60.


at
Jadi, jumlah kedua bilangan tersebut adalah 𝑥 + 𝑦 = 70 + 60 = 130.
a nm
at
at
.c
w
w
//w
s:
tp
ht

Pembahasan OSK Matematika SMA 2018 by Pak Anang (http://pak-anang.blogspot.com)

120
Shared by: www.catatanmatematika.com

8. Diberikan satu koin yang tidak seimbang. Bila koin tersebut ditos satu kali, peluang muncul angka
1
adalah . Jika ditos 𝑛 kali, peluang muncul tepat dua angka sama dengan peluang muncul tepat tiga
4
angka. Nilai 𝑛 adalah ….

Pembahasan:
Perhatikan, dengan menggunakan konsep distribusi binomial, misal 𝑝 = peluang kejadian muncul
1 3

m
angka, maka 𝑝 = 4 dan 1 − 𝑝 = 4.

co
Apabila satu koin ditos 𝑛 kali, maka peluang muncul tepat dua angka sama dengan peluang muncul
tepat tiga angka dapat dinyatakan sebagai
𝑃(𝑋 = 2) = 𝑃(𝑋 = 3)

a.
⇔ 2 (1
𝑛 𝐶2 ∙ 𝑝 ∙ − 𝑝)(𝑛−2) = 𝑛 𝐶3 ∙ 𝑝3 ∙ (1 − 𝑝)(𝑛−3)
𝑛! 1 2 3 (𝑛−2) 𝑛! 1 3 3 (𝑛−3)

ik
⇔ ∙( ) ∙( ) = ∙( ) ∙( )
(𝑛 − 2)! 2! 4 4 (𝑛 − 3)! 3! 4 4
𝑛 ∙ (𝑛 − 1) ∙ (𝑛 − 2)! 3 𝑛 ∙ (𝑛 − 1) ∙ (𝑛 − 2) ∙ (𝑛 − 3)! 1

at
⇔ ∙ = ∙
(𝑛 − 2)! ∙ 2 4 (𝑛 − 3)! ∙ 3! 4
3 𝑛−2

em
⇔ =
2 6
⇔ 18 = 2𝑛 − 4
⇔ 22 = 2𝑛
⇔ 𝑛 = 11 at
Jadi, nilai 𝑛 yang memenuhi adalah 11.
a nm
at
at
.c
w
w
//w
s:
tp
ht

Pembahasan OSK Matematika SMA 2018 by Pak Anang (http://pak-anang.blogspot.com)

121
Shared by: www.catatanmatematika.com

9. Panjang sisi-sisi dari segitiga merupakan bilangan asli yang berurutan. Diketahui bahwa garis berat
dari segitiga tegak lurus dengan salah satu garis baginya. Keliling segitiga itu adalah ….

Pembahasan:
Perhatikan gambar segitiga berikut
𝐶

m
𝐸
𝑥

co
𝐴 𝑥 𝐷 𝑥 𝐵

a.
𝐶𝐷 merupakan garis berat dan 𝐴𝐸 merupakan garis bagi, keduanya berpotongan saling tegak lurus.

ik
Perhatikan segitiga 𝐴𝐷𝐶 sama kaki, sehingga 𝐴𝐷 = 𝐴𝐶. Misal 𝐴𝐷 = 𝐴𝐶 = 𝐷𝐵 = 𝑥.

at
Perhatikan juga, karena sisi-sisi segitiga merupakan bilangan asli yang berurutan, maka selisih dari

em
dua sisi segitiga adalah 1 atau 2.

Kasus pertama, selisih dua sisi segitiga adalah 1, sehingga 2𝑥 − 𝑥 = 1 ⇒ 𝑥 = 1


Karena 𝑥 = 1, maka 𝑏 = 1, 𝑐 = 2, sehingga
at
• 𝑎 = 0, tidak memenuhi karena sisi segitiga tidak mungkin nol
• 𝑎 = 3, tidak mungkin karena tidak memenuhi ketaksamaan 𝑏 + 𝑐 > 𝑎
nm
Kasus kedua, selisih dua sisi segitiga adalah 2, sehingga 2𝑥 − 𝑥 = 2 ⇒ 𝑥 = 2
Karena 𝑥 = 2, maka 𝑏 = 2, 𝑐 = 4, sehingga
• 𝑎 = 3, memenuhi.
a
at

Sehingga, sisi segitiga adalah 𝑎 = 3, 𝑏 = 2, 𝑐 = 4.

Jadi keliling segitiga adalah 𝑎 + 𝑏 + 𝑐 = 3 + 2 + 4 = 9.


at

TRIK SUPERKILAT:
.c

Perhatikan, karena panjang sisi-sisi segitiga adalah bilangan asli yang berurutan, dan dari gambar
kita tahu bahwa 𝐴𝐵 = 2 ∙ 𝐴𝐶.
w

Jadi, kemungkinan tiga bilangan urut, dimana salah satunya adalah dua kali dari yang lain adalah:
w

• 1, 2, 3
Namun karena 1 + 2 ≯ 3, maka jelas segitiga ini tidak memenuhi ketaksamaan segitiga.
//w

• 2, 3, 4
Benar, bahwa 2 + 3 > 4, maka jelas segitiga ini memenuhi ketaksamaan segitiga.
s:

Jadi, keliling segitiga adalah 2 + 3 + 4 = 9.


tp
ht

Pembahasan OSK Matematika SMA 2018 by Pak Anang (http://pak-anang.blogspot.com)

122
Shared by: www.catatanmatematika.com

10. Diberikan suku banyak 𝑝(𝑥) dengan 𝑝(𝑥)2 + 𝑝(𝑥 2 ) = 2𝑥 2 untuk setiap bilangan real 𝑥. Jika
𝑝(1) ≠ 1 maka jumlah semua nilai 𝑝(10) yang mungkin adalah ….

Pembahasan:
Perhatikan, anggap 𝑝(𝑥) = 𝑎𝑥 𝑛 + 𝑞(𝑥), 𝑎 ≠ 0, 𝑞(𝑥) suku banyak derajat 𝑘 dengan 0 ≤ 𝑘 < 𝑛, maka
𝑝(𝑥)2 + 𝑝(𝑥 2 ) = 2𝑥 2
2
⇔ (𝑎𝑥 𝑛 + 𝑞(𝑥)) + (𝑎(𝑥 2 )𝑛 + 𝑞(𝑥 2 )) = 2𝑥 2

m
⇔ 𝑎2 𝑥 2𝑛 + 2𝑎𝑥 𝑛 𝑞(𝑥) + 𝑞(𝑥)2 + 𝑎𝑥 2𝑛 + 𝑞(𝑥 2 ) = 2𝑥 2
⇔ (𝑎2 + 𝑎)𝑥 2𝑛 + 2𝑎𝑥 𝑛 𝑞(𝑥) + 𝑞(𝑥)2 + 𝑞(𝑥 2 ) = 2𝑥 2

co
Sehingga, dengan memperhatikan kesamaan di atas, maka kemungkinan yang terjadi adalah

a.
• (𝑎2 + 𝑎)𝑥 2𝑛 = 2𝑥 2 , maka 𝑛 = 1 dengan 𝑎2 + 𝑎 = 2.
• (𝑎2 + 𝑎)𝑥 2𝑛 + 2𝑎𝑥 𝑛 𝑞(𝑥) = 2𝑥 2 , apabila 𝑎2 + 𝑎 = 0 maka 𝑛 + 𝑘 = 2.

ik
Perhatikan, 𝑛 + 𝑘 = 2 ⇒ 𝑘 = 2 − 𝑛, maka
0≤𝑘 <𝑛 ⇒ 0≤2−𝑛 <𝑛

at
⇔ 𝑛 ≤ 2 < 2𝑛
⇔ 1<𝑛≤2
Jelas bahwa 𝑛 = 2.

em
Jadi, suku banyak 𝑝(𝑥)2 + 𝑝(𝑥 2 ) = 2𝑥 2 , agar kesamaan berlaku maka
• 𝑝(𝑥) adalah suku banyak berderajat satu.
• 𝑝(𝑥) adalah suku banyak berderajat dua. at
nm
Kasus pertama, 𝑝(𝑥) adalah suku banyak berderajat satu.
Misal, 𝑝(𝑥) = 𝑎𝑥 + 𝑏, 𝑎 ≠ 0, maka
𝑝(𝑥)2 + 𝑝(𝑥 2 ) = 2𝑥 2
⇔ (𝑎𝑥 + 𝑏)2 + (𝑎𝑥 2 + 𝑏) = 2𝑥 2
a

⇔ 𝑎2 𝑥 2 + 2𝑎𝑏𝑥 + 𝑏 2 + 𝑎𝑥 2 + 𝑏 = 2𝑥 2
at

⇔ (𝑎2 + 𝑎)𝑥 2 + 2𝑎𝑏𝑥 + (𝑏 2 + 𝑏) = 2𝑥 2

Sehingga, dari kesamaan suku banyak diperoleh


at

2𝑎𝑏 = 0 ⇒ 𝑎 = 0 (𝑇𝑀) atau 𝑏 = 0


.c

𝑎2 + 𝑎 = 2 ⇒ 𝑎2 + 𝑎 − 2 = 0
⇔ (𝑎 + 2)(𝑎 − 1) = 0
w

⇔ 𝑎 = −2 atau 𝑎 = 1
w

Dari kasus ini 𝑝(𝑥) yang memenuhi hanya jika 𝑏 = 0, sehingga


• Apabila 𝑎 = −2, jadi 𝑝(𝑥) yang memenuhi adalah 𝑝(𝑥) = −2𝑥, sehingga karena 𝑝(1) = −2 ≠ 1,
//w

maka 𝑝(10) = −2(10) = −20.


• Apabila 𝑎 = 1, jadi 𝑝(𝑥) yang memenuhi adalah 𝑝(𝑥) = 𝑥, sehingga karena 𝑝(1) = 1, dan
mengingat 𝑝(1) ≠ 1, maka 𝑝(𝑥) = 𝑥 tidak memenuhi. Sehingga tidak ada nilai 𝑝(10) yang
s:

memenuhi.
tp

Kasus kedua, 𝑝(𝑥) adalah suku banyak berderajat dua.


Misal, 𝑝(𝑥) = 𝑎𝑥 2 + 𝑏𝑥 + 𝑐, 𝑎 ≠ 0 maka
ht

𝑝(𝑥)2 + 𝑝(𝑥 2 ) = 2𝑥 2
⇔ (𝑎𝑥 2 + 𝑏𝑥 + 𝑐)2 + (𝑎𝑥 4 + 𝑏𝑥 2 + 𝑐) = 2𝑥 2
⇔ 𝑎 𝑥 + 𝑎𝑥 + 2𝑎𝑏𝑥 + 𝑏 𝑥 + 2𝑎𝑐𝑥 2 + 𝑏𝑥 2 + 2𝑏𝑐 𝑥 + 𝑐 2 + 𝑐 = 2𝑥 2
2 4 4 3 2 2

⇔ (𝑎2 + 𝑎)𝑥 4 + 2𝑎𝑏𝑥 3 + (𝑏 2 + 2𝑎𝑐 + 𝑏)𝑥 2 + 2𝑏𝑐𝑥 + (𝑐 2 + 𝑐) = 2𝑥 2

Pembahasan OSK Matematika SMA 2018 by Pak Anang (http://pak-anang.blogspot.com)

123
Shared by: www.catatanmatematika.com

Sehingga, dari kesamaan suku banyak diperoleh


𝑎2 + 𝑎 = 0 ⇒ 𝑎(𝑎 + 1) = 0
⇔ 𝑎 = 0 (𝑇𝑀) atau 𝑎 = −1

2𝑎𝑏 = 0 ⇒ 𝑎 = 0 (𝑇𝑀) atau 𝑏 = 0

𝑏 2 + 2𝑎𝑐 + 𝑏 = 2 ⇒ 2𝑎𝑐 = 2

m
⇔ −2𝑐 = 2
⇔ 𝑐 = −1

co
Dari kasus ini 𝑝(𝑥) yang memenuhi adalah 𝑝(𝑥) = −𝑥 2 − 1, sehingga 𝑝(10) = −(10)2 − 1 = −101.

a.
Jadi jumlah semua nilai 𝑝(10) yang mungkin adalah −20 − 101 = −121.

ik
at
em
at
a nm
at
at
.c
w
w
//w
s:
tp
ht

Pembahasan OSK Matematika SMA 2018 by Pak Anang (http://pak-anang.blogspot.com)

124
Shared by: www.catatanmatematika.com

11. Misalkan {𝑥𝑛 } adalah barisan bilangan bulat yang memenuhi 𝑥1 = 𝑥2 = ⋯ = 𝑥12 = 0, 𝑥13 = 2, dan
untuk setiap bilangan asli 𝑛 berlaku 𝑥𝑛+13 = 𝑥𝑛+4 + 2𝑥𝑛 . Nilai 𝑥143 adalah ….

Pembahasan:
Perhatikan, kita akan mencoba menguraikan kombinasi linear dari bilangan 143 terhadap bilangan
9 dan 13, sehingga diperoleh
9𝑝 + 13𝑞 = 143 ⇒ 9𝑝 = 143 − 13𝑞

m
⇔ 9 ∙ 𝑝 = 13 ∙ (11 − 𝑞)

co
Sehingga, terdapat dua penyelesaian bulat untuk 9𝑝 + 13𝑞 = 143, dengan 𝑝, 𝑞 bilangan cacah.
• 𝑝 = 13, sehingga 11 − 𝑞 = 9 ⇒ 𝑞 = 2, sehingga (𝑝1 , 𝑞1 ) = (13, 2)
• 𝑝 = 0, sehingga 11 − 𝑞 = 0 ⇒ 𝑞 = 11, sehingga (𝑝2 , 𝑞2 ) = (0,11)

a.
Padahal, untuk 𝑥𝑛+13 = 𝑥𝑛+4 + 2𝑥𝑛 , rumus umumnya untuk 𝑛 = 9𝑝 + 13𝑞 adalah

ik
𝑘
𝑝 + 𝑞𝑖 − 1
𝑥𝑛 = ∑ 2𝑞𝑖 ∙ ( 𝑖 )
𝑝𝑖

at
𝑖=1

em
Sehingga,
14 10
𝑥𝑛 = 22 ∙ ( ) + 211 ∙ ( )
13 0
= 4 ∙ 14 + 2048 ∙ 1
= 56 + 2048
= 2104
at
nm
Cara alternatif:
Jika kita uraikan secara terus menerus, maka kita juga akan bertemu pola kombinatorik pada suku-
suku yang dihasilkan. Perhatikan,
1 1
a

𝑥143 = 20 ( ) 𝑥134 + 21 ( ) 𝑥130


0 1
at

0 2 1 2 2
= 2 ( ) 𝑥125 + 2 ( ) 𝑥121 + 22 ( ) 𝑥117
0 1 2
=⋯
at

10 10 10 10
= 20 ( ) 𝑥53 + 21 ( ) 𝑥49 + 22 ( ) 𝑥44 + ⋯ + 210 ( ) 𝑥13
0 1 2 10
.c

Jadi, rumus umum penjabaran dari 𝑥143 hingga langkah ke-𝑘 adalah
w

𝑘
𝑘
𝑥𝑛 = ∑ 2𝑖 ( ) 𝑥𝑛−9𝑘−4𝑖
w

𝑖
𝑖=0
//w

Nah, jika kita perhatikan seksama pola yang terbentuk, hanya suku dari penjabaran tersebut harus
kita uraikan menjadi 𝑥13 , jika tidak dapat diuraikan menjadi 𝑥13 maka nilainya nol, mengingat
bahwa 𝑥1 = 𝑥2 = ⋯ = 𝑥12 = 0.
s:

Sehingga, kita akan mencoba menguraikan bilangan 13 dari pengurangan bilangan 143 oleh
tp

kombinasi linear dari 9 dan 4, sehingga diperoleh


13 = 143 − 9(14) − 4(1)
13 = 143 − 9(10) − 4(10)
ht

Jadi, nilai dari 𝑥143 dapat diperoleh untuk (𝑘1 , 𝑖1 ) = (14,1) dan (𝑘2 , 𝑖2 ) = (10,9)
14 10
𝑥143 = 21 ( ) 𝑥13 + 210 ( ) 𝑥13 = 2 ∙ 14 ∙ 2 + 1024 ∙ 1 ∙ 2 = 56 + 2048 = 2104
1 10

Pembahasan OSK Matematika SMA 2018 by Pak Anang (http://pak-anang.blogspot.com)

125
Shared by: www.catatanmatematika.com

12. Untuk setiap bilangan real 𝑧, ⌊𝑧⌋ menyatakan bilangan bulat terbesar yang lebih kecil dari atau sama
dengan 𝑧. Jika diketahui ⌊𝑥⌋ + ⌊𝑦⌋ + 𝑦 = 43,8 dan 𝑥 + 𝑦 − ⌊𝑥⌋ = 18,4. Nilai 10(𝑥 + 𝑦) adalah ….

Pembahasan:
Perhatikan, misal 0 ≤ 𝛿 < 1, maka untuk setiap 𝑧 bilangan real berlaku:
𝑧 = ⌊𝑧⌋ + 𝛿𝑧

m
Dari persamaan ⌊𝑥⌋ + ⌊𝑦⌋ + 𝑦 = 43,8, diperoleh
⌊𝑥⌋ + ⌊𝑦⌋ + 𝑦 = 43,8 ⇒ ⌊𝑥⌋ + ⌊𝑦⌋ + ⌊𝑦⌋ + 𝛿𝑦 = 43,8

co
⇔ ⌊𝑥⌋ + 2⌊𝑦⌋ + 𝛿𝑦 = 43,8

Sehingga diperoleh, ⌊𝑥⌋ + 2⌊𝑦⌋ = 43 dan 𝛿𝑦 = 0,8.

a.
Dan dari persamaan 𝑥 + 𝑦 − ⌊𝑥⌋ = 18,4 diperoleh

ik
𝑥 + 𝑦 − ⌊𝑥⌋ = 18,4 ⇒ ⌊𝑥⌋ + 𝛿𝑥 + ⌊𝑦⌋ + 𝛿𝑦 − ⌊𝑥⌋ = 18,4
⇔ ⌊𝑦⌋ + 𝛿𝑥 + 𝛿𝑦 = 18,4

at
⇔ ⌊𝑦⌋ + 𝛿𝑥 + 0,8 = 18,4
⇔ ⌊𝑦⌋ + 𝛿𝑥 = 17,6

em
Sehingga diperoleh ⌊𝑦⌋ = 17, dan 𝛿𝑥 = 0,6.

at
Perhatikan kembali bahwa ⌊𝑥⌋ + 2⌊𝑦⌋ = 43, sehingga diperoleh
⌊𝑥⌋ + 2⌊𝑦⌋ = 43 ⇒ ⌊𝑥⌋ + 2(17) = 43
⌊𝑥⌋ + 34 = 43
nm

⇔ ⌊𝑥⌋ = 9

Jadi, diperoleh nilai 𝑥 dan 𝑦 adalah


a

𝑥 = ⌊𝑥⌋ + 𝛿𝑥 = 9 + 0,6 = 9,6


𝑦 = ⌊𝑦⌋ + 𝛿𝑦 = 17 + 0,8 = 17,8
at

Jadi, nilai 10(𝑥 + 𝑦) = 10(9,6 + 17,8) = 10(27,4) = 274.


at
.c
w
w
//w
s:
tp
ht

Pembahasan OSK Matematika SMA 2018 by Pak Anang (http://pak-anang.blogspot.com)

126
Shared by: www.catatanmatematika.com

13. Misalkan 𝐴𝐵𝐶𝐷 adalah trapesium siku-siku dengan 𝐴𝐵 sejajar 𝐷𝐶 dan 𝐴𝐵 tegak lurus 𝐴𝐷. Misalkan
juga 𝑃 adalah titik potong diagonal 𝐴𝐶 dan 𝐵𝐷. Jika perbandingan luas segitiga 𝐴𝑃𝐷 dan luas
𝐴𝐵
trapesium 𝐴𝐵𝐶𝐷 adalah 4 : 25 maka nilai 𝐷𝐶 adalah ….

Pembahasan:
Perhatikan trapesium 𝐴𝐵𝐶𝐷 berikut. 𝐷 𝐶
𝑃 adalah titik potong diagonal 𝐴𝐶 dan 𝐵𝐷.

m
𝑄 𝑅
Misal, 𝑃

co
𝐴𝐵
= 𝑚 ⇒ 𝐴𝐵 = 𝑚 ∙ 𝐷𝐶
𝐷𝐶

a.
𝐴 𝐵
Sehingga, dari perbandingan luas segitiga 𝐴𝑃𝐷 dan
trapesium 𝐴𝐵𝐶𝐷 diperoleh

ik
1
[𝐴𝑃𝐷] ∙ 𝐴𝐷 ∙ 𝑃𝑄
= 2

at
[𝐴𝐵𝐶𝐷] 1 (𝐴𝐵 + 𝐷𝐶)
2 ∙ 𝐴𝐷 ∙
4 𝑃𝑄

em
=
25 𝐴𝐵 + 𝐷𝐶
4 𝑃𝑄
=
25 (1 + 𝑚)𝐷𝐶
at
Padahal, dari kesebangunan segitiga 𝐴𝐵𝐷 dan segitiga 𝐶𝐴𝐵, diperoleh 𝑃𝑄 = 𝑃𝑅.
nm
Sedangkan, dari kesebangunan 𝐴𝑃𝑄 dan 𝐴𝐶𝐷 serta 𝐵𝑃𝑅 dan 𝐵𝐷𝐶 diperoleh.

𝐴𝑄 𝑃𝑄 𝐴𝐵 − 𝑃𝑅
a

= =
𝐷𝑄 𝐷𝐶 − 𝑃𝑄 𝑃𝑅
at

Maka,
𝑃𝑄 2 = (𝐷𝐶 − 𝑃𝑄)(𝐴𝐵 − 𝑃𝑄) ⇒ 𝑃𝑄 2 = (𝐷𝐶 − 𝑃𝑄)(𝑚 ∙ 𝐷𝐶 − 𝑃𝑄)
at

⇔ 𝑃𝑄 2 = 𝑚 ∙ 𝐷𝐶 2 − (1 + 𝑚) ∙ 𝐷𝐶 ∙ 𝑃𝑄 + 𝑃𝑄 2
⇔ (1 + 𝑚) ∙ 𝐷𝐶 ∙ 𝑃𝑄 = 𝑚 ∙ 𝐷𝐶 2
.c

𝑃𝑄 𝑚
⇔ =
𝐷𝐶 (1 + 𝑚)
w

Sehingga,
w

4 𝑃𝑄 4 𝑚
= ⇒ =
25 (1 + 𝑚)𝐷𝐶 25 (1 + 𝑚)2
//w

⇔ 4(1 + 𝑚)2 = 25𝑚


⇔ 4 + 8𝑚 + 4𝑚2 = 25𝑚
⇔ 4𝑚2 − 17𝑚 + 4 = 0
s:

⇔ (4𝑚 − 1)(𝑚 − 4) = 0
1
tp

⇔ 𝑚 = atau 𝑚 = 4
4
ht

𝐴𝐵 1 𝐴𝐵
Jadi, perbandingan 𝐷𝐶 = 4 atau 𝐷𝐶 = 4.

Catatan penulis:
Pembuat soal kurang waspada terhadap perbandingan panjang 𝐴𝐵 dan 𝐷𝐶. Seharusnya pada soal
ditambahkan keterangan 𝐴𝐵 > 𝐷𝐶 atau 𝐴𝐵 < 𝐷𝐶.

Pembahasan OSK Matematika SMA 2018 by Pak Anang (http://pak-anang.blogspot.com)

127
Shared by: www.catatanmatematika.com

14. Himpunan 𝑆 merupakan himpunan bilangan-bilangan 7 digit sehingga masing-masing angka 1, 2,


3, 4, 5, 6, atau 7 tepat muncul satu kali. Bilangan-bilangan di 𝑆 diurutkan mulai dari yang paling
kecil sampai yang paling besar. Bilangan yang berapa pada urutan ke-2018 adalah ….

Pembahasan
Pertama, kita akan memeriksa banyak bilangan dengan memeriksa digit pada tempat terbesar,
yaitu tempat jutaan.

m
• Bilangan 1XXXXXX menyumbang sebanyak 6! = 720 bilangan. Jadi bilangan ke-1 sampai
dengan bilangan ke-720 adalah berformat 1XXXXXX.

co
• Bilangan 2XXXXXX menyumbang sebanyak 6! = 720 bilangan. Jadi bilangan ke-721 sampai
dengan bilangan ke-1440 adalah berformat 2XXXXXX.
• Bilangan 3XXXXXX menyumbang sebanyak 6! = 720 bilangan. Jadi bilangan ke-1441 sampai

a.
dengan bilangan ke-2160 adalah berformat 3XXXXXX. Sehingga bilangan ke-2018 berada pada
format 3XXXXXX.

ik
Selanjutnya akan diperiksa digit pada tempat ratusan ribu.

at
• Bilangan 31XXXXX menyumbang sebanyak 5! = 120 bilangan. Jadi bilangan ke-1441 sampai
dengan bilangan ke-1560 adalah berformat 31XXXXX.

em
• Bilangan 32XXXXX menyumbang sebanyak 5! = 120 bilangan. Jadi bilangan ke-1561 sampai
dengan bilangan ke-1680 adalah berformat 32XXXXX.
• Bilangan 34XXXXX menyumbang sebanyak 5! = 120 bilangan. Jadi bilangan ke-1681 sampai
dengan bilangan ke-1800 adalah berformat 34XXXXX.at
• Bilangan 35XXXXX menyumbang sebanyak 5! = 120 bilangan. Jadi bilangan ke-1801 sampai
dengan bilangan ke-1920 adalah berformat 35XXXXX.
nm
• Bilangan 36XXXXX menyumbang sebanyak 5! = 120 bilangan. Jadi bilangan ke-1921 sampai
dengan bilangan ke-2040 adalah berformat 36XXXXX. Sehingga bilangan ke-2018 berada pada
format 36XXXXX.
a

Selanjutnya akan diperiksa digit pada tempat puluhan ribu.


at

• Bilangan 361XXXX menyumbang sebanyak 4! = 24 bilangan. Jadi bilangan ke-1921 sampai


dengan bilangan ke-1944 adalah berformat 361XXXX.
at

• Bilangan 362XXXX menyumbang sebanyak 4! = 24 bilangan. Jadi bilangan ke-1945 sampai


dengan bilangan ke-1968 adalah berformat 362XXXX.
• Bilangan 364XXXX menyumbang sebanyak 4! = 24 bilangan. Jadi bilangan ke-1969 sampai
.c

dengan bilangan ke-1992 adalah berformat 364XXXX.


• Bilangan 365XXXX menyumbang sebanyak 4! = 24 bilangan. Jadi bilangan ke-1993 sampai
w

dengan bilangan ke-2016 adalah berformat 365XXXX.


w

• Bilangan 367XXXX menyumbang sebanyak 4! = 24 bilangan. Jadi bilangan ke-2017 sampai


dengan bilangan ke-2040 adalah berformat 367XXXX. Sehingga bilangan ke-2018 berada pada
//w

format 367XXXX.

Selanjutnya akan diperiksa secara manual digit pada tempat ribuan.


• Bilangan 3671245 adalah bilangan ke-2017.
s:

• Bilangan 3671254 adalah bilangan ke-2018.


tp

Jadi, bilangan ke-2018 adalah 3671254.


ht

Pembahasan OSK Matematika SMA 2018 by Pak Anang (http://pak-anang.blogspot.com)

128
Shared by: www.catatanmatematika.com

TRIK SUPERKILAT:
Perhatikan,
2018 = 2 × 6! + 578
578 = 4 × 5! + 98
98 = 4 × 4! + 2
2 = 0 × 3! + 2
2 = 0 × 2! + 2

m
2 = 1 × 1! + 1
1 = 0 × 0! + 1

co
1234567 3 (bilangan ke 2+1)
124567  6 (bilangan ke 4+1)

a.
12457  7 (bilangan ke 4+1)
1245  1 (bilangan ke 0+1)
245  2 (bilangan ke 0+1)

ik
45  5 (bilangan ke 1+1)
4  4 (bilangan ke 0+1)

at
Jadi, bilangan yang dimaksud adalah 3671254.

em
Catatan Penulis:
Menurut kunci jawaban yang beredar, jawaban untuk soal ini adalah 3561274.
at
Nah, penulis mencoba membuat “analisis forensik” mengapa pembuat soal bisa membuat kunci
jawaban seperti tersebut di atas.
nm
Langkah yang kurang tepat ditunjukkan oleh warna merah.
1234567 3 (bilangan ke 2+1)
124567  5 (bilangan ke 4+1)  pembuat soal lupa bahwa bilangan ke-5 bukan 5, tapi 6.
a

12467  6 (bilangan ke 4+1)  pembuat soal lupa bahwa bilangan ke-5 bukan 6, tapi 7.
1247  1 (bilangan ke 0+1)
at

247  2 (bilangan ke 0+1)


47  7 (bilangan ke 1+1)
at

4  4 (bilangan ke 0+1)
.c

Jadi, bilangan yang dimaksud adalah 3561274. (jawaban akhir menurut kunci jawaban yang dipakai
korektor untuk mengoreksi jawaban peserta)
w
w
//w
s:
tp
ht

Pembahasan OSK Matematika SMA 2018 by Pak Anang (http://pak-anang.blogspot.com)

129
Shared by: www.catatanmatematika.com

15. Misalkan 𝑆 = {𝑥 ∈ ℝ|0 ≤ 𝑥 ≤ 1}. Banyaknya pasangan bilangan asli (𝑎, 𝑏) sehingga tepat ada 2018
𝑥 𝑦
anggota 𝑆 yang dapat dinyatakan dalam bentuk 𝑎 + 𝑏 untuk suatu bilangan bulat 𝑥 dan 𝑦 adalah ….

Pembahasan:
Perhatikan, dari 𝐵𝑒𝑧𝑜𝑢𝑡 ′ 𝑠 𝑇ℎ𝑒𝑜𝑟𝑒𝑚, yaitu “Jika 𝑎 dan 𝑏 dua bilangan bulat yang keduanya taknol
maka terdapat bilangan bulat 𝑥 dan 𝑦 sehingga 𝐹𝑃𝐵(𝑎. 𝑏) = 𝑎𝑥 + 𝑏𝑦”.

m
Sehingga banyaknya anggota 𝑆 yang dapat dinyatakan adalah banyaknya 0 ≤ 𝑥𝑏 + 𝑦𝑎 ≤ 𝑎𝑏; 𝑥𝑏 +
𝑦𝑎 ∈ ℤ yang memiliki solusi 𝑥, 𝑦 ∈ ℤ.

co
Dengan membagi kasus, yaitu

a.
Kasus 1.
Apabila 𝑎 dan 𝑏 adalah relative prima, maka dari 𝐵𝑒𝑧𝑜𝑢𝑡 ′ 𝑠 𝑇ℎ𝑒𝑜𝑟𝑒𝑚, semua 𝑘 ∈ ℤ dapat

ik
dinyatakan dalam 𝑥𝑏 + 𝑦𝑎 sehingga haruslah 𝑎𝑏 + 1 = 2018, sehingga
𝑎𝑏 + 1 = 2018 ⇒ 𝑎𝑏 = 2017

at
Sehingga, diperoleh penyelesaian (𝑎, 𝑏) = {(1,2017), (2017,1)}.

em
Kasus 2.
Apabila 𝐹𝑃𝐵(𝑎, 𝑏) = 𝑑 > 1, maka 𝑎, 𝑏 > 1. Sehingga dari 𝐵𝑒𝑧𝑜𝑢𝑡 ′ 𝑠 𝑇ℎ𝑒𝑜𝑟𝑒𝑚, semua (dan hanya)
𝑎𝑏 𝑏
𝑑𝑘, 𝑘 ∈ ℤ dapat dinyatakan dalam 𝑥𝑏 + 𝑦𝑎. Semua 𝑑𝑘 ada sebanyak ⌊ 𝑑 ⌋ + 1 = 𝑎 (𝑑) + 1, sehingga
𝑏 𝑏 at
𝑎 ( ) + 1 = 2018 ⇒ 𝑎 ( ) = 2017 ⇒ 𝑎 = 2017
𝑑 𝑑
nm
Sehingga diperoleh penyelesaian (𝑎, 𝑏) = {(2017,2017)}

Jadi, banyaknya pasangan bilangan asli (𝑎, 𝑏) yang memenuhi adalah 3 buah.
a
at
at
.c
w
w
//w
s:
tp
ht

Pembahasan OSK Matematika SMA 2018 by Pak Anang (http://pak-anang.blogspot.com)

130
Shared by: www.catatanmatematika.com

16. Diberikan segitiga 𝐴𝐵𝐶 dan lingkaran Γ yang berdiameter 𝐴𝐵. Lingkaran Γ memotong sisi
1
𝐴𝐶 dan 𝐵𝐶 berturut-turut di 𝐷 dan 𝐸. Jika 𝐴𝐵 = 30, 𝐴𝐷 = 3 𝐴𝐶, dan
1
𝐵𝐸 = 4 𝐵𝐶, maka luas segitiga 𝐴𝐵𝐶 adalah ….

Pembahasan:
Perhatikan, gambar segitiga 𝐴𝐵𝐶 dan lingkaran Γ.

m
𝐶
Misal,

co
1
𝐴𝐷 = 𝑥, karena 𝐴𝐷 = 3 𝐴𝐶 maka 𝐴𝐶 = 3𝐴𝐷,
3𝑦 sehingga 𝐴𝐶 = 3𝑥, akibatnya 𝐶𝐷 = 2𝑥.

a.
2𝑥
1
𝐷 𝐵𝐸 = 𝑦, karena 𝐵𝐸 = 4 𝐵𝐶 maka 𝐵𝐶 = 4𝐵𝐸,

ik
𝐸
sehingga 𝐵𝐶 = 4𝑦, akibatnya 𝐶𝐸 = 3𝑦.

at
𝑥 𝑦

𝐴 𝐵 Berdasarkan power of point diperoleh


30

em
𝐶𝐷 × 𝐶𝐴 = 𝐶𝐸 × 𝐶𝐵
⇔ 2𝑥 × 3𝑥 = 3𝑦 × 4𝑦
⇔ 6𝑥 2 = 12𝑦 2
at ⇔ 𝑥 2 = 2𝑦 2

Luas ∆𝐴𝐵𝐶 dapat dicari menggunakan sisi alas 𝐴𝐶 dan tinggi 𝐵𝐷. Sehingga kita akan
nm
mencari 𝐴𝐶 dengan terlebih dahulu mencari nilai 𝐴𝐷, lalu mencari 𝐵𝐷 dengan
menggunakan aturan Pythagoras pada ∆𝐴𝐵𝐷.
a

𝐵𝐷 dapat dicari dengan memandang aturan Pythagoras pada ∆𝐴𝐵𝐷 dan ∆𝐵𝐷𝐶, yaitu:
at

𝐵𝐷2 = 𝐵𝐷2
⇔ 𝐴𝐵 − 𝐴𝐷2 = 𝐵𝐶 2 − 𝐶𝐷2
2
at

⇔ 302 − 𝑥 2 = (4𝑦)2 − (2𝑥)2


⇔ 900 − 𝑥 2 = 16𝑦 2 − 4𝑥 2
.c

⇔ 900 − 𝑥 2 = 8𝑥 2 − 4𝑥 2
⇔ 900 = 5𝑥 2
w

⇔ 𝑥 2 = 180
w

Sehingga, 𝐵𝐷 = √𝐴𝐵 2 − 𝐴𝐷2 = √302 − 𝑥 2 = √900 − 180 = √720 = 12√5.


//w

Mengingat 𝐴𝐷 = 𝑥 = √180 = 6√5, maka 𝐴𝐶 = 3𝐴𝐷 = 3𝑥 = 18√5.


s:

1 1
Jadi, luas ∆𝐴𝐵𝐶 adalah 𝐿 = 2 × 𝐴𝐶 × 𝐵𝐷 = 2 × 18√5 × 12√5 = 108 × 5 = 540.
tp
ht

Pembahasan OSK Matematika SMA 2018 by Pak Anang (http://pak-anang.blogspot.com)

131
Shared by: www.catatanmatematika.com

TRIK SUPERKILAT:
Perhatikan, gambar segitiga 𝐴𝐵𝐶 dan lingkaran Γ.

𝐶 Perhatikan, dengan dalil de Ceva pada segitiga 𝐴𝐵𝐶


diperoleh
𝐴𝐹 𝐵𝐸 𝐶𝐷 𝐴𝐹 3
× × =1⇒ =

m
3𝑦 𝐹𝐵 𝐸𝐶 𝐷𝐴 𝐹𝐵 2
2𝑥
3 3
Jadi, 𝐴𝐹 = 5 𝐴𝐵 = 5 × 30 = 18.

co
𝐷
𝐸
𝑥 𝑦 Berdasarkan power of point diperoleh

a.
𝐴𝐷 × 𝐴𝐶 = 𝐴𝐹 × 𝐴𝐵
𝐴 𝐹 𝐵 1 2 3 2

ik
⇔ 𝐴𝐶 = 𝐴𝐵
3 5
9

at
⇔ 𝐴𝐶 2 = (30)2
5
𝐴𝐶 2 = 1620

em

Sehingga, 𝐶𝐹 = √𝐴𝐶 2 − 𝐴𝐹 2 = √1620 − 324 = √1296 = 36.


1 1
at
Jadi, luas ∆𝐴𝐵𝐶 adalah 𝐿 = 2 × 𝐴𝐵 × 𝐶𝐹 = 2 × 30 × 36 = 540.
a nm
at
at
.c
w
w
//w
s:
tp
ht

Pembahasan OSK Matematika SMA 2018 by Pak Anang (http://pak-anang.blogspot.com)

132
Shared by: www.catatanmatematika.com

1 𝑥 𝑥 2𝑦
17. Diberikan bilangan real 𝑥 dan 𝑦 yang memenuhi 2 < 𝑦 < 2. Nilai minimum 2𝑦−𝑥 + 2𝑥−𝑦 adalah ….

Pembahasan:
𝑥
𝑥 𝑥 2𝑦 ( ) 2 𝑥 𝑝 2
𝑦
Perhatikan, 𝑓 (𝑦) = 2𝑦−𝑥
+ 2𝑥−𝑦
= 𝑥 +𝑥 , dan misal 𝑦 = 𝑝, maka 𝑓(𝑝) = 2−𝑝 + 2𝑝−1.
2−( ) 2( )−1
𝑦 𝑦
2 4 2(2𝑝−1) −4(2−𝑝)2
2 4𝑝2 +8𝑝−14
Sehingga, 𝑓 ′ (𝑝) = (2−𝑝)2 − (2𝑝−1)2 = = (2−𝑝)2 (2𝑝−1)2

m
(2−𝑝)2 (2𝑝−1)2

Diperoleh titik stasioner 𝑓(𝑝) adalah saat 𝑓 ′ (𝑝) = 0, yaitu

co
4𝑝2 + 8𝑝 − 14
𝑓 ′ (𝑝) = 0 ⇒ =0
(2 − 𝑝)2 (2𝑝 − 1)2

a.
⇔ 4𝑝2 + 8𝑝 − 14 = 0
7
𝑝2 + 2𝑝 − = 0

ik

2
9

at
2
⇔ 𝑝 + 2𝑝 + 1 =
2
9
(𝑝 + 1)2 =

em

2
3
⇔ 𝑝+1 = ±
√2

at 𝑝 = −1 ±
3
√2
Perhatikan garis bilangan dari 𝑓 ′ (𝑝)
nm
+ − +

3 3
a

−1 − −1 +
√2 √2
at

3 3√2−2 1
at

Sehingga, 𝑝 = −1 + = adalah titik balik minimum dan karena < 𝑝 < 2 maka nilai
√2 2 2
3
minimum dari 𝑓(𝑝) adalah 𝑓 (−1 + ), yaitu
.c

√2
3√2 − 2
( 2 )
3 2
w

𝑓 (−1 + )= +
√2 3√2 − 2 3√2 − 2
2−( 2 ) 2( 2 )−1
w

3√2 − 2 2
//w

= +
6 − 3√2 3√2 − 3
3√2 − 2 2√2
= +
s:

6 − 3√2 6 − 3√2
5√2 − 2
tp

=
6 − 3√2
5√2 − 2 6 + 3√2
ht

= ×
6 − 3√2 6 + 3√2
18 + 24√2
=
18
4√2
=1+
3

Pembahasan OSK Matematika SMA 2018 by Pak Anang (http://pak-anang.blogspot.com)

133
Shared by: www.catatanmatematika.com

TRIK SUPERKILAT 1:
2𝑛+𝑚 2𝑚+𝑛
Misal, 𝑚 = 2𝑦 − 𝑥 dan 𝑛 = 2𝑥 − 𝑦, maka diperoleh 𝑥 = dan 𝑦 = .
3 3

Sehingga,
1 𝑥 1 2𝑛 + 𝑚
< <2⇒ < < 2 ⇒ 𝑚 > 0, 𝑛 > 0
2 𝑦 2 2𝑚 + 𝑛
Dan, diperoleh

m
𝑥 2𝑦 2𝑛 + 𝑚 2(2𝑚 + 𝑛) 2𝑛 4𝑚
𝑓(𝑥, 𝑦) = + ⇒ 𝑓(𝑚, 𝑛) = + = + +1
2𝑦 − 𝑥 2𝑥 − 𝑦 3𝑚 3𝑛 3𝑚 3𝑛

co
𝑚 𝑛
Karena 𝑚 > 0 dan 𝑛 > 0, sehingga > 0 dan > 0, maka menurut 𝐴𝑀 − 𝐺𝑀, diperoleh
𝑛 𝑚

a.
2𝑚 4𝑛
+
3𝑛 3𝑚 ≥ √2𝑚 ∙ 4𝑛

ik
2 3𝑛 3𝑚
2𝑚 4𝑛 4√2

at
⇔ + ≥
3𝑛 3𝑚 3
2𝑚 4𝑛 4√2

em
⇔ + +1≥ 1+
3𝑛 3𝑚 3
𝑥 2𝑦 4√2
Jadi, nilai minimum 2𝑦−𝑥 + 2𝑥−𝑦 adalah 1 + .

TRIK SUPERKILAT 2:
3 at
nm
Perhatikan,
𝑥 2𝑦 𝑥 1 2𝑦 2 2 2𝑥 − 𝑦 4 2𝑦 − 𝑥
𝑓(𝑥, 𝑦) = + =( − )+( − )+1 = ( )+ ( )+1
2𝑦 − 𝑥 2𝑥 − 𝑦 2𝑦 − 𝑥 3 2𝑥 − 𝑦 3 3 2𝑦 − 𝑥 3 2𝑥 − 𝑦
a

1 𝑥
Karena < < 2, maka 2𝑥 − 𝑦 > 0 dan 2𝑦 − 𝑥 > 0.
at

2 𝑦
2𝑥−𝑦 2𝑦−𝑥
Sehingga, untuk > 0 dan > 0 , maka menurut 𝐴𝑀 − 𝐺𝑀, diperoleh
2𝑦−𝑥 2𝑥−𝑦
at

2 2𝑥 − 𝑦 4 2𝑦 − 𝑥
3 (2𝑦 − 𝑥 ) + 3 (2𝑥 − 𝑦) 2 2𝑥 − 𝑦 4 2𝑦 − 𝑥
≥√ ( )∙ ( )
.c

2 3 2𝑦 − 𝑥 3 2𝑥 − 𝑦
2 2𝑥 − 𝑦 4 2𝑦 − 𝑥 4√2
w

⇔ ( )+ ( )≥
3 2𝑦 − 𝑥 3 2𝑥 − 𝑦 3
w

2 2𝑥 − 𝑦 4 2𝑦 − 𝑥 4√2
⇔ ( )+ ( )+1≥ 1+
3 2𝑦 − 𝑥 3 2𝑥 − 𝑦 3
//w

𝑥 2𝑦 4√2
⇔ + ≥ 1+
2𝑦 − 𝑥 2𝑥 − 𝑦 3
s:

𝑥 2𝑦 4√2
Jadi, nilai minimum 2𝑦−𝑥 + 2𝑥−𝑦 adalah 1 + 3
.
tp
ht

Pembahasan OSK Matematika SMA 2018 by Pak Anang (http://pak-anang.blogspot.com)

134
Shared by: www.catatanmatematika.com

18. Diberikan sembilan titik pada bidang yang membentuk segitiga sama sisi seperti pada gambar. Pada
tiap sisi, dua titik yang bukan titik sudut segitiga membagi sisi menjadi tiga bagian sama panjang.
Kesembilan titik ini akan diwarnai masing-masing dengan warna merah atau biru. Peluang bahwa
dari kesembilan titik tersebut, terdapat tiga titik yang warnanya sama dan membentuk segitiga
siku-siku adalah ….

m
co
a.
ik
at
Pembahasan:
Perhatikan gambar berikut!

em
𝑃

𝐴 𝐹 at
nm
𝐵 𝐸

𝑄 𝐶 𝐷 𝑅
a

Dari keenam titik yang bukan titik sudut segitiga dapat dibuat sebuah lingkaran yang di dalamnya
at

terdapat segienam beraturan.


at

Sepasang titik sudut segienam beraturan yang saling berhadapan dapat membentuk garis yang
merupakan diameter lingkaran, yaitu 𝐴𝐷, 𝐵𝐸, dan 𝐶𝐹.
.c

Sehingga, apabila sepasang titik sudut yang berhadapan memiliki warna yang sama, maka jika satu
w

titik dipilih dari empat titik yang lain pada lingkaran berwarna sama, maka jelas tiga titik berwarna
sama tersebut akan terbentuk segitiga siku-siku. Ingat kembali bahwa sudut keliling yang
w

menghadap ke diameter lingkaran pastilah siku-siku.


//w

Sekarang, coba perhatikan bahwa kondisi terburuk yang mungkin terjadi adalah dua pasang titik
sudut segienam beraturan yang saling berhadapan memiliki warna berbeda. Misalnya, 𝐴 dan 𝐷
berwarna merah, sedangkan 𝐵 dan 𝐸 berwarna biru, maka jika satu saja titik yang lain dari 𝐶 atau
s:

𝐹 diberi warna apapun, pastilah akan terbentuk segitiga siku-siku dengan titik-titik sudutnya
sewarna.
tp

Kondisi terburuk lain yang mungkin terjadi adalah 𝐴, 𝐵, 𝐶 dan 𝐷, 𝐸, 𝐹 berlainan warna, maka jika
ht

satu saja titik sudut segitiga 𝑄, 𝑅 diberi warna apapun, pastilah akan terbentuk segitiga siku-siku
dengan titik-titik sudutnya sewarna.

Sehingga peluang bahwa dari kesembilan titik tersebut, terdapat tiga titik yang warnanya sama dan
membentuk segitiga siku-siku adalah 1.

Pembahasan OSK Matematika SMA 2018 by Pak Anang (http://pak-anang.blogspot.com)

135
Shared by: www.catatanmatematika.com

19. Bilangan prima terbesar yang dapat dinyatakan dalam bentuk 𝑎4 + 𝑏 4 + 13 untuk suatu bilangan-
bilangan prima 𝑎 dan 𝑏 adalah ….

Pembahasan:
Perhatikan, teorema tentang bilangan prima yaitu “Setiap bilangan prima 𝑝 dan 𝑝 > 3, maka 𝑝 dapat
dinyatakan sebagai 𝑝 = 6𝑛 ± 1, dengan 𝑛 adalah bilangan asli”.

m
Untuk 𝑎 > 3, 𝑏 > 3, berarti 𝑏 = 6𝑛 ± 1, 𝑛 ≥ 1, maka 𝑎4 + 𝑏 4 + 13 ≡ 1 + 1 + 13 ≡ 15 ≡ 0(mod 3)

co
Untuk 𝑎 = 2, 𝑏 ≤ 3, maka 𝑎4 + 𝑏 4 + 13 ≡ 1 + 1 + 13 ≡ 15 ≡ 0(mod 5)
Jadi untuk 𝑎 = 2, 𝑏 ≤ 3, maka 𝑎4 + 𝑏 4 + 13 adalah bukan bilangan prima.
Begitu pula untuk 𝑏 = 2, 𝑎 ≤ 3, maka 𝑎4 + 𝑏 4 + 13 adalah bukan bilangan prima.

a.
Untuk 𝑎 = 𝑏 = 3, maka maka 𝑎4 + 𝑏 4 + 13 ≡ 1 + 1 + 13 ≡ 15 ≡ 0(mod 5)

ik
Jadi untuk 𝑎 = 𝑏 = 3, maka 𝑎4 + 𝑏 4 + 13 adalah bukan bilangan prima.

at
Untuk 𝑎 = 2, 𝑏 > 3, berarti 𝑏 = 6𝑛 ± 1, 𝑛 ≥ 1, maka 𝑎4 + 𝑏 4 + 13 ≡ 0 + 1 + 13 ≡ 14 ≡ 0(mod 2)
Jadi untuk 𝑎 = 2, 𝑏 > 3, maka 𝑎4 + 𝑏 4 + 13 adalah bukan bilangan prima.

em
Begitu pula untuk 𝑏 = 2, 𝑎 > 3, maka 𝑎4 + 𝑏 4 + 13 adalah bukan bilangan prima.

Untuk 𝑎 = 3, 𝑏 > 3, berarti 𝑏 = 6𝑛 ± 1, 𝑛 ≥ 1,


• Untuk 𝑏 = 6𝑛 + 1, maka misal 𝑛 berbentuk 5𝑘 + 𝑎, dengan 𝑘 ≥ 0. Disini, 𝑎 ≠ 4 sebab jika
𝑎 = 4, maka 𝑏 tak prima.
at
Maka, 𝑎4 + 𝑏 4 + 13 ≡ (81 + 6(5𝑘 + 𝑎) + 1)4 + 13) (mod 5)
nm
≡ ((𝑎 + 1)4 + 4) (mod 5)
≡ 0 (mod 5), untuk 𝑎 ≠ 4
• Untuk 𝑏 = 6𝑛 − 1, maka misal 𝑛 berbentuk 5𝑘 + 𝑎, dengan 𝑘 ≥ 0. Disini, 𝑎 ≠ 1 sebab jika
𝑎 = 1, maka 𝑏 tak prima, kecuali untuk 𝑛 = 1.
a

Maka, 𝑎4 + 𝑏 4 + 13 ≡ (81 + 6(5𝑘 + 𝑎) − 1)4 + 13) (mod 5)


at

≡ ((𝑎 + 1)4 − 1) (mod 5)


≡ 0 (mod 5), untuk 𝑎 ≠ 1
at

Maka, solusi satu-satunya adalah jika 𝑛 = 1, sehingga 𝑎4 + 𝑏 4 + 13 adalah bilangan prima


terbesar untuk 𝑎 = 3, 𝑏 = 5.
.c

Jadi, 𝑎4 + 𝑏 4 + 13 = 34 + 54 + 13 = 81 + 625 + 13 = 719.


w
w
//w
s:
tp
ht

Pembahasan OSK Matematika SMA 2018 by Pak Anang (http://pak-anang.blogspot.com)

136
Shared by: www.catatanmatematika.com

20. Pada segitiga 𝐴𝐵𝐶, panjang sisi 𝐵𝐶 adalah 1 satuan. Ada tepat satu titik 𝐷 pada sisi 𝐵𝐶 yang
memenuhi |𝐷𝐴|2 = |𝐷𝐵| ∙ |𝐷𝐶|. Jika 𝑘 menyatakan keliling 𝐴𝐵𝐶, jumlah semua 𝑘 yang mungkin
adalah ….

Pembahasan:
Perhatikan,
𝐴(𝑥, 𝑦)
|𝐴𝐶| = 𝑝 = √(𝑥 − 1)2 + 𝑦 2 = √(𝑥 2 + 𝑦 2 ) − 2𝑥 + 1

m
|𝐴𝐵| = 𝑞 = √𝑥 2 + 𝑦 2
|𝐵𝐶| = 1

co
|𝐷𝐵| = 𝑟
|𝐷𝐶| = 1 − 𝑟

a.
|𝐷𝐴| = √(𝑥 − 𝑟)2 + 𝑦 2 .

Sehingga, keliling ∆𝐴𝐵𝐶 adalah

ik
𝑘 = |𝐵𝐶| + |𝐴𝐶| + |𝐴𝐵|
𝐵(0,0) 𝐷(𝑟, 0) 𝐶(1,0) .
=1+𝑝+𝑞

at
Perhatikan juga bahwa pada ∆𝐴𝐵𝐶 berlaku

em
|𝐷𝐴|2 = |𝐷𝐵| ∙ |𝐷𝐶|
⇔ (𝑥 − 𝑟)2 + 𝑦 2 = 𝑟(1 − 𝑟)
⇔ 𝑥 2 − 2𝑥𝑟 + 𝑟 2 + 𝑦 2 = 𝑟 − 𝑟 2
at
⇔ 2𝑟 − (2𝑥 + 1)𝑟 + (𝑥 2 + 𝑦 2 ) = 0
2
nm
Ingat, bahwa agar tepat diperoleh satu titik 𝐷(𝑟, 0) maka penyelesaian 𝑟 real kembar (𝐷 = 0)
𝐷=0⇒ 𝑏 2 − 4𝑎𝑐 = 0
2
⇔ (−(2𝑥 + 1)) − 4(2)(𝑥 2 + 𝑦 2 ) = 0
a

⇔ (2𝑥 + 1)2 − 8(𝑥 2 + 𝑦 2 ) = 0


(2𝑥 + 1)2
at

⇔ = (𝑥 2 + 𝑦 2 )
8
at

Padahal, 0 < 𝑟 < 1, sehingga


2𝑥 + 1
.c

0<𝑟<1⇒ 0< <1


4
⇔ 0 < 2𝑥 + 1 < 4
w

⇔ −1 < 2𝑥 < 3
1 3
⇔ − <𝑥<
w

2 2
//w

Jadi, keliling ∆𝐴𝐵𝐶 adalah


𝑘 =1+𝑝+𝑞
= 1 + √(𝑥 2 + 𝑦 2 ) − 2𝑥 + 1 + √(𝑥 2 + 𝑦 2 )
s:

(2𝑥 + 1)2 (2𝑥 + 1)2


=1+√ − 2𝑥 + 1 + √
tp

8 8

(2𝑥 − 3)2 (2𝑥 + 1)2


ht

=1+√ +√
8 8
(2𝑥 − 3) 2𝑥 + 1
= 1 + (− )+
2√2 2√2
= 1 + √2

Pembahasan OSK Matematika SMA 2018 by Pak Anang (http://pak-anang.blogspot.com)

137

Anda mungkin juga menyukai